Sei sulla pagina 1di 110

KATZUNG END-OF-CHAPTER QUESTIONS (B) The half-life is the same regardless of the plasma

concentration
QUESTIONS (C) The drug is largely metabolized in the liver after
1. A 3-year-old is brought to the emergency oral administration and has low bioavailability
department having just ingested a large overdose of (D) The rate of elimination is proportional to the rate
tolbutamide, an oral antidiabetic drug. Tolbutamide of administration at all times
is a weak acid with a pKa of 5.3. It is capable of (E) The drug is distributed to only 1 compartment
entering most tissues, including the brain. On outside the vascular system
physical examination, the heart rate is 100/min, 5. The pharmacokinetics of a new drug are under
blood pressure 90/50 mm Hg, and respiratory rate study in a phase 1 clinical trial. Which statement
20/min. Which of the following statements about about the distribution of drugs to specific tissues is
this case of tolbutamide overdose is most correct? most correct?
(A) Urinary excretion would be accelerated by (A) Distribution to an organ is independent of blood
administration of NH4Cl, an acidifying agent flow
(B) Urinary excretion would be accelerated by giving (B) Distribution is independent of the solubility of
NaHCO3, an alkalinizing agent the drug in that tissue
(C) Less of the drug would be ionized at blood pH (C) Distribution into a tissue depends on the
than at stomach pH unbound drug concentration gradient between
(D) Absorption of the drug would be slower from the blood and the tissue
stomach than from the small intestine (D) Distribution is increased for drugs that are
(E) Hemodialysis is the only effective therapy strongly bound to plasma proteins
2. Botulinum toxin is a large protein molecule. Its (E) Distribution has no effect on the half-life of the
action on cholinergic transmission depends on an drug
intracellular action within nerve endings. Which one 6. The pharmacokinetic process or property that
of the following processes is best suited for distinguishes the elimination of ethanol and high
permeation of very large protein molecules into doses of phenytoin and aspirin from the elimination
cells? of most other drugs is called
(A) Aqueous diffusion (A) Distribution
(B) Endocytosis (B) Excretion
(C) First-pass effect (C) First-pass effect
(D) Lipid diffusion (D) First-order elimination
(E) Special carrier transport (E) Zero-order elimination
3. A 12-year-old child has bacterial pharyngitis and is
to receive an oral antibiotic. She complains of a sore 7. A new drug was administered intravenously, and
throat and pain on swallowing. The tympanic its plasma levels were measured for several hours. A
membranes are slightly reddened bilaterally, but she graph was prepared as shown below, with the
does not complain of earache. Blood pressure is plasma levels plotted on a logarithmic ordinate and
105/70 mm Hg, heart rate 100/mm, temperature time on a linear abscissa. It was concluded that the
37.8 °C (100.1 °F). Ampicillin is a weak organic acid drug has first-order kinetics. From this graph, what is
with a pKa of 2.5. What percentage of a given dose the best estimate of the half-life?
will be in the lipid-soluble form in the duodenum at
a pH of 4.5?
(A) About 1%
(B) About 10%
(C) About 50%
(D) About 90%
(E) About 99%
4. Ampicillin is eliminated by first-order kinetics.
Which of the following statements best describes
(A) 0.5 h
the process by which the plasma concentration of
(B) 1 h
this drug declines?
(C) 3 h
(A) There is only 1 metabolic path for drug
(D) 4 h
elimination
(E) 7 h (D) A group of 1000–5000 subjects with a clinical
8. A large pharmaceutical company has conducted condition requiring analgesia
extensive animal testing of a new drug for the (E) Prior submission of an NDA (new drug
treatment of advanced prostate cancer. The chief of application) to the FDA
research and development recommends that the 12. Which of the following statements about the
company now submit an IND application in order to testing of new compounds for potential therapeutic
start clinical trials. Which of the following use in the treatment of hypertension is most
statements is most correct regarding clinical trials of correct?
new drugs? (A) Animal tests cannot be used to predict the types
(A) Phase 1 involves the study of a small number of of clinical toxicities that may occur because there is
normal volunteers by highly trained clinical no correlation with human toxicity
pharmacologists (B) Human studies in normal individuals will be done
(B) Phase 2 involves the use of the new drug in a before the drug is used in individuals with
large number of patients (1000–5000) who have the hypertension
disease to be treated under conditions of proposed (C) The degree of risk must be assessed in at least 3
use (eg, outpatients) species of animals, including 1 primate species
(C) Chronic animal toxicity studies must be complete (D) The animal therapeutic index must be known
and reported in the IND before trial of the agents in humans
(D) Phase 4 involves the detailed study of toxic 13. The Ames test is frequently carried out before
effects that have been discovered in phase 3 clinical trials are begun. The Ames test is a method
(E) Phase 2 requires the use of a positive control (a that detects
known effective drug) and a placebo (A) Carcinogenesis in primates
9. Which of the following statements about animal (B) Carcinogenesis in rodents
testing of potential new therapeutic agents is most (C) Mutagenesis in bacteria
correct? (D) Teratogenesis in any mammalian species
(A) Extends at least 3 years to discover late toxicities (E) Teratogenesis in primates
(B) Requires at least 1 primate species (eg, rhesus 14. Which of the following statements about new
monkey) drug development is most correct?
(C) Requires the submission of histopathologic slides (A) Drugs that test positive for teratogenicity,
and specimens to the FDA for evaluation by mutagenicity, or carcinogenicity can be tested in
government scientists humans
(D) Has good predictability for drug allergy-type (B) Food supplements and herbal (botanical)
reactions remedies are subject to the same FDA regulation as
(E) May be abbreviated in the case of some very ordinary drugs
toxic agents used in cancer (C) All new drugs must be studied in at least 1
10. The “dominant lethal” test involves the primate species before NDA submission
treatment of a male adult animal with a chemical (D) Orphan drugs are drugs that are no longer
before mating; the pregnant female is later produced by the original manufacturer
examined for fetal death and abnormalities. The (E) Phase 4 (surveillance) is the most rigidly
dominant lethal test therefore is a test of regulated phase of clinical drug trials
(A) Teratogenicity
(B) Mutagenicity ANSWERS
(C) Carcinogenicity 1.
(D) Sperm viability Questions that deal with acid-base (Henderson-
11. Which of the following would probably not be Hasselbalch) manipulations are common on
included in an optimal phase 3 clinical trial of a new examinations. Since absorption involves permeation
analgesic drug for mild pain? across lipid membranes, we can in theory treat an
(A) A negative control (placebo) overdose by decreasing absorption from the gut and
(B) A positive control (current standard analgesic reabsorption from the tubular urine by making the
therapy) drug less lipid-soluble. Ionization attracts water
(C) Double-blind protocol (in which neither the molecules and decreases lipid solubility.
patient nor immediate observers of the patient Tolbutamide is a weak acid, which means that it is
know which agent is active)
less ionized when protonated, ie, at acid pH. Choice This is a straightforward question of
C suggests that the drug would be less ionized at pH pharmacokinetic distribution concepts. From the list
7.4 than at pH 2.0, which is clearly wrong for weak of determinants of drug distribution given on page 6,
acids. Choice D says (in effect) that the more ionized choice C is correct.
form is absorbed faster, which is incorrect. A and B 6.
are opposites because NH4Cl is an acidifying salt and The excretion of most drugs follows first-order
sodium bicarbonate an alkalinizing one. (From the kinetics. However, ethanol and, in higher doses,
point of view of test strategy, opposites in a list of aspirin and phenytoin follow zero-order kinetics;
answers always deserve careful attention.) E is a that is, their elimination rates are constant
distracter. Because an alkaline environment favors regardless of blood concentration. The answer is E.
ionization of a weak acid, we should give 7.
bicarbonate. The answer is B. Note that clinical Drugs with first-order kinetics have constant half-
management of overdose involves many other lives, and when the log of the concentration in a
considerations in addition to trapping the drug in body compartment is plotted versus time, a straight
urine; manipulation of urine pH may be line results. The half-life is defined as the time
contraindicated for other reasons. required for the concentration to decrease by 50%.
2. As shown in the graph, the concentration decreased
Endocytosis is an important mechanism for transport from 16 units at 1 h to 8 units at 4 h and 4 units at 7
of very large molecules across membranes. Aqueous h; therefore, the half-life is 7 h minus 4 h or 3 h. The
diffusion is not involved in transport across the lipid answer is C.
barrier of cell membranes. Lipid diffusion and special 8.
carrier transport are common for smaller molecules. Except for known toxic drugs (eg, cancer
The first-pass effect has nothing to do with the chemotherapy drugs), phase 1 is carried out in 25–
mechanisms of permeation; rather, it denotes drug 50 normal volunteers. Phase 2 is carried out in
metabolism or excretion before absorption into the several hundred closely monitored patients with the
systemic circulation. The answer is B. disease. Results of chronic toxicity studies in animals
3. are required in the NDA and are usually underway at
U.S. Medical Licensing Examination (USMLE)-type the time of IND submission. However, they do not
questions often contain a lengthy clinical description have to be completed and reported in the IND.
in the stem. One can often determine the relevance Phase 4 is the general surveillance phase that follows
of the clinical data by scanning the last sentence in marketing of the new drug. It is not targeted at
the stem and the list of answers, see Appendix IV. In specific effects. Positive controls and placebos are
this question, the emphasis is clearly on not a rigid requirement of any phase of clinical trials,
pharmacokinetic principles. Ampicillin is an acid, so it although placebos are often used in phase 2 and
is more ionized at alkaline pH and less ionized at phase 3 studies. The answer is A.
acidic pH. The Henderson-Hasselbalch equation 9.
predicts that the ratio changes from 50/50 at the pH Drugs proposed for short-term use may not require
equal to the pKa to 1/10 (protonated/unprotonated) long-term chronic testing. For some drugs, no
at 1 pH unit more alkaline than the pKa and 1/100 at primates are used; for other agents, only 1 species is
2 pH units more alkaline. For acids, the protonated used. The data from the tests, not the evidence
form is the nonionized, more lipid-soluble form. The itself, must be submitted to the FDA. Prediction of
answer is A. human drug allergy from animal testing is useful but
4. not definitive (see answer 12). The answer is E.
“First-order” means that the elimination rate is 10.
proportional to the concentration perfusing the The description of the test indicates that a
organ of elimination. The half-life is a constant. The chromosomal change (passed from father to fetus) is
rate of elimination is proportional to the rate of the toxicity detected. This is a mutation. The answer
administration only at steady state. The order of is B.
elimination is independent of the number of 11.
compartments into which a drug distributes. The The first 4 items (A–D) are correct; they would be
answer is B. included. An NDA cannot be acted upon until the
5. first 3 phases of clinical trials have been completed.
(The IND must be approved before clinical trials can 2. Graded and quantal dose-response curves are
be conducted.) The answer is E. being used for evaluation of a new antiasthmatic
12. drug in the animal laboratory and in clinical trials.
Animal tests in a single species do not always predict Which of the following statements best describes
human toxicities. However, when these tests are graded dose-response curves?
carried out in several species, most acute toxicities (A) More precisely quantitated than quantal dose-
that occur in humans also appear in at least 1 animal response curves
species. According to current FDA rules, the “degree (B) Obtainable from isolated tissue preparations but
of risk” must be determined in at least 2 species. Use not from the study of intact subjects
of primates is not always required. The therapeutic (C) Used to determine the maximal efficacy of the
index is not required. Except for cancer drug
chemotherapeutic agents and antivirals used in (D) Used to determine the therapeutic index of the
AIDS, phase 1 clinical trials are carried out in normal drug
subjects. The answer is B. (E) Used to determine the variation in sensitivity of
13. subjects to the drug
The Ames test is carried out in Salmonella and 3. Prior to clinical trials in patients with heart failure,
detects mutations in the bacterial DNA. Because an animal study was carried out to compare two
mutagenic potential is associated with carcinogenic new positive inotropic drugs (A and B) to a current
risk for many chemicals, a positive Ames test is often standard agent (C). The results of cardiac output
used to suggest that a particular agent may be a measurements are shown in the graph below. Which
carcinogen. However, the test itself only detects of the following statements is correct?
mutations. The answer is C.
14.
Food supplements and botanicals are much more
loosely regulated than conventional drugs. Primates
are not required in any phase of new drug testing,
although they are sometimes used. Orphan drugs
are those for which the anticipated patient
population is smaller than 200,000 patients in the (A) Drug A is most effective
United States. Phase 4 surveillance is the most (B) Drug B is least potent
loosely regulated phase of clinical trials. Many drugs (C) Drug C is most potent
in current clinical use test positive for teratogenicity, (D) Drug B is more potent than drug C and more
effective than drug A
mutagenicity, or carcinogenicity. Such drugs are
(E) Drug A is more potent than drug B and more
usually labeled with warnings about these toxicities effective than drug C
and, in the case of teratogenicity, are labeled as 4. A study was carried out in isolated intestinal
contraindicated in pregnancy. The answer is A. smooth muscle preparations to determine the action
of a new drug “novamine,” which in separate studies
QUESTIONS bound to the same receptors as acetylcholine. In the
1. A 55-year-old woman with hypertension is to be absence of other drugs, acetylcholine caused
treated with a thiazide diuretic. Thiazide A in a dose contraction of the muscle. Novamine alone caused
of 5 mg produces the same decrease in blood relaxation of the preparation. In the presence of a
pressure as 500 mg of thiazide B. Which of the low concentration of novamine, the EC50 of
following statements best describes these results? acetylcholine was unchanged, but the Emax was
(A) Thiazide A is more efficacious than thiazide B reduced. In the presence of a high concentration of
(B) Thiazide A is about 100 times more potent than novamine, extremely high concentrations of
thiazide B acetylcholine had no effect. Which of the following
(C) Toxicity of thiazide A is less than that of thiazide expressions best describes novamine?
B (A) A chemical antagonist
(D) Thiazide A has a wider therapeutic window than (B) An irreversible antagonist
thiazide B (C) A partial agonist
(E) Thiazide A has a longer half-life than thiazide B (D) A physiologic antagonist
(E) A spare receptor agonist
5. Beta adrenoceptors in the heart regulate cardiac
rate and contractile strength. Several studies have
indicated that in humans and experimental animals,
about 90% of β adrenoceptors in the heart are spare
receptors. Which of the following statements about
spare receptors is most correct?
(A) Spare receptors, in the absence of drug, are
sequestered in the cytoplasm
(B) Spare receptors may be detected by finding that
8. Which of the curves in the graph describes the
the drug-receptor interaction lasts longer than the
percentage of binding of a large dose of full agonist
intracellular effect
to its receptors as the concentration of a partial
(C) Spare receptors influence the maximal efficacy of
agonist is increased from low to very high levels?
the drug-receptor system
(A) Curve 1
(D) Spare receptors activate the effector machinery
(B) Curve 2
of the cell without the need for a drug
(C) Curve 3
(E) Spare receptors may be detected by the finding
(D) Curve 4
that the EC50 is smaller than the Kd for the agonist (E) Curve 5
9. Which of the curves in the graph describes the
6. Two cholesterol-lowering drugs, X and Y, were
percentage effect observed when a large dose of full
studied in a large group of patients, and the
agonist is present throughout the experiment and
percentages of the group showing a specific
the concentration of a partial agonist is increased
therapeutic effect (35% reduction in low-density
from low to very high levels?
lipoprotein [LDL] cholesterol) were determined. The
(A) Curve 1
results are shown in the following table. Which of (B) Curve 2
the following statements about these results is
(C) Curve 3
correct?
(D) Curve 4
(E) Curve 5
10. Which of the curves in the graph describes the
percentage of binding of the partial agonist whose
effect is shown by Curve 4 if the system has many
spare receptors?
(A) Drug X is safer than drug Y (A) Curve 1
(B) Drug Y is more effective than drug X (B) Curve 2
(C) The 2 drugs act on the same receptors (C) Curve 3
(D) Drug X is less potent than drug Y (D) Curve 4
(E) The therapeutic index of drug Y is 10 (E) Curve 5
7. Sugammadex is a new drug that reverses the ANSWERS
action of rocuronium and certain other skeletal 1.
muscle-relaxing agents (nondepolarizing No information is given regarding the maximal
neuromuscular blocking agents). It appears to antihypertensive response to either drug. Similarly,
interact directly with the rocuronium molecule and no information about half-life or toxicity is provided.
not at all with the rocuronium receptor. Which of The fact that a given response is achieved with a
the following terms best describes sugammadex? smaller dose of thiazide A indicates that A is more
(A) Chemical antagonist potent than B in the ratio of 500:5. The answer is B.
(B) Noncompetitive antagonist 2.
(C) Partial agonist Precise quantitation is possible with both types of
(D) Pharmacologic antagonist dose-response curves. Quantal dose-response curves
(E) Physiologic antagonist show the frequency of occurrence of a specified
DIRECTIONS: 8–10. Each of the curves in the graph response, which may be therapeutically effective
below may be considered a concentration-effect (ED) or toxic (TD). Thus, quantal studies are used to
curve or a concentration-binding curve. determine the therapeutic index and the variation in
sensitivity to the drug. Graded (not quantal) dose- that bind the full agonist drops to zero, that is, Curve
response curves are used to determine maximal 5. The answer is E.
efficacy (maximal response). The answer is C. 9.
3. Curve 1 describes the response of the system when a
Drug A produces 50% of its maximal effect at a lower full agonist is displaced by increasing concentrations
dose than either B or C and thus is the most potent; of partial agonist. This is because the increasing
drug C is the least potent. However, drug A, a partial percentage of receptors binding the partial agonist
agonist, is less efficacious than drugs B and C. The finally produce the maximal effect typical of the
answer is D. partial agonist. The answer is A.
4. 10.
Choices involving chemical or physiologic Partial agonists, like full agonists, bind 100% of their
antagonism are incorrect because novamine is said receptors when present in a high enough
to act at the same receptors as acetylcholine. When concentration. Therefore, the binding curve (but not
given alone, the novamine effect is opposite to that
the effect curve) will go to 100%. If the effect curve
of acetylcholine, so choice C is incorrect. “Spare
receptor agonist” is a nonsense distracter. The is Curve 4 and many spare receptors are present, the
answer is B. binding curve must be displaced to the right of Curve
5. 4 (Kd > EC50). Therefore, Curve 3 fits the description
There is no difference in location between “spare” better than Curve 2. The answer is C.
and other receptors. Spare receptors may be defined
as those that are not needed for binding drug to QUESTIONS
achieve the maximal effect. Spare receptors 1. Mr Jones has zero kidney function and is
influence the sensitivity of the system to an agonist undergoing hemodialysis while awaiting a kidney
because the statistical probability of a drug-receptor transplant. He takes metformin for type 2 diabetes
interaction increases with the total number of mellitus and was previously stabilized (while his
receptors. They do not alter the maximal efficacy. If kidney function was adequate) at a dosage of 500
they do not bind an agonist molecule, spare mg twice daily, given orally. The plasma
receptors do not activate an effector molecule. EC50 concentration at this dosage with normal kidney
less than Kd is an indication of the presence of spare function was found to be 1.4 mg/L. He has been on
receptors. The answer is E. dialysis for 10 days and metformin toxicity is
6. suspected. A blood sample now shows a metformin
No information is presented regarding the safety of concentration of 4.2 mg/L. What was Mr. Jones’
these drugs. Similarly, no information on efficacy clearance of metformin while his kidney function
(maximal effect) is presented; this requires graded was normal?
(A) 238 L/d
dose-response curves. Although both drugs are said
(B) 29.8 L/h
to be producing a therapeutic effect, no information
(C) 3 L/d
on their receptor mechanisms is given. Since no data (D) 238 L/h
on toxicity are available, the therapeutic index (E) 30 L/min
cannot be determined. The answer is D because the 2. Ms Smith, a 65-year-old woman with pneumonia,
ED50 of drug Y (20 mg/d) is less than that of drug X was given tobramycin, 150 mg, intravenously. After
(50 mg/d). 20 minutes, the plasma concentration was measured
and was found to be 3 mg/L. Assuming no
7. elimination of the drug in 20 minutes, what is the
Sugammadex interacts directly with rocuronium and apparent volume of distribution of tobramycin in Ms
not with the rocuronium receptor; therefore, it is a Smith?
chemical antagonist. The answer is A. (A) 3 L/min
8. (B) 3 L
The binding of a full agonist decreases as the (C) 50 L
concentration of a partial agonist is increased to very (D) 7 L
high levels. As the partial agonist displaces more and (E) 0.1 mg/min
more of the full agonist, the percentage of receptors 3. St John’s Wort, a popular botanical remedy, is a
potent inducer of hepatic phase I CYP3A4 enzymes.
Verapamil and phenytoin are both eliminated from 100 times the amount remaining in the blood
the body by metabolism in the liver. Verapamil has a plasma. What is the probable volume of distribution
clearance of 1.5 L/min, approximately equal to liver in a hypothetical person with 8 L of blood and 4 L of
blood flow, whereas phenytoin has a clearance of plasma?
0.1 L/min. Based on this fact, which of the following (A) Insufficient data to calculate
is most correct? (B) 8 L
(A) St John’s Wort will increase the half-life of (C) 14.14 L
phenytoin and verapamil (D) 100 L
(B) St John’s Wort will decrease the volume of (E) 404 L
distribution of CYP3A4 substrates 8. A 63-year-old woman in the intensive care unit
(C) St John’s Wort will decrease the hepatic requires an infusion of procainamide. Its half-life is 2
extraction of phenytoin h. The infusion is
(D) St John’s Wort will decrease the first-pass effect begun at 9 am. At 1 pm on the same day, a blood
for verapamil sample
(E) St John’s Wort will increase the clearance of is taken; the drug concentration is found to be 3
phenytoin mg/L. What is the probable steady-state drug
concentration after 16 or more hours of infusion?
5. You are the only physician in a clinic that is cut off (A) 3 mg/L
from the outside world by violent storms, flooding, (B) 4 mg/L
and landslides. A 15-year-old girl is brought to the (C) 6 mg/L
clinic with severe asthmatic wheezing. Because of (D) 9.9 mg/L
the lack of other drugs, you decide to use (E) 15 mg/L
intravenous theophylline for treatment. The 9. A 30-year-old man is brought to the emergency
pharmacokinetics of theophylline include the department in a deep coma. Respiration is severely
following average parameters: Vd 35 L; CL 48 depressed and he has pinpoint pupils. His friends
mL/min; half-life 8 h. If an intravenous infusion of state that he self-administered a large dose of
theophylline is started at a rate of 0.48 mg/min, how morphine 6 h earlier. An immediate blood analysis
long would it take to reach 93.75% of the final shows a morphine blood level of 0.25 mg/L.
steady-state concentration? Assuming that the Vd of morphine in this patient is
(A) Approximately 48 min 200 L and the half-life is 3 h, how much morphine
(B) Approximately 7.4 h did the patient inject 6 h earlier?
(C) Approximately 8 h (A) 25 mg
(D) Approximately 24 h (B) 50 mg
(E) Approximately 32 h (C) 100 mg
6. A 74-year-old retired mechanic is admitted with a (D) 200 mg
myocardial infarction and a severe acute cardiac (E) Not enough data to predict
arrhythmia. You decide to give lidocaine to correct 10. Gentamicin, an aminoglycoside antibiotic, is
the arrhythmia. A continuous intravenous infusion of sometimes given in intermittent intravenous bolus
lidocaine, 1.92 mg/min, is started at 8 am. The doses of 100 mg 3 times a day to achieve target peak
average pharmacokinetic parameters of lidocaine plasma concentrations of about 5 mg/L.
are: Vd 77 L; clearance 640 mL/min; half-life 1.4 h. Gentamicin’s clearance (normally 5.4 L/h/70 kg) is
What is the expected steady-state plasma almost entirely by glomerular filtration. Your patient,
concentration? however, is found to have a creatinine clearance one
(A) 40 mg/L third of normal. What should your modified dosage
(B) 3.0 mg/L regimen for this patient be?
(C) 0.025 mg/L (A) 20 mg 3 times a day
(D) 7.2 mg/L (B) 33 mg 3 times a day
(E) 3.46 mg/L (C) 72 mg 3 times a day
7. A new drug is under study in phase 1 trials. It is (D) 100 mg 2 times a day
found that this molecule is avidly taken up by (E) 150 mg 2 times a day
extravascular tissues so that the final total amount in
the extravascular compartment at steady state is ANSWERS
1.
Examination questions often provide more 5.
information than is needed—to test the student’s The approach of the drug plasma concentration to
ability to classify and organize data. In question 1, steady-state concentration during continuous
the data provided for Mr Jones on dialysis is infusion follows a stereotypical curve (Figure 3–3)
irrelevant, even though choice A, 238 L/d, is the that rises rapidly at first and gradually reaches a
correct clearance while on dialysis. By definition, plateau. It reaches 50% of steady state at 1 half-life,
clearance is calculated by dividing the rate of 75% at 2 half-lives, 87.5% at 3, 93.75% at 4, and
elimination by the plasma concentration: progressively halves the difference between its
Rate in = rate out (elimination rate) at steady state current level and 100% of steady state with each
(ss) half-life. The answer is E, 32 h, or 4 half-lives.
=CLrateinCp(ss) 6.
=CL1000mg/24h1.4mg/L The drug is being administered continuously and the
CL = 29.8 L/h steady-state concentration (Cpss) for a continuously
The answer is B. administered drug is given by the equation in
2. question 1. The answer is B.
The volume of distribution (Vd) is the apparent 7.
volume into which the loading dose is distributed. It Let Z be the amount in the blood plasma. If the
is calculated by dividing the dose by the resulting amount in the rest of the body is 100 times greater,
plasma concentration, Cp: then the total amount in the body is 101Z. The
=VloadingdoseCdp concentration in the blood plasma (Cp) is Z/4 L.
=V150mg3mg/Ld The answer is E.
Vd = 50 L 8.
The answer is C. According to the curve that relates plasma
3. concentration to infusion time (Figure 3–3), a drug
Induction of phase I metabolizing enzymes will reaches 50% of its final steady-state concentration in
decrease the half-life of substrates of these 1 half-life, 75% in 2 half-lives, etc. From 9 am to 1 pm
enzymes. P450 enzyme induction has no effect on is 4 h, or 2 half-lives. Therefore, the measured
volume of distribution. Hepatic extraction, the first- concentration at 1 pm is 75% of the steady-state
pass effect, and clearance for CYP3A4 substrates will value (0.75 × Cpss). The steady-state concentration is
be increased by inducers. However, the extraction of 3 mg/L divided by 0.75, or 4 mg/L. The answer is B.
verapamil is already equal to the hepatic blood flow, 9.
so further increase in metabolism will not increase According to the curve that relates the decline of
clearance of this drug. The answer is E. plasma concentration to time as the drug is
4. eliminated (Figure 3–3), the plasma concentration of
By inspection of the data in the table, it is clear that morphine was 4 times higher immediately after
the steady-state plasma concentration is administration than at the time of the measurement,
approximately 4 mg/L. None of the measured which occurred 6 h, or 2 half-lives, later. Therefore,
concentrations is equal to one half of the steady the initial plasma concentration was 1 mg/L. Since
state value, so the half-life is not immediately the amount in the body at any time is equal to Vd ×
apparent. However, according to the constant plasma concentration (text Equation 1), the amount
infusion principle (Figure 3–3), 2 half-lives are injected was 200 L × 1 mg/L, or 200 mg. The answer
required to reach 75% of the final concentration; is D.
75% (3.0 mg/L) of the final steady-state 10.
concentration was reached at 8 h. If 8 h equals 2 If the drug is cleared almost entirely by the kidney
half-lives, the half-life must be 4 h. Rearranging the and creatinine clearance is reduced to one third of
equation for maintenance dosing (dosing rate = CL × normal, the total daily dose should also be reduced
Cp), it can be determined that the clearance (CL) = to one third. The answer is B.
dosing rate/plasma concentration (Cp), or 2 L/h. The
volume of distribution (Vd) can be calculated from QUESTIONS
the half-life equation (t1/2 = 0.693 × Vd/CL) and is Questions 1–2. You are planning to treat chronic
equal to 11.5 L. This drug follows first-order kinetics, major depression in a 35-year-old patient with
as indicated by the progressive approach to the recurrent suicidal thoughts. She has several
steady-state plasma concentration. The answer is A. comorbid conditions that require drug therapy. You
are concerned about drug interactions caused by (A) Amiodarone
changes in drug metabolism in this patient. (B) Ethanol
1. Drug metabolism in humans usually results in a (C) Phenobarbital
product that is (D) Procainamide
(A) Less lipid soluble than the original drug (E) Rifampin
(B) More likely to distribute intracellularly 7. Which of the following drugs, if used chronically, is
(C) More likely to be reabsorbed by kidney tubules most likely to increase the toxicity of
(D) More lipid soluble than the original drug acetaminophen?
(E) Less water soluble than the original drug (A) Cimetidine
2. If therapy with multiple drugs causes induction of (B) Ethanol
drug metabolism in your depressed patient, it will (C) Ketoconazole
(A) Be associated with increased smooth (D) Procainamide
endoplasmic reticulum (E) Quinidine
(B) Be associated with increased rough endoplasmic (F) Ritonavir
reticulum (G) Succinylcholine
(C) Be associated with decreased enzymes in the (H) Verapamil
soluble cytoplasmic fraction 8. Which of the following drugs has higher first-pass
(D) Require 3–4 months to reach completion metabolism in men than in women?
(E) Be irreversible (A) Cimetidine
3. Which of the following factors is likely to increase (B) Ethanol
the duration of action of a drug that is metabolized (C) Ketoconazole
by CYP3A4 in the liver? (D) Procainamide
(A) Chronic administration of rifampin during (E) Quinidine
therapy with the drug in question (F) Ritonavir
(B) Chronic therapy with amiodarone (G) Succinylcholine
(C) Displacement from tissue-binding sites by (H) Verapamil
another drug 9. Which of the following drugs is an established
(D) Increased cardiac output inhibitor of P-glycoprotein (P-gp) drug transporters?
(E) Chronic administration of carbamazepine (A) Cimetidine
4. Reports of cardiac arrhythmias caused by (B) Ethanol
unusually high blood levels of 2 antihistamines, (C) Ketoconazole
terfenadine and astemizole, led to their removal (D) Procainamide
from the market. Which of the following best (E) Quinidine
explains these effects? (F) Ritonavir
(A) Concomitant treatment with rifampin (G) Succinylcholine
(B) Use of these drugs by chronic alcoholics (H) Verapamil
(C) Use of these drugs by chronic smokers 10. Which of the following cytochrome isoforms is
(D) Treatment of these patients with ketoconazole, responsible for metabolizing the largest number of
an azole antifungal agent drugs?
5. Which of the following agents, when used in (A) CYP1A2
combination with other anti-HIV drugs, permits dose (B) CYP2C9
reductions? (C) CYP2C19
(A) Cimetidine (D) CYP2D6
(B) Efavirenz (E) CYP3A4
(C) Ketoconazole ANSWERS
(D) Procainamide 1.
(E) Quinidine Biotransformation usually results in a product that is
(F) Ritonavir less lipid-soluble. This facilitates elimination of drugs
(G) Succinylcholine that would otherwise be reabsorbed from the renal
(H) Verapamil tubule. The answer is A.
6. Which of the following drugs may inhibit the 2.
hepatic microsomal P450 responsible for warfarin The smooth endoplasmic reticulum, which contains
metabolism? the mixed-function oxidase drug-metabolizing
enzymes, is selectively increased by inducers. The 1. A 59-year-old man with acute coronary syndrome
answer is A. is admitted to the hospital for emergency
3. percutaneous insertion of a coronary stent. Which of
Rifampin and carbamazepine can induce drug- the following drugs might cause unexpected results
metabolizing enzymes and thereby may reduce the based on the patient’s CYP2C19 genotype?
duration of drug action. Displacement of drug from (A) Clopidogrel
tissue may transiently increase the intensity of the (B) Codeine
effect but decreases the volume of distribution. (C) Prasugrel
Amiodarone is recognized as an inhibitor of P450 (D) Ticagrelor
and may decrease clearance of drugs metabolized by (E) Warfarin
CYP2C9, CYP2D6, and CYP3A4. The answer is B. 2. A 62-year-old woman with advanced colon cancer
4. is treated with intravenous 5-fluorouracil. Within a
Treatment with rifampin and chronic alcohol use are few days, she develops severe diarrhea, and within a
associated with increased drug metabolism and week, she shows severe neutropenia. Which of the
lower, not higher, blood levels. Ketoconazole, following polymorphisms is most likely to be
itraconazole, erythromycin, and some substances in responsible?
grapefruit juice slow the metabolism of certain older (A) CYP2D6 *1x3
non-sedating antihistamines (Chapter 16). The (B) CYP2C19*2
answer is D. (C) CYP2C9*3
5. (D) DYPD *2A
Ritonavir inhibits hepatic drug metabolism, and its (E) UGT1A1*28
use at low doses in combination regimens has 3. A 38-year-old man is being treated for HIV-
permitted dose reductions of other HIV protease induced acquired immunodeficiency syndrome
inhibitors (eg, indinavir). The answer is F. (AIDS). When abacavir therapy is begun, he develops
6. a severe skin rash. Which of the following
Amiodarone is an important antiarrhythmic drug and pharmacogenomic diagnoses might explain this skin
has a well-documented ability to inhibit the hepatic rash?
metabolism of many drugs. The answer is A. (A) CYP2D6 *3 (PM)
7. (B) CYP3A5 *3 (PM)
Acetaminophen is normally eliminated by phase II (C) HLA-B *57:01 (EM)
conjugation reactions. The drug’s toxicity is caused (D) SLCO1B1*5 (PM)
by an oxidized reactive metabolite produced by 4. A college student volunteers to have his genome
phase I oxidizing P450 enzymes. Ethanol and certain decoded as part of a population-wide study of
other drugs induce P450 enzymes and thus reduce polymorphisms. He receives a call from the principal
the hepatotoxic dose. Alcoholic cirrhosis reduces the investigator informing him that his genome
hepatotoxic dose even more. The answer is B. unexpectedly contains an important single
8. nucleotide polymorphism. Which of the following
Ethanol is subject to metabolism in the stomach as polymorphisms is associated with risk of hemolysis
well as in the liver. Independent of body weight and and increased resistance to malaria?
other factors, men have greater gastric ethanol (A) CYP2D6 *3
metabolism and thus a lower ethanol bioavailability (B) CYP2D19*2
than women. The answer is B. (C) TPMT *2
9. (D) UGT1A1*28
Verapamil is an inhibitor of P-glycoprotein drug (E) G6PD-(A)–Canton
transporters and has been used to enhance the 5. A 7-year-old child is brought to the emergency
cytotoxic actions of methotrexate in cancer department in coma with cyanosis. Her mother
chemotherapy. The answer is H. states that the girl was given codeine with
10. acetaminophen because of severe bruising after a
While CYP2D6 is responsible for metabolizing fall. Shortly after the first dose, the child became
approximately 25% of drugs, CYP3A4 is involved in unresponsive and “turned blue.” Which of the
almost 50% of such reactions. The answer is E. following alleles might be responsible for this
presentation?
QUESTIONS (A) CYP2D6 *1x3
(B) CYP2C19*2 morphine. CYP2D6 *1xN and *2xN are gain-of-
(C) CYP2C9*3 function polymorphisms that result in more efficient
(D) DYPD*2A conversion to morphine and increased risk of opioid-
(E) UGT1A1*28 induced respiratory depression. The answer is A.
ANSWERS QUESTIONS
1. 1. A 3-year-old child has been admitted to the
Clopidogrel is a prodrug that must be metabolized to emergency department having swallowed the
an active platelet-inhibiting metabolite by CYP2C19. contents of 2 bottles of a nasal decongestant. The
Poor metabolizers achieve inadequate platelet active ingredient of the medication is a potent,
inhibition, and EMs and UMs may have excess effect selective α-adrenoceptor agonist drug. Which of the
and bleed. Prasugrel and ticagrelor do not require following is a sign of α-receptor activation that may
P450 activation and are not subject to this risk. The occur in this patient?
answer is A. (A) Bronchodilation
2. (B) Cardiac acceleration (tachycardia)
CYP2D6 *1x3 is a gain-of-function allele and is (C) Pupillary dilation (mydriasis)
associated with increased effect and toxicity of (D) Renin release from the kidneys
codeine. CYP2C19*2 is a nonfunctional allele (E) Vasodilation of the blood vessels of the skin
associated with reduced efficacy of clopidogrel. 2. Mr Green is a 60-year-old man with poorly
CYP2C9*3 with a reduced function allele of VCORC1 controlled hypertension of 170/110 mm Hg. He is to
is associated with reduced warfarin clearance. receive minoxidil. Minoxidil is a powerful arteriolar
UGT1A1*28 is a reduced function allele for uridine vasodilator that does not act on autonomic
5′-diphospho-(UDP) glucuronosyltransferase and receptors. Which of the following effects will be
enhances irinotecan toxicity. 5-Fluorouracil is observed if no other drugs are used?
cleared by dihydropyrimidine dehydrogenase (DPD). (A) Tachycardia and increased cardiac contractility
The DYPD*2A allele is nonfunctional. The answer is (B) Tachycardia and decreased cardiac output
D. (C) Decreased mean arterial pressure and decreased
3. cardiac contractility
Poor metabolizers of the CYP2D6 *3 genotype are (D) Decreased mean arterial pressure and increased
prone to reduced efficacy of codeine. Poor salt and water excretion by the kidney
metabolizers of the CYP3A5*3 type show reduced (E) No change in mean arterial pressure and
tacrolimus clearance. Simvastatin toxicity decreased cardiac contractility
(myopathy) is enhanced in SLCO1B1 poor 3. Full activation of the parasympathetic nervous
metabolizers. Enhanced metabolizers of the HLA- system is likely to produce which of the following
B*57:01 type are prone to abacavir rashes and effects?
flucloxacillin liver damage. The answer is C. (A) Bronchodilation
4. (B) Decreased intestinal motility
CYP2D6 *3 is associated with reduced codeine (C) Increased thermoregulatory sweating
efficacy. CYP2D19*2 results in reduced clopidogrel (D) Increased pupillary constrictor tone (miosis)
conversion to its active metabolite. TPMT *2 is (E) Increased heart rate (tachycardia)
associated with decreased clearance of 6- 6. Nicotinic receptor sites do not include which one
mercaptopurine and increased toxicity. UGT1A1*28 of the following sites?
results in decreased clearance and increased toxicity (A) Bronchial smooth muscle
of SN-38, the active metabolite of irinotecan. G6PD- (B) Adrenal medullary cells
(A)–Canton is a reduced function allele of the (C) Parasympathetic ganglia
glucose 6-phosphate dehydrogenase gene that (D) Skeletal muscle end plates
decreases intracellular stores of glutathione, (E) Sympathetic ganglia
increasing the risk of hemolysis but reducing 7. Several children at a summer camp were
susceptibility to malaria. The answer is E. hospitalized with symptoms thought to be due to
5. ingestion of food containing botulinum toxin. Which
As noted in answer 4, SNPs in CYP2D6 may increase one of the following signs or symptoms is consistent
or decrease the efficacy and toxicity of codeine with the diagnosis of botulinum poisoning?
because the CYP2D6 enzyme is responsible for (A) Bronchospasm
conversion of codeine to its active metabolite, (B) Cycloplegia
(C) Diarrhea When blood pressure increases, the
(D) Skeletal muscle spasms parasympathetic system is activated and heart rate
(E) Hyperventilation decreases. Acetylcholine is the transmitter at
8. Which one of the following is the primary parasympathetic nerve endings innervating the sinus
neurotransmitter agent normally released in the node (nerve endings of the vagus nerve). The answer
sinoatrial node of the heart in response to a blood is A.
pressure increase? QUESTIONS
(A) Acetylcholine 1. A 30-year-old woman undergoes abdominal
(B) Dopamine surgery. In spite of minimal tissue damage, complete
(C) Epinephrine ileus (absence of bowel motility) follows, and she
(D) Glutamate complains of severe bloating. She also finds it
(E) Norepinephrine difficult to urinate. Mild cholinomimetic stimulation
ANSWERS with bethanechol or neostigmine is often effective in
1. relieving these complications of surgery.
Mydriasis can be caused by contraction of the radial Neostigmine and bethanechol in moderate doses
fibers of the iris; these smooth muscle cells have α have significantly different effects on which one of
receptors. All the other listed responses are the following?
mediated by β adrenoceptors (Table 6–4). The (A) Gastric secretory cells
answer is C. (B) Vascular endothelium
2. (C) Salivary glands
Because of the compensatory responses, a drug that (D) Sweat glands
directly decreases blood pressure through a (E) Ureteral tone
decrease in peripheral vascular resistance will cause 2. Parathion has which one of the following
a reflex increase in sympathetic outflow, an increase characteristics?
in renin release, and a decrease in parasympathetic (A) It is inactivated by conversion to paraoxon
outflow. As a result, heart rate and cardiac force will (B) It is less toxic to humans than malathion
increase. In addition, salt and water retention will (C) It is more persistent in the environment than
occur. The answer is A. DDT
3. (D) It is poorly absorbed through skin and lungs
Parasympathetic discharge causes bronchial and (E) If treated early, its toxicity may be partly reversed
intestinal smooth muscle contraction and by pralidoxime
bradycardia. Thermoregulatory (eccrine) sweat 3. Ms Brown has been treated for myasthenia gravis
glands are innervated by sympathetic cholinergic for several years. She reports to the emergency
fibers, not parasympathetic. The answer is D. department complaining of recent onset of
6. weakness of her hands, diplopia, and difficulty
Both types of ganglia and the skeletal muscle swallowing. She may be suffering from a change in
neuromuscular junction have nicotinic response to her myasthenia therapy, that is, a
cholinoceptors, as does the adrenal medulla (a cholinergic or a myasthenic crisis. Which of the
modified form of sympathetic ganglionic neuron following is the best drug for distinguishing between
tissue). Bronchial smooth muscle contains myasthenic crisis (insufficient therapy) and
muscarinic cholinoceptors and noncholinergic cholinergic crisis (excessive therapy)?
receptors. The answer is A. (A) Atropine
7. (B) Edrophonium
Botulinum toxin impairs all types of cholinergic (C) Physostigmine
transmission, including transmission at ganglionic (D) Pralidoxime
synapses and somatic motor nerve endings. (E) Pyridostigmine
Botulinum toxin prevents discharge of vesicular
transmitter content from cholinergic nerve endings. 4. A crop duster pilot has been accidentally exposed
All of the signs listed except cycloplegia indicate to a high concentration of a highly toxic agricultural
increased muscle contraction; cycloplegia (paralysis organophosphate insecticide. If untreated, the cause
of accommodation) results in blurred near vision. of death from such exposure would probably be
The answer is B. (A) Cardiac arrhythmia
8. (B) Gastrointestinal bleeding
(C) Heart failure 10. Which of the following is the primary second-
(D) Hypotension messenger process in the contraction of the ciliary
(E) Respiratory failure muscle when focusing on near objects?
5. Mr Green has just been diagnosed with (A) cAMP (cyclic adenosine monophosphate)
dysautonomia (chronic idiopathic autonomic (B) DAG (diacylglycerol)
insufficiency). You are considering different (C) Depolarizing influx of sodium ions via a channel
therapies for his disease. Pyridostigmine and (D) IP3 (inositol 1,4,5-trisphosphate)
neostigmine may cause which one of the following in (E) NO (nitric oxide)
this patient? ANSWERS
(A) Bronchodilation 1.
(B) Cycloplegia Because neostigmine acts on the enzyme
(C) Diarrhea cholinesterase, which is present at all cholinergic
(D) Irreversible inhibition of acetylcholinesterase synapses, this drug increases acetylcholine effects at
(E) Reduced gastric acid secretion nicotinic junctions as well as muscarinic ones.
6. Parasympathetic nerve stimulation and a slow Bethanechol, on the other hand, is a direct-acting
infusion of bethanechol will each agent that is selective for muscarinic receptors
(A) Cause ganglion cell depolarization regardless of whether the receptors are innervated
(B) Cause skeletal muscle end plate depolarization or not. The muscarinic receptors on vascular
(C) Cause vasodilation endothelial cells are not innervated and respond
(D) Increase bladder tone only to direct-acting drugs. The answer is B.
(E) Increase heart rate 2.
7. Actions and clinical uses of muscarinic The “-thion” organophosphates (those containing
cholinoceptor agonists include which one of the the P:S bond) are activated, not inactivated, by
following? conversion to “-oxon” (P:O) derivatives. They are less
(A) Bronchodilation (treatment of asthma) stable than halogenated hydrocarbon insecticides of
(B) Miosis (treatment of glaucoma) the DDT type; therefore, they are less persistent in
(C) Decreased gastrointestinal motility (treatment of the environment. Parathion is more toxic than
diarrhea) malathion. It is very lipid-soluble and rapidly
(D) Decreased neuromuscular transmission and absorbed through the lungs and skin. Pralidoxime
relaxation of skeletal muscle (during surgical has very high affinity for the phosphorus atom and is
anesthesia) a chemical antagonist of organophosphates. The
(E) Increased sweating (treatment of fever) answer is E.
8. Which of the following is a direct-acting 3.
cholinomimetic that is lipid-soluble and is used to Any of the cholinesterase inhibitors (choices B, C, or
facilitate smoking cessation? E) would effectively correct myasthenic crisis.
(A) Acetylcholine However, because cholinergic crisis (if that is what is
(B) Bethanechol causing the symptoms) would be worsened by a
(C) Neostigmine cholinomimetic, we choose the shortest-acting
(D) Physostigmine cholinesterase inhibitor, edrophonium. The answer
(E) Varenicline is B.
9. A 3-year-old child is admitted to the emergency 4.
department after taking a drug from her parents’ Respiratory failure, from neuromuscular paralysis or
medicine cabinet. The signs suggest that the drug is CNS depression, is the most important cause of
an indirect-acting cholinomimetic with little or no acute deaths in cholinesterase inhibitor toxicity. The
CNS effect and a duration of action of about 2–4 h. answer is E.
Which of the following is the most likely cause of 5.
these effects? Cholinesterase inhibition is typically associated with
(A) Acetylcholine increased (never decreased) bowel activity.
(B) Bethanechol (Fortunately, many patients become tolerant to this
(C) Neostigmine effect.) The answer is C.
(D) Physostigmine 6.
(E) Pilocarpine Choice (E) is not correct because the vagus slows the
heart. Parasympathetic nerve stimulation does not
cause vasodilation (most vessels do not receive (A) Dehydration
parasympathetic innervation), so choice (C) is (B) Hallucinations
incorrect. Ganglion cells and the end plate contain (C) Hypertension
nicotinic receptors, which are not affected by (D) Hyperthermia
bethanechol, a direct-acting muscarinic agonist. The (E) Intraventricular heart block
answer is D. 3. Which one of the following can be blocked by
7. atropine?
Muscarinic agonists cause accommodation and (A) Decreased blood pressure caused by
cyclospasm, the opposite of paralysis of hexamethonium
accommodation (cycloplegia). In acute angle-closure (B) Increased blood pressure caused by nicotine
glaucoma and chronic open-angle glaucoma, this (C) Increased skeletal muscle strength caused by
may result in a desirable increased outflow of neostigmine
aqueous and decreased intraocular pressure. These (D) Tachycardia caused by exercise
agents may cause bronchospasm but have no effect (E) Sweating caused by exercise
on neuromuscular transmission. They may cause Questions 4–5. Two new synthetic drugs (X and Y)
diarrhea and are not used in its treatment. are to be studied for their cardiovascular effects. The
Muscarinic agonists may also cause sweating, but drugs are given to three anesthetized animals while
drug-induced sweating is of no value in the the blood pressure is recorded. The first animal has
treatment of fever. The answer is B. received no pretreatment (control), the second has
received an effective dose of a long-acting ganglion
8. blocker, and the third has received an effective dose
Varenicline is a lipid-soluble partial agonist at of a long-acting muscarinic antagonist.
nicotinic receptors and is used to reduce craving for 4. Drug X caused a 50 mm Hg rise in mean blood
tobacco in smokers. The answer is E. pressure in the control animal, no blood pressure
9. change in the ganglion-blocked animal, and a 75 mm
Neostigmine is the prototypical indirect-acting mean blood pressure rise in
cholinomimetic; it is a quaternary (charged) the atropine-pretreated animal. Drug X is probably a
substance with poor lipid solubility; its duration of drug similar to
action is about 2–4 h. Physostigmine is similar but (A) Acetylcholine
has good lipid solubility and significant CNS effects. (B) Atropine
The answer is C. (C) Epinephrine
10. (D) Hexamethonium
Cholinomimetics cause smooth muscle contraction (E) Nicotine
mainly through the release of intracellular calcium. 6. A 30-year-old man has been treated with several
This release is triggered by an increase in IP3 acting autonomic drugs for 4 weeks. He is now admitted to
on receptors in the endoplasmic reticulum. The the emergency department showing signs of drug
answer is D. toxicity. Which of the following signs would
QUESTIONS distinguish between an overdose of a ganglion
1. A 27-year old compulsive drug user injected a blocker versus a muscarinic blocker?
drug he thought was methamphetamine, but he has (A) Cycloplegia
not developed any signs of methamphetamine (B) Dry skin in a warm environment
action. He has been admitted to the emergency (C) Miosis
department and antimuscarinic drug overdose is (D) Postural hypotension
suspected. Probable signs of atropine overdose (E) Tachycardia
include which one of the following? 7. Which of the following is an accepted therapeutic
(A) Gastrointestinal smooth muscle cramping indication for the use of antimuscarinic drugs?
(B) Increased heart rate (A) Atrial fibrillation
(C) Increased gastric secretion (B) Botulinum poisoning
(D) Pupillary constriction (C) Chronic obstructive pulmonary disease (COPD)
(E) Urinary frequency (D) Glaucoma
2. Which of the following is the most dangerous (E) Postoperative urinary retention
effect of belladonna alkaloids in infants and 8. Which of the following is an expected effect of a
toddlers? therapeutic dose of an antimuscarinic drug?
(A) Decreased cAMP (cyclic adenosine acetylcholine released from sympathetic nerve fibers
monophosphate) in cardiac muscle at eccrine sweat glands. The answer is E.
(B) Decreased DAG (diacylglycerol) in salivary gland 4.
tissue Drug X causes an increase in blood pressure that is
(C) Increased IP3 (inositol trisphosphate) in intestinal blocked by a ganglion blocker but not by a
smooth muscle muscarinic blocker. The pressor response is actually
(D) Increased potassium efflux from smooth muscle increased by pretreatment with atropine, a
(E) Increased sodium influx into the skeletal muscle muscarinic blocker, suggesting that compensatory
end plate vagal discharge might have blunted the full
9. Which one of the following drugs causes response. This description fits a ganglion stimulant
vasodilation that can be blocked by atropine? like nicotine but not epinephrine, since
(A) Benztropine epinephrine’s pressor effects are produced at α
(B) Bethanechol receptors, not in the ganglia. The answer is E.
(C) Botulinum toxin 6.
(D) Cyclopentolate Neither ganglion blockers nor muscarinic blockers
(E) Edrophonium cause miosis; they cause mydriasis. Both classes of
(F) Neostigmine cholinoceptor blockers increase resting heart rate
(G) Pralidoxime and cause cycloplegia, because these are
10. Which one of the following drugs has a very high determined largely by parasympathetic tone.
affinity for the phosphorus atom in parathion and is Similarly, both can cause dry skin, since this requires
often used to treat life-threatening insecticide cholinergic transmission. Postural hypotension, on
toxicity? the other hand, is a sign of sympathetic blockade,
(A) Atropine which would occur with ganglion blockers but not
(B) Benztropine muscarinic blockers (Chapter 6). The answer is D.
(C) Bethanechol 7.
(D) Botulinum Atrial fibrillation and other arrhythmias are not
(E) Cyclopentolate responsive to antimuscarinic agents. Botulinum
(F) Neostigmine poisoning is associated with parasympathetic
(G) Pralidoxime blockade. Parkinson’s disease, not Huntington’s, is
ANSWERS partially responsive to antimuscarinic drugs.
1. Antimuscarinic drugs tend to cause urinary retention
Tachycardia is a characteristic atropine overdose and may precipitate or exacerbate glaucoma.
effect. Bradycardia is sometimes observed after Bronchospasm is mediated in part by vagal outflow
small doses. None of the other choices are typical of in many patients with COPD and in some with
atropine or methamphetamine overdose. The asthma. The answer is C.
answer is B. 8.
2. Muscarinic M1 and M3 receptors mediate increases
Choices B, D, and E are all possible effects of the in IP3 and DAG in target tissues (intestine, salivary
atropine group. In infants, however, the most glands). M2 receptors (heart) mediate a decrease in
dangerous effect is hyperthermia. Deaths with body cAMP and an increase in potassium permeability.
temperatures in excess of 42°C have occurred after Antimuscarinic agents block these effects. The
the use of atropine-containing eye drops in children. answer is B.
The answer is D. 9.
3. Bethanechol (Chapter 7) causes vasodilation by
Atropine blocks muscarinic receptors and inhibits directly activating muscarinic receptors on the
parasympathomimetic effects. Nicotine can induce endothelium of blood vessels. This effect can be
both parasympathomimetic and sympathomimetic blocked by atropine. Indirectly acting agents (AChE
effects by virtue of its ganglion-stimulating action. inhibitors) do not typically cause vasodilation
Hypertension and exercise-induced tachycardia because the endothelial receptors are not
reflect sympathetic discharge with norepinephrine innervated and acetylcholine is not released at this
release and therefore would not be blocked by site. Pralidoxime is a distracter in this answer list.
atropine. Exercise-induced sweating is another The answer is B.
sympathomimetic response, but it is mediated by 10.
Pralidoxime has a very high affinity for the (E) Tyramine
phosphorus atom in organophosphate insecticides. 5. A group of volunteers are involved in a phase 1
The answer is G. clinical trial of a new autonomic drug. When
administered by intravenous bolus, the blood
QUESTIONS pressure increases. When given orally for 1 week,
Questions 1 and 2. A 7-year-old boy with a previous the blood pressure decreases. Which of the
history of bee sting allergy is brought to the following standard agents does the new drug most
emergency department after being stung by 3 bees. resemble?
1. Which of the following are probable signs of the (A) Atropine
anaphylactic reaction to bee stings? (B) Clonidine
(A) Bronchodilation, tachycardia, hypertension, (C) Phentolamine (an alpha blocker)
vomiting, diarrhea (D) Phenylephrine
(B) Bronchospasm, tachycardia, hypotension, (E) Propranolol (a beta blocker)
laryngeal edema
(C) Diarrhea, bradycardia, vomiting 6. Your 30-year-old patient has moderately severe
(D) Laryngeal edema, bradycardia, hypotension, new onset asthma, and you prescribe a highly
diarrhea selective β2 agonist inhaler to be used when
(E) Miosis, tachycardia, vomiting, diarrhea needed. In considering the possible drug effects in
2. If this child has signs of anaphylaxis, what is the this patient, you would note that β2 stimulants
treatment of choice? frequently cause
(A) Diphenhydramine (an antihistamine) (A) Direct stimulation of renin release
(B) Ephedrine (B) Hypoglycemia
(C) Epinephrine (C) Itching due to increased cGMP (cyclic guanine
(D) Isoproterenol monophosphate) in mast cells
(E) Methylprednisolone (a corticosteroid) (D) Skeletal muscle tremor
3. A 65-year-old woman with impaired renal function (E) Vasodilation in the skin
and a necrotic ulcer in the sole of her right foot is 7. Mr Green, a 54-year-old banker, had a cardiac
admitted to the ward from the emergency transplant 6 months ago. His current blood pressure
department. She has long-standing type 2 diabetes is 120/70 mm Hg and heart rate is 100 bpm. Which
mellitus and you wish to examine her retinas for of the following drugs would have the least effect on
possible vascular changes. Which of the following Mr Green's heart rate?
drugs is a good choice when pupillary dilation—but (A) Albuterol
not cycloplegia—is desired? (B) Epinephrine
(A) Isoproterenol (C) Isoproterenol
(B) Norepinephrine (D) Norepinephrine
(C) Phenylephrine (E) Phenylephrine
(D) Pilocarpine
(E) Tropicamide ANSWERS
4. A 60-year-old immigrant from Latin America was 1.
told she had hypertension and should be taking Anaphylaxis is caused by the release of several
antihypertensive medication. She decides to take an mediators. Leukotrienes and certain proteins are the
herbal medication from an online “holistic most important of these. They cause bronchospasm
pharmacy.” One week after starting the medication, and laryngeal edema and marked vasodilation with
she is found unconscious in her apartment. In the severe hypotension. Tachycardia is a common reflex
emergency department, her blood pressure is 50/0 response to the hypotension. Gastrointestinal
mm Hg and heart rate is 40 bpm. Respirations are disturbance is not as common nor as dangerous. The
20/min; pupils are slightly constricted. Bowel sounds answer is B.
are present. Which of the following would be the 2.
most effective cardiovascular stimulant? The treatment of anaphylaxis requires a powerful
(A) Amphetamine physiologic antagonist with the ability to cause rapid
(B) Clonidine bronchodilation (β2 effect), and vasoconstriction (α
(C) Isoproterenol effect). Epinephrine is the most effective agent with
(D) Norepinephrine these properties. Antihistamines and corticosteroids
are sometimes used as supplementary agents, but after surgery. Therefore, indirect-acting
the prompt parenteral use of epinephrine is sympathomimetics are ineffective in changing heart
mandatory. The answer is C. rate. All the drugs listed are direct-acting, and all but
3. phenylephrine have significant effects on β
Antimuscarinics (tropicamide) are mydriatic and receptors. Phenylephrine usually causes reflex
cycloplegic; α-sympathomimetic agonists are only bradycardia, which requires intact vagal innervation.
mydriatic in the eye. Isoproterenol has negligible The answer is E. (Note that denervation may result
effects on the eye. Norepinephrine penetrates the in upregulation of both β1 and β2 receptors so that
conjunctiva poorly and would produce intense direct-acting β agonists may have a greater than
vasoconstriction. Pilocarpine causes miosis. normal effect.)
Phenylephrine is well-absorbed from the
conjunctival sac and produces useful mydriasis for QUESTIONS
10–30 minutes. The answer is C. 1. A patient is to receive epinephrine. She has
4. previously received an adrenoceptor-blocking agent.
“Herbal” medications often contain potent synthetic Which of the following effects of epinephrine would
drugs in addition to (or instead of) the advertised be blocked by phentolamine but not by metoprolol?
constituents. This patient shows signs of sympathetic (A) Cardiac stimulation
autonomic failure: hypotension, inappropriate (B) Increase of cAMP (cyclic adenosine
bradycardia, constricted pupils. These signs are monophosphate) in fat
compatible with a large overdose of a drug that (C) Mydriasis
causes marked depletion of stored catecholamine (D) Relaxation of bronchial smooth muscle
transmitter such as reserpine, an obsolete but (E) Relaxation of the uterus
inexpensive antihypertensive agent. The indirect- 2. Clinical studies have shown that adrenoceptor
acting agents (amphetamines and tyramine) act blockers have many useful effects in patients.
through catecholamines in (or released from) the However, a number of drug toxicities have been
nerve terminal and would therefore be ineffective in documented. Adverse effects that limit the use of
this patient. Clonidine acts primarily on presynaptic adrenoceptor blockers include which one of the
nerve endings although it can activate α2 receptors following?
located elsewhere. Isoproterenol would stimulate (A) Bronchoconstriction from α-blocking agents
the heart but has no α-agonist action and might (B) Acute heart failure exacerbation from β blockers
exacerbate the hypotension. Norepinephrine has the (C) Impaired blood sugar response with α blockers
necessary combination of direct action and a (D) Increased intraocular pressure with β blockers
spectrum that includes α1, α2, and β1 effects. The (E) Sleep disturbances from α-blocking drugs
answer is D. Questions 3–6. Four new synthetic drugs
(designated W, X, Y, and Z) are to be studied for their
5. cardiovascular effects. They are given to 4
The dual blood pressure effects of the drug suggest anesthetized animals while the heart rate is
that initially it is causing a direct α-agonist recorded. The first animal has received no
vasoconstrictor effect, but when given for a week, it pretreatment (control); the second has received an
is accumulating in a blood pressure-controlling effective dose of hexamethonium; the third has
center, eg, the CNS, and reducing sympathetic received an effective dose of atropine; and the
outflow. The answer is B. fourth has received an effective dose of
6. phenoxybenzamine. The net changes induced by W,
Tremor is a common β2 effect. Blood vessels in the X, Y, and Z in the animals are described in the
skin have almost exclusively α (vasoconstrictor) following questions.
receptors. Stimulation of renin release is a β1 effect. 3. Drug W increased heart rate in the control animal,
Beta2 agonists cause hyperglycemia and have little the atropine-pretreated animal, and the
effect on cGMP. The answer is D. phenoxybenzamine-pretreated animal. However,
7. drug W had no effect on heart rate in the
Heart transplantation involves cutting of the hexamethonium-pretreated animal. Drug W is
autonomic nerves to the heart. As a result, probably a drug similar to
autonomic nerve endings degenerate, and cardiac (A) Acetylcholine
transmitter stores are absent for 2 years or longer (B) Edrophonium
(C) Isoproterenol or reflex slowing of the heart mediated by the vagus.
(D) Nicotine Phenoxybenzamine blocks only α-receptor-mediated
(E) Norepinephrine processes. If the response produced in the
nonpretreated animal is blocked or reversed by
7. When given to a patient, phentolamine blocks hexamethonium, it is probably a direct nicotinic
which one of the following? effect or a reflex response to hypotension. In that
(A) Bradycardia induced by phenylephrine case, consider all the receptors involved in mediating
(B) Bronchodilation induced by epinephrine the reflex. Drug W causes tachycardia that is
(C) Increased cardiac contractile force induced by prevented by ganglion blockade. The only drug in the
norepinephrine list of choices that causes hypotension and
(D) Miosis induced by acetylcholine tachycardia that is not blocked by atropine is
(E) Vasodilation induced by isoproterenol isoproterenol, and the tachycardia caused by
8. Your 75-year-old patient with angina and isoproterenol is not blocked by ganglionic blockade.
glaucoma is to receive a β-blocking drug. Which of Thus, drug W must be nicotine or a drug similar to it.
the following statements is most correct regarding β- The answer is D.
blocking drugs?
(A) Esmolol’s pharmacokinetics are compatible with 7.
chronic topical use Phenylephrine, an α agonist, increases blood
(B) Metoprolol blocks β2 receptors selectively pressure and causes bradycardia through the
(C) Nadolol lacks β2-blocking action baroreceptor reflex. Blockade of this drug’s α-
(D) Pindolol is a β antagonist with high membrane- mediated vasoconstrictor effect prevents the
stabilizing (local anesthetic) activity bradycardia. The answer is A.
(E) Timolol lacks the local anesthetic effects of 8.
propranolol Esmolol is a short-acting β blocker for parenteral use
9. A 56-year-old man has hypertension and an only. Nadolol is a nonselective β blocker, and
enlarged prostate, which biopsy shows to be benign metoprolol is a β1-selective blocker. Timolol is useful
prostatic hyperplasia. He complains of urinary in glaucoma because it does not anesthetize the
retention. Which of the following drugs would be the cornea. The answer is E.
most appropriate initial therapy? 9.
(A) Albuterol An α blocker is appropriate therapy in a man with
(B) Atenolol both hypertension and benign prostatic hyperplasia
(C) Metoprolol because both conditions involve contraction of
(D) Prazosin smooth muscle containing α receptors. The answer
(E) Timolol is D.
ANSWERS
1. QUESTIONS
Mydriasis caused by contraction of the pupillary 1. A 32-year-old woman with hypertension wishes to
dilator radial smooth muscle is mediated by α become pregnant. Her physician informs her that
receptors. All the other effects listed are mediated she will have to switch to another antihypertensive
by β receptors. The answer is C. drug. Which of the following drugs is absolutely
2. contraindicated in pregnancy?
Although chronic heart failure is often treated with (A) Atenolol
certain β blockers, acute heart failure can be (B) Losartan
precipitated by these drugs. Choices A, C, and E (C) Methyldopa
reverse the correct pairing of receptor subtype (α (D) Nifedipine
versus β) with effect. Choice D reverses the direction (E) Propranolol
of change of intraocular pressure. The answer is B.
3. 2. A patient is admitted to the emergency
In developing a strategy for this type of question, department with severe tachycardia after a drug
consider first the actions of the known blocking overdose. His family reports that he has been
drugs. Hexamethonium blocks reflexes as well as the depressed about his hypertension. Which one of the
direct action of nicotine. Atropine would block direct following drugs increases the heart rate in a dose-
muscarinic effects of an unknown drug (if it had any) dependent manner?
(A) Captopril (B) Hypoglycemia
(B) Hydrochlorothiazide (C) Increased PR interval
(C) Losartan (D) Tachycardia
(D) Minoxidil (E) Thyrotoxicosis
(E) Verapamil 9. A 45-year-old man is brought to the emergency
3. Which one of the following is characteristic of department with mental obtundation. He is found to
nifedipine treatment in patients with essential have a blood pressure of 220/160 and retinal
hypertension? hemorrhages. Which one of the following is used in
(A) Competitively blocks angiotensin II at its receptor severe hypertensive emergencies, is short-acting,
(B) Decreases calcium efflux from skeletal muscle acts on a G protein-coupled receptor, and must be
(C) Decreases renin concentration in the blood given by intravenous infusion?
(D) Decreases calcium influx into smooth muscle (A) Aliskiren
(E) Decreases calcium flux into the urine (B) Captopril
4. A 73-year-old man with a history of a recent (C) Fenoldopam
change in his treatment for moderately severe (D) Hydralazine
hypertension is brought to the emergency (E) Losartan
department because of a fall at home. Which of the (F) Metoprolol
following drug groups is most likely to cause postural (G) Nitroprusside
hypotension and thus an increased risk of falls? (H) Prazosin
(A) ACE inhibitors (I) Propranolol
(B) Alpha1-selective receptor blockers 10. Which of the following is very short-acting and
(C) Arteriolar dilators acts by releasing nitric oxide?
(D) Beta1-selective receptor blockers (A) Atenolol
(E) Nonselective β blockers (B) Captopril
5. A significant number of patients started on ACE (C) Diltiazem
inhibitor therapy for hypertension are intolerant and (D) Fenoldopam
must be switched to a different class of drug. What (E) Hydrochlorothiazide
is the most common manifestation of this (F) Losartan
intolerance? (G) Minoxidil
(A) Angioedema (H) Nitroprusside
(B) Glaucoma (I) Prazosin
(C) Headache ANSWERS
(D) Incessant cough 1.
(E) Ventricular arrhythmias Methyldopa is often recommended in pregnant
6. Which one of the following is a significant patients because it has a good safety record. Calcium
unwanted effect of the drug named? channel blockers (choice D) and β blockers (choices
(A) Constipation with verapamil A and E) are not contraindicated. In contrast, ACE
(B) Heart failure with hydralazine inhibitors and ARBs (choice B) have been shown to
(C) Hemolytic anemia with atenolol be teratogenic. The answer is B.
(D) Hypokalemia with aliskiren 2.
(E) Lupus-like syndrome with hydrochlorothiazide ACE inhibitors (choice A), ARBs (choice C), and
7. Comparison of prazosin with atenolol shows that diuretics (choice B) do not significantly increase
(A) Both decrease heart rate heart rate. Although dihydropyridine calcium
(B) Both increase cardiac output channel blockers do not usually reduce rate
(C) Both increase renin secretion markedly (and may increase it), verapamil (choice E)
(D) Both increase sympathetic outflow from the CNS and diltiazem do inhibit the sinoatrial node and
(E) Both produce orthostatic hypotension predictably decrease rate. Other direct vasodilators
8. A patient with hypertension and angina is referred (choice D) regularly increase heart rate, and
for treatment. Metoprolol and verapamil are among minoxidil, a very efficacious vasodilator, causes
the drugs considered. Both metoprolol and severe tachycardia that must be controlled with β
verapamil are associated with which one of the blockers. The answer is D.
following? 3.
(A) Diarrhea
Nifedipine is a prototype L-type calcium channel 8.
blocker and lowers blood pressure by reducing Neither β blockers nor calcium channel blockers
calcium influx into vascular smooth muscle. It has no cause diarrhea. Hypoglycemia is not a common
effect on angiotensin-converting enzyme. Calcium effect of any of the antihypertensive drugs. Thyroid
efflux from skeletal muscle cells does not involve the disorders are not associated with either drug group.
L-type Ca channel. The plasma renin level may However, calcium blockers, especially verapamil and
increase as a result of the compensatory response to diltiazem, and β blockers are associated with
reduced blood pressure. Calcium channel blockers depression of calcium-dependent processes in the
have negligible effects on urine calcium. The answer heart, for example, contractility, heart rate, and
is D. atrioventricular conduction. Therefore, bradycardia
4. and increased PR interval may be expected. The
Drug-induced postural (orthostatic) hypotension is dihydropyridines do not often cause cardiac
usually due to venous pooling or excessive diuresis depression, probably because they evoke increased
and inadequate blood volume. Venous pooling is sympathetic outflow as a result of their dominant
normally prevented by α-receptor activation in vascular effects. The answer is C.
vascular smooth muscle; thus, orthostatic 9.
hypotension is caused or exacerbated by α1 Fenoldopam, nitroprusside, and propranolol are the
blockers, eg, prazosin. The answer is B. drugs in the list that have been used in hypertensive
emergencies. Fenoldopam and nitroprusside are
5. used by infusion only, but nitroprusside releases
Chronic, intolerable cough is an important adverse nitric oxide, which acts on intracellular guanylyl
effect of captopril and other ACE inhibitors. It may cyclase. The answer is C.
be reduced or prevented by prior administration of
aspirin. These drugs are very commonly used in 10.
hypertensive diabetic patients because of their The two agents in this list that act via a nitric oxide
proven benefits in reducing diabetic renal damage. mechanism are hydralazine and nitroprusside (see
The ACE inhibitors are not associated with glaucoma; Table 11–2). However, hydralazine has a duration of
angioedema is not as common as cough; and action of hours, whereas nitroprusside acts for
headache and arrhythmias are rare. The answer is D. seconds to minutes and must be given by
6. intravenous infusion. The answer is H.
Hydralazine (choice B) is sometimes used in heart
failure. Beta blockers (choice C) are not associated QUESTIONS
with hematologic abnormalities, but methyldopa is. Questions 1–4. A 60-year-old man presents to his
The thiazide diuretics (choice E) often cause mild primary care physician with a complaint of severe
hyperglycemia, hyperuricemia, and hyperlipidemia chest pain when he walks uphill to his home in cold
but not lupus; hydralazine is associated with a lupus- weather. The pain disappears when he rests. After
like syndrome. Aliskiren (choice D) and other evaluation and discussion of treatment options, a
inhibitors of the renin-angiotensin-aldosterone decision is made to treat him with nitroglycerin.
system may cause hyperkalemia, not hypokalemia. 1. Which of the following is a common direct or
Verapamil (choice A) often causes constipation, reflex effect of nitroglycerin?
probably by blocking L-type calcium channels in the (A) Decreased heart rate
colon. The answer is A. (B) Decreased venous capacitance
7. (C) Increased afterload
Atenolol, but not prazosin, may decrease heart rate (D) Increased cardiac force
(choice A). Prazosin—but not atenolol—may (E) Increased diastolic myocardial fiber tension
increase cardiac output, a compensatory effect 2. In advising the patient about the adverse effects
(choice B). Prazosin may increase renin output (a he may notice, you point out that nitroglycerin in
compensatory response), but β blockers inhibit its moderate doses often produces certain symptoms.
release by the kidney (choice C). By reducing blood Which of the following effects might occur due to
pressure, both may increase central sympathetic the mechanism listed?
outflow (a compensatory response). Beta blockers (A) Constipation
do not cause orthostatic hypotension. The answer is (B) Dizziness due to reduced cardiac force of
D. contraction
(C) Diuresis due to sympathetic discharge (D) Prazosin
(D) Headache due to meningeal vasodilation (E) Verapamil
(E) Hypertension due to reflex tachycardia 8. A 45-year-old woman with hyperlipidemia and
3. One year later, the patient returns complaining frequent migraine headaches develops angina of
that his nitroglycerin works well when he takes it for effort. Which of the following is relatively
an acute attack but that he is now having more contraindicated because of her migraines?
frequent attacks and would like something to (A) Amlodipine
prevent them. Useful drugs for the prophylaxis of (B) Diltiazem
angina of effort include (C) Metoprolol
(A) Amyl nitrite (D) Nitroglycerin
(B) Esmolol (E) Verapamil
(C) Sublingual isosorbide dinitrate 9. When nitrates are used in combination with other
(D) Sublingual nitroglycerin drugs for the treatment of angina, which one of the
(E) Verapamil following combinations results in additive effects on
4. If a β blocker were to be used for prophylaxis in the variable specified?
this patient, what is the most probable mechanism (A) Beta blockers and nitrates on end-diastolic
of action in angina? cardiac size
(A) Block of exercise-induced tachycardia (B) Beta blockers and nitrates on heart rate
(B) Decreased end-diastolic ventricular volume (C) Beta blockers and nitrates on venous tone
(C) Increased double product (D) Calcium channel blockers and β blockers on
(D) Increased cardiac force cardiac force
(E) Decreased ventricular ejection time (E) Calcium channel blockers and nitrates on heart
5. A new 60-year-old patient presents to the medical rate
clinic with hypertension and angina. He is 1.8 meters
tall with a waist measurement of 1.1 m. Weight is 97 10. Certain drugs can cause severe hypotension
kg. Blood pressure is 150/95 and pulse 85. In when combined with nitrates. Which of the
considering adverse effects of possible drugs for following interacts with nitroglycerin by inhibiting
these conditions, you note that an adverse effect the metabolism of cGMP?
that nitroglycerin and prazosin have in common is (A) Atenolol
(A) Bradycardia (B) Hydralazine
(B) Impaired sexual function (C) Isosorbide mononitrate
(C) Lupus erythematosus syndrome (D) Nifedipine
(D) Orthostatic hypotension (E) Ranolazine
(E) Weight gain (F) Sildenafil
6. A man is admitted to the emergency department (G) Terbutaline
with a brownish cyanotic appearance, marked ANSWERS
shortness of breath, and hypotension. Which of the 1.
following is most likely to cause Nitroglycerin increases heart rate and venous
methemoglobinemia? capacitance and decreases afterload and diastolic
(A) Amyl nitrite fiber tension. It increases cardiac contractile force
(B) Isosorbide dinitrate because the decrease in blood pressure evokes a
(C) Isosorbide mononitrate compensatory increase in sympathetic discharge.
(D) Nitroglycerin The answer is D.
(E) Sodium cyanide 2.
7. Another patient is admitted to the emergency The nitrates relax many types of smooth muscle, but
department after a drug overdose. He is noted to the effect on motility in the colon is insignificant.
have hypotension and severe bradycardia. He has Nitroglycerin causes hypotension as a result of
been receiving therapy for hypertension and angina. arterial and venous dilation. Dilation of arteries in
Which of the following drugs in high doses causes the meninges has no effect on central nervous
bradycardia? system function but does cause headache. The
(A) Amlodipine answer is D.
(B) Isosorbide dinitrate 3.
(C) Nitroglycerin
The calcium channel blockers and the β blockers are increase the synthesis of cGMP. This combination is
generally effective in reducing the number of attacks synergistic. The answer is F.
of angina of effort, and most have durations of 4–8
h. Oral and transdermal nitrates have similar or QUESTIONS
longer durations. Amyl nitrite and the sublingual Questions 1–2. A 73-year-old man with an
nitrates have short durations of action inadequate response to other drugs is to receive
(a few minutes to 30 min). Esmolol (an intravenous β digoxin for chronic heart failure. He is in normal
blocker) must be given intravenously and also has a sinus rhythm with a heart rate of 88 and blood
very short duration of action. These drugs are of no pressure of 135/85 mm Hg.
value in prophylaxis. The answer is E. 1. Which of the following is the best-documented
4. mechanism of beneficial action of cardiac
Propranolol blocks tachycardia but has none of the glycosides?
other effects listed. Only revascularization increases (A) A decrease in calcium uptake by the sarcoplasmic
double product; drugs that decrease cardiac work reticulum
increase exercise time by decreasing double product. (B) An increase in ATP synthesis
The answer is A. (C) A modification of the actin molecule
5. (D) An increase in systolic cytoplasmic calcium levels
Both drugs cause venodilation and reduce venous (E) A block of cardiac β adrenoceptors
return sufficiently to cause some degree of postural 2. After your patient has been receiving digoxin for 3
hypotension. Bradycardia, lupus, weight gain, and wk, he presents to the emergency department with
urinary retention occur with neither of them, but an arrhythmia. Which one of the following is most
prazosin has been used to relieve urinary retention likely to contribute to the arrhythmogenic effect of
in men with prostatic hyperplasia. The answer is D. digoxin?
6. (A) Increased parasympathetic discharge
Read carefully! Nitrites, not nitrates, cause (B) Increased intracellular calcium
methemoglobinemia in adults. Methemoglobinemia (C) Decreased sympathetic discharge
is delibrately induced in one of the treatments of (D) Decreased intracellular ATP
cyanide poisoning. The answer is A. (E) Increased extracellular potassium
7. 3. A patient who has been taking digoxin for several
Isosorbide dinitrate (like all the nitrates) and years for atrial fibrillation and chronic heart failure is
prazosin can cause reflex tachycardia. Amlodipine, a about to receive atropine for another condition. A
dihydropyridine calcium channel blocker, causes common effect of digoxin (at therapeutic blood
much more vasodilation than cardiac depression and levels) that can be almost entirely blocked by
may also cause reflex tachycardia. Verapamil atropine is
typically slows heart rate and high doses may cause (A) Decreased appetite
severe bradycardia. The answer is E. (B) Headaches
8. (C) Increased atrial contractility
Acute migraine headache is associated with (D) Increased PR interval on ECG
vasodilation of meningeal arteries. Of the drugs (E) Tachycardia
listed, only nitroglycerin is commonly associated 4. A 65-year-old woman has been admitted to the
with headache. In fact, calcium channel blockers and coronary care unit with a left ventricular myocardial
β blockers have been used with some success as infarction. She develops acute severe heart failure
prophylaxis for migraine. The answer is D. with marked pulmonary edema, but no evidence of
9. peripheral edema. Which one of the following drugs
The effects of β blockers (or calcium channel would be most useful?
blockers) and nitrates on heart size, force, venous (A) Digoxin
tone, and heart rate are opposite. The effects of β (B) Furosemide
blockers and calcium channel blockers on the (C) Minoxidil
variables specified here are the same. The answer is (D) Propranolol
D. (E) Spironolactone
10. 5. A 72-year-old woman has long-standing heart
Sildenafil inhibits phosphodiesterase 5, an enzyme failure. Which one of the following drugs has been
that inactivates cGMP. The nitrates (via nitric oxide) shown to reduce mortality in chronic heart failure?
(A) Atenolol ANSWERS
(B) Digoxin 1.
(C) Dobutamine Digitalis does not decrease calcium uptake by the
(D) Furosemide sarcoplasmic reticulum or increase ATP synthesis; it
(E) Spironolactone does not modify actin. Cardiac adrenoceptors are
not affected. The most accurate description of
7. Which one of the following drugs is associated digitalis’s mechanism in this list is that it increases
with clinically useful or physiologically important systolic cytoplasmic calcium indirectly by inhibiting
positive inotropic effect? Na+/K+ ATPase and altering Na/Ca exchange. The
(A) Captopril answer is D.
(B) Dobutamine 2.
(C) Enalapril The effects of digitalis include increased vagal action
(D) Losartan on the heart (not arrhythmogenic) and increased
(E) Nesiritide intracellular calcium, including calcium overload, the
8. A 68-year-old man with a history of chronic heart most important cause of toxicity. Decreased
failure goes on vacation and abandons his low-salt sympathetic discharge and increased extracellular
diet. Three days later, he develops severe shortness potassium and magnesium reduce digitalis
of breath and is admitted to the local hospital arrhythmogenesis. The answer is B.
emergency department with significant pulmonary 3.
edema. The first-line drug of choice in most cases of The parasympathomimetic effects of digitalis can be
acute decompensation in patients with chronic heart blocked by muscarinic blockers such as atropine. The
failure is only parasympathomimetic effect in the list provided
(A) Atenolol is increased PR interval, a manifestation of slowed
(B) Captopril AV conduction. The answer is D.
(C) Carvedilol 4.
(D) Digoxin Acute severe congestive failure with pulmonary
(E) Diltiazem edema often requires a vasodilator that reduces
(F) Dobutamine intravascular pressures in the lungs. Furosemide has
(G) Enalapril such vasodilating actions in the context of acute
(H) Furosemide failure. Pulmonary edema also involves a shift of
(I) Metoprolol fluid from the intravascular compartment to the
(J) Spironolactone lungs. Minoxidil would decrease arterial pressure
9. Which of the following has been shown to prolong and increase the heart rate excessively. Digoxin has a
life in patients with chronic congestive failure in slow onset of action and lacks vasodilating effects.
spite of having a negative inotropic effect on cardiac Spironolactone is useful in chronic failure but not in
contractility? acute pulmonary edema. Pulmonary vasodilation
(A) Carvedilol and removal of edema fluid by diuresis are
(B) Digoxin accomplished by furosemide. The answer is B.
(C) Dobutamine 5.
(D) Enalapril Of the drugs listed, only spironolactone has been
(E) Furosemide shown to reduce mortality in this highly lethal
disease. Digoxin, dobutamine, and furosemide are
10. A 5-year-old child was vomiting and was brought used in the management of symptoms. The answer
to the emergency department with sinus arrest and is E.
a ventricular rate of 35 bpm. An empty bottle of his 7.
uncle’s digoxin was found where he was playing. Although they are extremely useful in heart failure,
Which of the following is the drug of choice in ACE inhibitors (eg, captopril, enalapril), and
treating a severe overdose of digoxin? angiotensin receptor blockers (ARBs, eg, losartan)
(A) Digoxin antibodies have no positive inotropic effect on the heart.
(B) Lidocaine infusion Nesiritide is a vasodilator with diuretic effects and
(C) Magnesium infusion renal toxicity. Dobutamine is a β1-selective
(D) Phenytoin by mouth adrenoceptor agonist. The answer is B.
(E) Potassium by mouth 8.
In both acute and chronic failure and systolic and (B) Increases contractility
diastolic heart failure, the initial treatment of choice (C) Increases PR interval
is usually furosemide. The answer is H. (D) Reduces abnormal automaticity
9. (E) Reduces resting potential
Several β blockers, including carvedilol, have been 4. A 36-year-old woman with a history of poorly
shown to prolong life in heart failure patients even controlled thyrotoxicosis has recurrent episodes of
though these drugs have a negative inotropic action tachycardia with severe shortness of breath. When
on the heart. Their benefits presumably result from she is admitted to the emergency department with
some other effect, and at least one β blocker has one of these episodes, which of the following drugs
failed to show a mortality benefit. The answer is A. would be most suitable?
10. (A) Amiodarone
The drug of choice in severe, massive overdose with (B) Disopyramide
any cardiac glycoside is digoxin antibody, Digibind. (C) Esmolol
The other drugs listed are used in moderate (D) Quinidine
overdosage associated with increased automaticity. (E) Verapamil
The answer is A. 5. A 16-year-old girl has paroxysmal attacks of rapid
heart rate with palpitations and shortness of breath.
QUESTIONS These episodes occasionally terminate
Questions 1 and 2. A 76-year-old patient with spontaneously but often require a visit to the
rheumatoid arthritis and chronic heart disease is emergency department of the local hospital. Her
being considered for treatment with procainamide. ECG during these episodes reveals an AV nodal
She is already receiving digoxin, hydrochlorothiazide, tachycardia. The antiarrhythmic of choice in most
and potassium supplements for her cardiac cases of acute AV nodal tachycardia is
condition. (A) Adenosine
1. In deciding on a treatment regimen with (B) Amiodarone
procainamide for this patient, which of the following (C) Flecainide
statements is most correct? (D) Propranolol
(A) A possible drug interaction with digoxin suggests (E) Verapamil
that digoxin blood levels should be obtained before 6. A 55-year-old man is admitted to the emergency
and after starting procainamide department and is found to have an abnormal ECG.
(B) Hyperkalemia should be avoided to reduce the Overdose of an antiarrhythmic drug is considered.
likelihood of procainamide toxicity Which of the following drugs is correctly paired with
(C) Procainamide cannot be used if the patient has its ECG effects?
asthma because it has a β-blocking effect (A) Quinidine: Increased PR and decreased QT
(D) Procainamide cannot be used if the patient has intervals
angina because it has a β-agonist effect (B) Flecainide: Increased PR, QRS, and QT intervals
(E) Procainamide is not active by the oral route (C) Verapamil: Increased PR interval
2. If this patient should take an overdose and (D) Lidocaine: Decreased QRS and PR interval
manifest severe acute procainamide toxicity with (E) Metoprolol: Increased QRS duration
markedly prolonged QRS, which of the following 7. A 60-year-old man comes to the emergency
should be given immediately? department with severe chest pain. ECG reveals
(A) A calcium chelator such as EDTA ventricular tachycardia with occasional normal sinus
(B) Digitalis beats, and ST-segment changes suggestive of
(C) Nitroprusside ischemia. A diagnosis of myocardial infarction is
(D) Potassium chloride made, and the man is admitted to the cardiac
(E) Sodium lactate intensive care unit. His arrhythmia should be treated
3. A 57-year-old man is admitted to the emergency immediately with
department with chest pain and a fast irregular heart (A) Adenosine
rhythm. The ECG shows an inferior myocardial (B) Digoxin
infarction and ventricular tachycardia. Lidocaine is (C) Lidocaine
ordered. When used as an antiarrhythmic drug, (D) Quinidine
lidocaine typically (E) Verapamil
(A) Increases action potential duration
8. Which of the following drugs slows conduction Calcium channel blockers are effective in
through the AV node and has its primary action supraventricular AV nodal tachycardias. However,
directly on L-type calcium channels? adenosine is just as effective in most acute nodal
(A) Adenosine tachycardias and is less toxic because of its
(B) Amiodarone extremely short duration of action. The answer is A.
(C) Diltiazem 6.
(D) Esmolol All the associations listed are incorrect except
(E) Flecainide verapamil (see Table 14–1). Because calcium
(F) Lidocaine blockers slow AV conduction, group 4 drugs such as
(G) Mexiletine verapamil and diltiazem increase PR interval and
(H) Procainamide have little effect on the other ECG variables. The
(I) Quinidine answer is C.
9. When working in outlying areas, this 62-year-old 7.
rancher is away from his house for 12–14 h at a Lidocaine has limited applications as an
time. He has an arrhythmia that requires chronic antiarrhythmic drug, but emergency treatment of
therapy. Which of the following has the longest half- myocardial infarction arrhythmias is one of the most
life of all antiarrhythmic drugs? important. Lidocaine is also useful in digoxin-induced
(A) Adenosine arrhythmias. After recovery from the acute phase of
(B) Amiodarone a myocardial infarction, β blockers are used for 2 yr
(C) Disopyramide or more to prevent sudden death arrhythmias. The
(D) Esmolol answer is C.
(E) Flecainide 8.
(F) Lidocaine Diltiazem is the calcium channel blocker in this list.
(G) Mexiletine (Beta blockers also slow AV conduction but have
(H) Procainamide much smaller effects on calcium channels.) The
(I) Quinidine answer is C.
(J) Verapamil 9.
Amiodarone has the longest half-life of all the
ANSWERS antiarrhythmics (weeks). The answer is B.
1.
Hyperkalemia facilitates procainamide toxicity. QUESTIONS
Procainamide is active by the oral route and has a 1. A 70-year-old retired businessman is admitted
duration of action of 2–4 h (in the prompt-release with a history of recurrent heart failure and
form). Procainamide has no significant documented metabolic derangements. He has marked peripheral
interaction with digoxin and no significant β-agonist edema and metabolic alkalosis. Which of the
or β-blocking action. The answer is B. following drugs is most appropriate for the
2. treatment of his edema?
The most effective therapy for procainamide toxicity (A) Acetazolamide
appears to be concentrated sodium lactate. This (B) Digoxin
drug may (1) increase sodium current by increasing (C) Dobutamine
the sodium ion gradient and (D) Eplerenone
(2) reduce drug-receptor binding by alkalinizing the (E) Hydrochlorothiazide
tissue. The answer is E. 2. A 50-year-old man has a history of frequent
3. episodes of renal colic with calcium-containing renal
Lidocaine reduces automaticity in the ventricles; the stones. A careful workup indicates that he has a
drug does not alter resting potential or AP duration defect in proximal tubular calcium reabsorption,
and does not increase contractility. The answer is D. which results in high concentrations of calcium salts
4. in the tubular urine. The most useful diuretic agent
Beta blockers are the most effective agents in acute in the treatment of recurrent calcium stones is
thyrotoxic arrhythmias. Esmolol is a parenteral, (A) Chlorthalidone
rapid-acting β blocker (see Chapter 10). The answer (B) Diazoxide
is C. (C) Ethacrynic acid
5. (D) Mannitol
(E) Spironolactone (E) Spironolactone
3. Which of the following is an important effect of 9. A 58-year-old woman with lung cancer has
chronic therapy with loop diuretics? abnormally low serum osmolality and hyponatremia.
(A) Decreased urinary excretion of calcium A drug that increases the formation of dilute urine
(B) Elevation of blood pressure and is used to treat SIADH is
(C) Elevation of pulmonary vascular pressure (A) Acetazolamide
(D) Metabolic alkalosis (B) Amiloride
(E) Teratogenic action in pregnancy (C) Desmopressin
(D) Ethacrynic acid
5. Which of the following diuretics would be most (E) Furosemide
useful in the acute treatment of a comatose patient (F) Hydrochlorothiazide
with traumatic brain injury and cerebral edema? (G) Mannitol
(A) Acetazolamide (H) Spironolactone
(B) Amiloride (I) Triamterene
(C) Chlorthalidone (J) Tolvaptan
(D) Furosemide 10. A graduate student is planning to make a high-
(E) Mannitol altitude climb in South America while on vacation.
6. A 62-year-old man with advanced prostate cancer He will not have time to acclimate slowly to altitude.
is admitted to the emergency department with A drug that is useful in preventing high-altitude
mental obtundation. An electrolyte panel shows a sickness is
serum calcium of 16.5 (normal 8.5–10.5 mg/dL). (A) Acetazolamide
Which of the following therapies would be most (B) Amiloride
useful in the management of severe hypercalcemia? (C) Demeclocycline
(A) Acetazolamide plus saline infusion (D) Desmopressin
(B) Furosemide plus saline infusion (E) Ethacrynic acid
(C) Hydrochlorothiazide plus saline infusion ANSWERS
(D) Mannitol plus saline infusion 1.
(E) Spironolactone plus saline infusion Although acetazolamide is rarely used in heart
7. A 60-year-old patient complains of paresthesias failure, carbonic anhydrase inhibitors are quite
and occasional nausea associated with one of her valuable in patients with edema and metabolic
drugs. She is found to have hyperchloremic alkalosis. The high bicarbonate levels in these
metabolic acidosis. She is probably taking patients make them particularly susceptible to the
(A) Acetazolamide for glaucoma action of carbonic anhydrase inhibitors. Digoxin is
(B) Amiloride for edema associated with useful in chronic systolic failure but is not first-line
aldosteronism therapy. Dobutamine is appropriate only when
(C) Furosemide for severe hypertension and heart diuresis has already been accomplished in severe
failure acute failure. Hydrochlorothiazide and
(D) Hydrochlorothiazide for hypertension spironolactone are not adequate for first-line
(E) Mannitol for cerebral edema therapy of edema in failure. The answer is A.
8. A 70-year-old woman is admitted to the 2.
emergency department because of a “fainting spell” The thiazides are useful in the prevention of calcium
at home. She appears to have suffered no trauma stones because these drugs reduce tubular calcium
from her fall, but her blood pressure is 120/60 when concentration, probably by increasing passive
lying down and 60/20 when she sits up. Neurologic proximal tubular and distal convoluted tubule
examination and an ECG are within normal limits reabsorption of calcium. In contrast, the loop agents
when she is lying down. Questioning reveals that she (choice C) facilitate calcium excretion. Diazoxide is a
has recently started taking “water pills” (diuretics) thiazide-like vasodilator molecule but has no diuretic
for a heart condition. Which of the following drugs is action; in fact, it may cause sodium retention. It is
the most likely cause of her fainting spell? used in hypertension and insulinoma (see Chapter
(A) Acetazolamide 11). The answer is A.
(B) Amiloride
(C) Furosemide 3.
(D) Hydrochlorothiazide
Loop diuretics increase urinary calcium excretion and unknown peptides. The effects of serotonin in this
decrease blood pressure (in hypertension) and patient are most likely to include
pulmonary vascular pressure (in congestive heart (A) Constipation
failure). They have no recognized teratogenic action. (B) Episodes of bronchospasm
They cause metabolic alkalosis (Table 15–1). Loop (C) Hypersecretion of gastric acid
diuretics also cause ototoxicity. The answer is D. (D) Hypotension
5. (E) Urinary retention
An osmotic agent is needed to remove water from 2. A 23-year-old woman suffers from recurrent
the cells of the edematous brain and reduce episodes of angioneurotic edema with release of
intracranial pressure rapidly. The answer is E. histamine and other mediators. Which of the
6. following drugs is the most effective physiologic
Diuretic therapy of hypercalcemia requires a antagonist of histamine in smooth muscle?
reduction in calcium reabsorption in the thick (A) Cetirizine
ascending limb, an effect of loop diuretics. However, (B) Epinephrine
a loop diuretic alone would reduce blood volume (C) Granisetron
around the remaining calcium so that serum calcium (D) Ranitidine
would not decrease appropriately. Therefore, saline (E) Sumatriptan
infusion should accompany the loop diuretic. The 3. A 20-year-old woman is taking diphenhydramine
answer is B. for severe hay fever. Which of the following adverse
7. effects is she most likely to report?
Paresthesias and gastrointestinal distress are (A) Muscarinic increase in bladder tone
common adverse effects of acetazolamide, (B) Nausea
especially when it is taken chronically, as in (C) Nervousness, anxiety
glaucoma. The observation that the patient has (D) Sedation
metabolic acidosis also suggests the use of (E) Vertigo
acetazolamide. The answer is A. 4. A laboratory study of new H2 blockers is planned.
8. Which of the following will result from blockade of
The case history suggests that the syncope (fainting) H2 receptors?
is associated with diuretic use. Complications of (A) Increased cAMP (cyclic adenosine
diuretics that can result in syncope include both monophosphate) in cardiac muscle
postural hypotension (which this patient exhibits) (B) Decreased channel opening in enteric nerves
due to excessive reduction of blood volume and (C) Decreased cAMP in gastric mucosa
arrhythmias due to excessive potassium loss. (D) Increased IP3 (inositol trisphosphate) in platelets
Potassium wasting is more common with thiazides (E) Increased IP3 in smooth muscle
(because of their long duration of action), but these 5. You are asked to consult on a series of cases of
drugs rarely cause reduction of blood volume drug toxicities. Which of the following is a
sufficient to result in orthostatic hypotension. The recognized adverse effect of cimetidine?
answer is C, furosemide. (A) Blurred vision
9. (B) Diarrhea
Retention of water with hyponatremia and inability (C) Orthostatic hypotension
to form dilute urine in the fully hydrated condition is (D) P450 hepatic enzyme inhibition
characteristic of SIADH. Antagonists of ADH are (E) Sedation
needed to treat this condition. The answer is J, 6. A 40-year-old patient is about to undergo cancer
tolvaptan. chemotherapy with a highly emetogenic (nausea-
10. and vomiting-causing) drug combination. The
Carbonic anhydrase inhibitors are useful in the antiemetic drug most likely to be included in her
prevention of altitude sickness. The answer is A. regimen is
(A) Bromocriptine
QUESTIONS (B) Cetirizine
1. Your 37-year-old patient has been diagnosed with (C) Cimetidine
a rare metastatic carcinoid tumor. This neoplasm is (D) Ketanserin
releasing serotonin, bradykinin, and several (E) Ondansetron
7. The hospital Pharmacy Committee is preparing a 2.
formulary for staff use. Which of the following is a The smooth muscle effects of histamine are
correct application of the drug mentioned? mediated mainly by H1 receptors. Cetirizine is a
(A) Alosetron: for obstetric bleeding pharmacologic antagonist of histamine at these
(B) Cetirizine: for hay fever receptors. Granisetron is a 5-HT3 antagonist.
(C) Ergonovine: for Alzheimer’s disease Sumatriptan is a 5-HT1D/1B agonist. Ranitidine is a
(D) Ondansetron: for acute migraine headache histamine antagonist but blocks the H2 receptor in
(E) Ranitidine: for Parkinson’s disease the stomach and the heart, not H1 receptors in
8. A 26-year-old woman presents with amenorrhea smooth muscle. Epinephrine has a physiologic
and galactorrhea. Her prolactin level is grossly antagonist action that reverses histamine’s effects
elevated (200 ng/mL vs normal 20 ng/mL). Which of on smooth muscle. The answer is B.
the following is most useful in the treatment of 3.
hyperprolactinemia? H1 blockers do not activate muscarinic receptors,
(A) Bromocriptine mediate vasoconstriction, or cause vertigo. Some
(B) Cimetidine relieve vertigo or motion sickness. They do not cause
(C) Ergotamine nervousness or anxiety. Diphenhydramine is a
(D) Ketanserin potent sedative. The answer is D.
(E) LSD 4.
(F) Ondansetron H2 receptors are Gs-protein-coupled receptors, like
(G) Sumatriptan β adrenoceptors. Blockade of this system will cause
9. A 28-year-old office worker suffers from intense a decrease in cAMP. The answer is C.
migraine headaches. Which of the following is a 5.
serotonin agonist useful for aborting an acute The older H1 blockers, not H2 blockers, cause
migraine headache? blurred vision, orthostatic hypotension, and
(A) Bromocriptine sedation. Neither group typically causes diarrhea.
(B) Cimetidine Cimetidine (unlike other H2 blockers) is a potent
(C) Ephedrine CYP3A4 inhibitor. The answer is D.
(D) Ketanserin 6.
(E) Loratadine Ondansetron and other 5-HT3 antagonists have
(F) Ondansetron significant antiemetic effects. Diphenhydramine and
(G) Sumatriptan prednisone are also used for this purpose. The
answer is E.
10. A 33-year-old woman attempted to induce an 7.
abortion using ergotamine. She is admitted to the Alosetron is indicated in irritable bowel syndrome.
emergency department with severe pain in both Ergonovine is used in uterine bleeding. Ondansetron
legs. On examination, her legs are cold and pale with is useful for chemotherapy-induced emesis.
absent arterial pulses. Which of the following is the Cetirizine, a second-generation H1 blocker, is used in
most useful antidote for reversing severe ergot- the treatment of hay fever. The answer is B.
induced vasospasm? 8.
(A) Bromocriptine Bromocriptine is an effective dopamine agonist in
(B) Cimetidine the CNS with the advantage of oral activity. The drug
(C) Ergotamine inhibits prolactin secretion by activating pituitary
(D) Ketanserin dopamine receptors. The answer is A.
(E) LSD 9.
(F) Nitroprusside Sumatriptan, an agonist at 5-HT1D receptors, is
(G) Sumatriptan indicated for prevention or treatment of migraine
(H) Ondansetron and cluster headaches. Ergotamine (not on the list) is
ANSWERS also effective for acute migraine but is produced by
1. the fungus Claviceps purpurea. The answer is G.
Serotonin causes bronchospasm, but the other 10.
effects listed are not observed. Carcinoid is A very powerful vasodilator is necessary to reverse
associated with diarrhea and hypertension. The ergot-induced vasospasm; nitroprusside is such a
answer is B. drug (see Chapter 11). The answer is F.
found to increase in the blood of patients treated
QUESTIONS with large doses of diuretics. Which of the following
1. Field workers exposed to a plant toxin develop is the most likely endogenous peptide?
painful fluid-filled blisters. Analysis of the blister fluid (A) Angiotensin I
reveals high concentrations of a peptide. Which of (B) Angiotensin II
the following is a peptide that causes increased (C) Atrial natriuretic peptide
capillary permeability and edema? (D) Bradykinin
(A) Angiotensin II (E) Calcitonin gene-related peptide
(B) Bradykinin (F) Endothelin
(C) Captopril (G) Neuropeptide Y
(D) Histamine (H) Renin
(E) Losartan (I) Substance P
2. In a laboratory study of several peptides, one is (J) Vasoactive intestinal peptide
found that decreases peripheral resistance but 7. Which of the following is a vasodilator that
constricts veins. Which of the following causes increases in the blood or tissues of patients treated
arteriolar vasodilation and venoconstriction? with captopril?
(A) Angiotensin II (A) Angiotensin II
(B) Bradykinin (B) Bradykinin
(C) Endothelin-1 (C) Brain natriuretic peptide
(D) Substance P (D) Calcitonin gene-related peptide
(E) Vasoactive intestinal peptide (E) Endothelin
3. Which of the following endogenous molecules is (F) Neuropeptide Y
elevated in heart failure and when given as a drug is (G) Renin
a vasodilator with significant renal toxicity? 8. Which of the following is an antagonist at NK1
(A) Angiotensin I receptors and is used to prevent or reduce
(B) Angiotensin II chemotherapy-induced nausea and vomiting?
(C) Histamine (A) Angiotensin I
(D) Nesiritide (B) Aprepitant
(E) Vasoactive intestinal peptide (C) Bosentan
4. A 45-year-old painter presents with respiratory (D) Bradykinin
symptoms and careful workup reveals idiopathic (E) Brain natriuretic peptide
pulmonary hypertension. Which of the following (F) Enalapril
binds endothelin receptors and is approved for use (G) Ondansetron
in pulmonary hypertension?
(A) Aliskiren I ANSWERS
(B) Bosentan 1.
(C) Capsaicin Histamine and bradykinin both cause a marked
(D) Losartan increase in capillary permeability that is often
(E) Nesiritide associated with edema, but histamine is not a
5. A 60-year-old financial consultant presents with peptide. The answer is B.
severe pain in a neuronal dermatome region of her 2.
chest. This area was previously affected by a herpes Substance P is a potent arterial vasodilator and
zoster rash. Which of the following might be of venoconstrictor. The answer is D.
benefit in controlling this post-herpetic pain? 3.
(A) Aliskiren BNP is an atrial and brain peptide found in increased
(B) Aprepitant amounts in patients with heart failure. The
(C) Bosentan commercial formulation (nesiritide) is approved for
(D) Capsaicin use in severe acute heart failure but has significant
(E) Captopril renal toxicity. The answer is D.
(F) Losartan 4.
(G) Nesiritide Aliskiren, captopril, and losartan are used in primary
6. In a phase 2 clinical trial in hypertensive patients, hypertension. Bosentan, an endothelin antagonist, is
an endogenous octapeptide vasoconstrictor was used in pulmonary hypertension. The answer is B.
5. (A) Angiotensin II
Substance P is an important pain-mediating (B) Ergotamine
neurotransmitter peptide and appears to be (C) Prostaglandin PGF2α
involved in post-herpetic pain as well as arthritic (D) Prostacyclin
pain. Capsaicin can be used topically to deplete (E) Thromboxane
substance P stores from sensory nerves. The answer 4. A 19-year-old woman complains of severe
is D. dysmenorrhea. A uterine stimulant derived from
6. membrane lipid in the endometrium is
Angiotensin II, an octapeptide, increases when blood (A) Angiotensin II
volume decreases (a diuretic effect) because the (B) Oxytocin
compensatory response causes an increase in renin (C) Prostacyclin (PGI2)
secretion. Its precursor, angiotensin I, would also (D) Prostaglandin PGF2α
increase, but it is a decapeptide. The answer is B. (E) Serotonin
7. 5. Inflammation is a complex tissue reaction that
Bradykinin increases because the enzyme inhibited includes the release of cytokines, leukotrienes,
by captopril, converting enzyme, degrades kinins in prostaglandins, and peptides. Prostaglandins
addition to synthesizing angiotensin II (see Figure involved in inflammatory processes are typically
11–3). The answer is B. produced from arachidonic acid by which of the
following enzymes?
8. (A) Cyclooxygenase-1
Aprepitant and ondansetron are both used to reduce (B) Cyclooxygenase-2
or prevent chemotherapy-induced nausea and (C) Glutathione-S-transferase
vomiting. Ondansetron is an antagonist at 5-HT3 (D) Lipoxygenase
receptors. The answer is B. (E) Phospholipase A2
6. A newborn infant is diagnosed with transposition
QUESTIONS of the great vessels, wherein the aorta exits from the
1. A 50-year-old woman with moderately severe right ventricle and the pulmonary artery from the
arthritis has been treated with nonsteroidal anti- left ventricle. Which of the following drugs is likely to
inflammatory drugs for 6 mo. She now complains of be used in preparation for surgical correction of this
heartburn and indigestion. You give her a anomaly?
prescription for a drug to be taken along with the (A) Aspirin
anti-inflammatory agent, but 2 d later she calls the (B) Leukotriene LTC4
office complaining that your last prescription has (C) Prednisone
caused severe diarrhea. Which of the following is (D) Prostaglandin PGE1
most likely to be associated with increased (E) Prostaglandin PGF2α
gastrointestinal motility and diarrhea?
(A) Aspirin 7. A patient with a bleeding tendency presents in the
(B) Famotidine hematology clinic. He is apparently taking large
(C) Leukotriene LTB4 amounts of an unidentified drug that inhibits
(D) Misoprostol platelet activity. Which of the following is taken
(E) Zileuton orally and directly and reversibly inhibits platelet
2. Which of the following drugs inhibits cyclooxygenase?
thromboxane synthesis much more effectively than (A) Alprostadil
prostacyclin synthesis? (B) Aspirin
(A) Aspirin (C) Ibuprofen
(B) Hydrocortisone (D) Leukotriene LTC4
(C) Ibuprofen (E) Misoprostol
(D) Indomethacin (F) Prednisone
(E) Zileuton (G) Prostacyclin
3. A 57-year-old man has severe pulmonary (H) Zafirlukast
hypertension and right ventricular hypertrophy. (I) Zileuton
Which of the following agents causes vasodilation 8. Which of the following is a component of slow-
and may be useful in pulmonary hypertension? reacting substance of anaphylaxis (SRS-A)?
(A) Alprostadil Although serotonin and, in some species, histamine
(B) Aspirin may cause uterine stimulation, these substances are
(C) Leukotriene LTB4 not derived from membrane lipid. Similarly, oxytocin
(D) Leukotriene LTC4 causes uterine contraction, but it is a peptide
(E) Misoprostol hormone released from the posterior pituitary.
(F) Prednisone Prostacyclin relaxes the uterus (Table 18–1). The
(G) Prostacyclin answer is D.
(H) Zafirlukast 5.
(I) Zileuton See Figure 18–1. Phospholipase A2 converts
9. A 17-year-old patient complains that he develops membrane phospholipid to arachidonic acid.
wheezing and severe shortness of breath whenever Cyclooxygenases convert arachidonic acid to
he takes aspirin for headache. Increased levels of prostaglandins. COX-1 products appear to be
which of the following may be responsible, in part, important in normal physiologic processes. COX-2 is
for some cases of aspirin hypersensitivity? the enzyme responsible for this reaction in
(A) Alprostadil inflammatory cells. The answer is B.
(B) Hydrocortisone 6.
(C) Ibuprofen Infants with great vessel transposition pump venous
(D) Leukotriene LTC4 blood to the aorta and oxygenated blood back to the
(E) Misoprostol lungs. Therefore, they require surgical correction as
(F) PGE2 soon as they are strong enough to withstand the
(G) Prostacyclin procedure. In the meantime, they are dependent on
(H) Thromboxane a patent ductus arteriosus to allow some oxygenated
(I) Zileuton blood to flow from the left ventricle via the
10. Which of the following is a leukotriene receptor pulmonary artery and ductus to the aorta. The
blocker? ductus can be prevented from closing by infusing the
(A) Alprostadil vasodilator PGE1. The answer is D.
(B) Aspirin 7.
(C) Ibuprofen Aspirin is a direct but irreversible inhibitor of
(D) Leukotriene LTC4 cyclooxygenase. NSAIDs other than aspirin (such as
(E) Montelukast ibuprofen) are reversible inhibitors of COX.
(F) Prednisone Corticosteroids reduce the synthesis of
(G) Prostacyclin cyclooxygenase. The answer is C.
(H) Zileuton 8.
ANSWERS The leukotriene C and D series are major
1. components of SRS-A. Leukotriene LTB4 is a
Aspirin and zileuton rarely cause diarrhea. LTB4 is a chemotactic eicosanoid. The answer is D.
chemotactic factor. Famotidine is an H2 blocker that 9.
does not cause diarrhea (Chapter 16). The answer is When cyclooxygenase is blocked, leukotrienes may
D. be produced in increased amounts by diversion of
2. prostaglandin precursors into the lipoxygenase
Hydrocortisone and other corticosteroids inhibit pathway (Figure 18–1). In patients with aspirin
phospholipase. Ibuprofen and indomethacin inhibit hypersensitivity, this might precipitate the
cyclooxygenase reversibly, whereas zileuton inhibits bronchoconstriction often observed in this
lipoxygenase. Because aspirin inhibits condition. The answer is D.
cyclooxygenase irreversibly, its action is more 10.
effective in platelets, which lack the ability to Zileuton blocks the synthesis of leukotrienes.
synthesize new enzyme, than in the endothelium. Montelukast and zafirlukast block LTD4 receptors.
The answer is A. The answer is E.
3.
Prostacyclin (PGI2) is a very potent vasodilator. All QUESTIONS
the other choices in the list are vasoconstrictors. The 1. Which one of the following is not a nitric oxide
answer is D. donor but causes it to be synthesized and released
4.
from endogenous precursors, resulting in Nitroprusside is the only molecule in this list that
vasodilation? spontaneously releases NO in the bloodstream. The
(A) Acetylcholine answer is E.
(B) Arginine 3.
(C) Isosorbide mononitrate The inducible form of NOS is associated with
(D) Nitroglycerin inflammation, and the enzyme is found in highest
(E) Nitroprusside concentration in macrophages, cells that are
2. A molecule that releases nitric oxide in the blood particularly involved in inflammation. The answer is
is C.
(A) Citrulline 4.
(B) Histamine Arginine is the substrate and citrulline and NO are
(C) Isoproterenol the products of NOS. The answer is C.
(D) Nitroglycerin 5.
(E) Nitroprusside NO does not cause arrhythmias or constipation. It
3. The inducible isoform of nitric oxide synthase causes bronchodilation and may hasten graft
(iNOS, isoform 2) is found primarily in which of the rejection. NO does cause pulmonary vasodilation.
following? The answer is E.
(A) Cartilage 6.
(B) Eosinophils Heme and hemoglobin inactivate NO. The answer is
(C) Macrophages D.
(D) Platelets QUESTIONS
(E) Vascular endothelial cells 1. One effect that theophylline, nitroglycerin,
4. The primary endogenous substrate for the enzyme isoproterenol, and histamine have in common is
nitric oxide synthase (NOS) is (A) Direct stimulation of cardiac contractile force
(A) Acetylcholine (B) Tachycardia
(B) Angiotensinogen (C) Bronchodilation
(C) Arginine (D) Postural hypotension
(D) Citrulline (E) Throbbing headache
(E) Heme 2. A 23-year-old woman is using an albuterol inhaler
5. Which of the following is a recognized effect of for frequent acute episodes of asthma and
nitric oxide (NO)? complains of symptoms that she ascribes to the
(A) Arrhythmia albuterol. Which of the following is not a recognized
(B) Bronchoconstriction action of albuterol?
(C) Constipation (A) Diuretic effect
(D) Inhibition of acute graft rejection (B) Positive inotropic effect
(E) Pulmonary vasodilation (C) Skeletal muscle tremor
6. Which of the following is an endogenous (D) Smooth muscle relaxation
inhibitor/inactivator of nitric oxide? (E) Tachycardia
(A) Arginine 3. A 10-year-old child has severe asthma and was
(B) Angiotensinogen hospitalized
(C) Arachidonic acid 5 times between the ages of 7 and 9. He is now
(D) Hemoglobin receiving outpatient medications that have greatly
(E) Thromboxane reduced the frequency of severe attacks. Which of
ANSWERS the following is most likely to have adverse effects
1. when used daily over long periods for severe
Nitroprusside and organic nitrites (eg, amyl nitrite) asthma?
and nitrates (eg, nitroglycerin, isosorbide dinitrate, (A) Albuterol by aerosol
and isosorbide mononitrate) contain NO groups that (B) Beclomethasone by aerosol
can be released as NO. Arginine is the normal source (C) Ipratropium by inhaler
of endogenous NO. Acetylcholine stimulates the (D) Prednisone by mouth
synthesis of NO from arginine. The answer is A. (E) Theophylline in long-acting oral form
2. 4–5. A 16-year-old patient is in the emergency
department receiving nasal oxygen. She has a heart
rate of 125 bpm, a respiratory rate of 40 (I) Zileuton
breaths/min, and a peak expiratory flow <50% of the
predicted value. Wheezing and rales are audible 8. Which of the following in its parenteral form is
without a stethoscope. life-saving in severe status asthmaticus and acts, at
4. Which of the following drugs does not have a least in part, by inhibiting phospholipase A2?
direct bronchodilator effect? (A) Aminophylline
(A) Epinephrine (B) Cromolyn
(B) Terbutaline (C) Epinephrine
(C) Prednisone (D) Ipratropium
(D) Theophylline (E) Metaproterenol
(E) Ipratropium (F) Metoprolol
5. After successful treatment of the acute attack, the (G) Prednisone
patient was referred to the outpatient clinic for (H) Salmeterol
follow-up treatment for asthma. Which of the (I) Zafirlukast
following is not an established prophylactic strategy (J) Zileuton
for asthma? 9. Which of the following has a slow onset but long
(A) Avoidance of antigen exposure duration of action and is always used in combination
(B) Blockade of histamine receptors with a corticosteroid by inhalation?
(C) Blockade of leukotriene receptors (A) Aminophylline
(D) IgE antibody blockade (B) Cromolyn
(E) Inhibition of phospholipase A2 (C) Epinephrine
6. Mr Green is a 60-year-old former smoker with (D) Ipratropium
cardiac disease and severe chronic obstructive (E) Metaproterenol
pulmonary disease (COPD) associated with frequent (F) Metoprolol
episodes of bronchospasm. Which of the following is (G) Prednisone/prednisolone
a bronchodilator useful in COPD and least likely to (H) Salmeterol
cause cardiac arrhythmia? (I) Zafirlukast
(A) Aminophylline (J) Zileuton
(B) Cromolyn 10. Oral medications are popular for the treatment
(C) Epinephrine of asthma in children because young children may
(D) Ipratropium have difficulty with the proper use of aerosol
(E) Metaproterenol inhalers. Which of the following is an orally active
(F) Metoprolol inhibitor of leukotriene receptors?
(G) Prednisone (A) Albuterol
(H) Salmeterol (B) Aminophylline
(I) Zafirlukast (C) Ipratropium
(J) Zileuton (D) Montelukast
7. A 22-year-old man is brought to the emergency (E) Zileuton
department after suffering seizures resulting from ANSWERS
an overdose of a drug he has been taking. His friends 1.
state that he took the drug orally and sometimes Theophylline does not ordinarily cause headache or
had insomnia after taking it. Which of the following postural hypotension. Nitroglycerin does not cause
is a direct bronchodilator that is most often used in direct cardiac stimulation but does evoke a
asthma by the oral route and is capable of causing compensatory sympathetic reflex. Histamine does
insomnia and seizures? not cause bronchodilation. The answer is B.
(A) Cromolyn 2.
(B) Epinephrine Albuterol is a β2-selective receptor agonist, but in
(C) Ipratropium moderate to high doses it produces β1 cardiac
(D) Metaproterenol effects as well as β2-mediated smooth and skeletal
(E) Metoprolol muscle effects. It does not cause diuresis. The
(F) Prednisone answer is A.
(G) Salmeterol 3.
(H) Theophylline
Chronic systemic corticosteroids have important (D) Glycine
toxicities (see Chapter 39). If oral corticosteroids (E) Substance P
must be used, alternate-day therapy is preferred 2. Neurotransmitters may
because it interferes less with normal growth and (A) Increase chloride conductance to cause inhibition
metabolism in children. The answer is D. (B) Increase potassium conductance to cause
inhibition
4. (C) Increase sodium conductance to cause excitation
Although extremely important in severe chronic (D) Increase calcium conductance to cause excitation
asthma and status asthmaticus, corticosteroids do (E) Exert all of the above actions
not have a demonstrable direct bronchodilator 3. All of the listed neurotransmitters change
action. The answer is C. membrane excitability by decreasing K+ conductance
5. EXCEPT
Histamine does not appear to play a significant role (A) Acetylcholine
in asthma, and antihistaminic drugs, even in high (B) Dopamine
doses, are of little or no value. Antigen avoidance is (C) Glutamic acid
well established. Blockade of leukotriene receptors (D) Norepinephrine
with montelukast; inhibition of phospholipase with (E) Serotonin
corticosteroids; and inhibition of mediator release 4. Which of the following receptors shares the same
with the IgE antibody are also useful. The answer is potassium channel as the 5-HT1A receptor?
B. (A) Dopamine D2 receptor
6. (B) GABAB receptor
Ipratropium or a similar antimuscarinic agent is the (C) Mu opioid receptor
bronchodilator that is most likely to be useful in (D) Muscarinic M1 receptor
COPD without causing arrhythmias. The answer is D. (E) Substance P receptor
7. 5. Which of the following chemicals is most likely to
Theophylline is a bronchodilator that is active by the function as a neurotransmitter in hierarchical
oral route. It causes insomnia in therapeutic doses systems?
and seizures in overdosage. The answer is H. (A) GABA
8. (B) Glutamate
Parenteral corticosteroids such as prednisolone (the (C) Met-enkephalin
active metabolite of prednisone) are lifesaving in (D) Nitric oxide
status asthmaticus. They probably act by reducing (E) Norepinephrine
production of leukotrienes (see Chapter 18). The 6. Activation of metabotropic receptors located
answer is G. presynaptically causes inhibition by decreasing the
9. inward flux of
Salmeterol is a β2-selective agonist that has a slow (A) Calcium
onset and long duration of action. Used alone, it (B) Chloride
increases asthma mortality, but in combination with (C) Potassium
inhaled corticosteroids prophylactic use improves (D) Sodium
asthma control. The answer is H. (E) None of the above
10. 7. This transmitter is mostly located in diffuse
Zileuton is an inhibitor of the lipoxygenase enzyme neuronal systems in the CNS, with cell bodies
involved in the synthesis of leukotrienes. particularly in the raphe nuclei. It appears to play a
Montelukast and zafirlukast are leukotriene major role in the expression of mood states, and
antagonists at the leukotriene receptor. The answer many antidepressant drugs are thought to increase
is D. its functional activity.
(A) Acetylcholine
QUESTIONS (B) Dopamine
1. Which of the following chemicals does not satisfy (C) GABA
the criteria for a neurotransmitter role in the CNS? (D) Glutamate
(A) Acetylcholine (E) Serotonin
(B) Cyclic AMP
(C) Dopamine
8. Cyclic adenosine monophosphate (cAMP) glutamate (metabotropic), norepinephrine (α1 and
functions as a diffusible second messenger after β1), and serotonin (5-HT2A). The answer is B.
activation of 4.
(A) Acetylcholine M1 receptors GABAB receptors and 5-HT1A receptors share the
(B) Beta1 adrenoceptors same potassium ion channel, with a G protein
(C) 5-HT3 receptors involved in the coupling mechanism. The spasmolytic
(D) GABAA receptors drug baclofen is an activator of GABAB receptors in
(E) Glutamate NMDA receptors the spinal cord. The anxiolytic drug buspirone may
9. One of the first neurotransmitter receptors to be act as a partial agonist at brain 5-HT1A receptors.
identified in the CNS is located on the Renshaw cell The answer is B.
in the spinal cord. Activation of this receptor results 5.
in excitation via an increase in cation (Na+, K+ ) Catecholamines (dopamine, norepinephrine), opioid
conductance independently of G protein-coupled peptides, and serotonin act as neurotransmitters in
mechanisms. Which of the following compounds is nonspecific or diffuse neuronal systems. Glutamate
most likely to activate this receptor? is the primary excitatory transmitter in hierarchical
(A) Dopamine neuronal systems. These systems also contain
(B) Glycine numerous inhibitory neurons, which use GABA and
(C) GABA glycine. Nitric oxide, though present in many brain
(D) Nicotine regions, does not meet the critera for a CNS
(E) Serotonin neurotransmitter. The answer is B.
10. This neurotransmitter, found in high 6.
concentrations in cell bodies in the pons and brain Activation of metabotropic receptors located
stem, can exert both excitatory and inhibitory presynaptically results in the inhibition of calcium
actions. Multiple receptor subtypes have been influx with a resultant decrease in the release of
identified, some of which are targets for drugs that neurotransmitter from nerve endings. This type of
can exert both CNS and peripheral actions. presynaptic inhibition occurs after activation of
(A) Acetylcholine dopamine D2, norepinephrine α2, glutamate, and
(B) Beta-endorphin mu opioid peptide receptors. The answer is A.
(C) Glycine 7.
(D) Glutamate Amine transmitters thought to be involved in the
(E) Norepinephrine control of mood states include norepinephrine and
ANSWERS serotonin. Cell bodies of serotonergic neurons are
1. found in the raphe nuclei. Many of the drugs used
Cyclic AMP (cAMP) is a mediator in many receptor for the treatment of major depressive disorders act
mechanisms in the CNS, including those for to increase serotonergic activity in the CNS. The
acetylcholine (M2), and norepinephrine (β1). answer is E.
However, the characteristics of cAMP do not satisfy
the criteria for a neurotransmitter role (see A. QUESTIONS
Criteria for Transmitter Status). The answer is B. 1. A 43-year-old very overweight man complains of
2. not sleeping well and feeling tired during the day. He
Activation of chloride or potassium ion channels says that his wife is the cause of the problem
commonly generates inhibitory postsynaptic because she wakes him up several times during the
potentials (IPSPs). Activation of sodium and calcium night because of his loud snores. This appears to be
channels (and inhibition of potassium ion channels) a breathing-related sleep disorder, so you should
generate excitatory postsynaptic potentials (EPSPs). probably write a prescription for
The answer is E. (A) Clorazepate
3. (B) Diazepam
A decrease in K+ conductance is associated with (C) Flurazepam
neuronal excitation. With the exception of (D) Pentobarbital
dopamine, all of the neurotransmitters listed are (E) None of the above
able to cause excitation by this mechanism via their 2. Which statement concerning the barbiturates is
activation of specific receptors: acetylcholine (M1), accurate?
(A) Abstinence syndromes are more severe during (E) Triazolam does not cause rebound insomnia on
withdrawal from phenobarbital than from abrupt discontinuance
secobarbital 6. The most likely explanation for the increased
(B) Alkalinization of the urine accelerates the sensitivity of elderly patients after a single dose of a
elimination of phenobarbital benzodiazepine is
(C) Barbiturates may increase the half-lives of drugs (A) Age-dependent changes in brain function
metabolized by the liver (B) Decreases in plasma protein binding
(D) Compared with barbiturates, the (C) Decreased metabolism of lipid-soluble drugs
benzodiazepines exhibit a steeper dose-response (D) Decreases in renal function
relationship (E) Increased cerebral blood flow
(E) Respiratory depression caused by barbiturate 7. A 40-year-old woman has sporadic attacks of
overdosage can be reversed by flumazenil intense anxiety with marked physical symptoms,
3. A 24-year-old stockbroker has developed a including hyperventilation, tachycardia, and
“nervous disposition.” He is easily startled, worries sweating. If she is diagnosed as suffering from a
about inconsequential matters, and sometimes panic disorder, the most appropriate drug to use is
complains of stomach cramps. At night he grinds his (A) Alprazolam
teeth in his sleep. There is no history of drug abuse. (B) Eszopiclone
Diagnosed as suffering from generalized anxiety (C) Flurazepam
disorder, he is prescribed buspirone. The patient (D) Propranolol
should be informed to anticipate (E) Ramelteon
(A) A need to continually increase drug dosage 8. Which drug used in the maintenance treatment of
because of tolerance patients with tonic-clonic or partial seizure states
(B) A significant effect of the drug on memory increases the hepatic metabolism of many drugs
(C) Additive CNS depression with alcoholic beverages including both phenytoin and warfarin?
(D) That the drug is likely to take a week or more to (A) Buspirone
begin working (B) Clonazepam
(E) That if he stops taking the drug abruptly, he will (C) Eszopiclone
experience withdrawal signs (D) Phenobarbital
4. Which of the following best describes the (E) Triazolam
mechanism of action of benzodiazepines? 9. A patient with liver dysfunction is scheduled for a
(A) Activate GABAB receptors in the spinal cord surgical procedure. Lorazepam or oxazepam can be
(B) Block glutamate receptors in hierarchical used for preanesthetic sedation in this patient
neuronal pathways in the brain without special concern regarding excessive CNS
(C) Increase frequency of opening of chloride ion depression because these drugs are
channels coupled to GABAA receptors (A) Actively secreted in the renal proximal tubule
(D) Inhibit GABA transaminase to increase brain (B) Conjugated extrahepatically
levels of GABA (C) Eliminated via the lungs
(E) Stimulate release of GABA from nerve endings in (D) Reversible by administration of naloxone
the brain (E) Selective anxiolytics like buspirone
5. An 82-year-old woman, otherwise healthy for her
age, has difficulty sleeping. Triazolam is prescribed 10. This drug used in the management of insomnia
for her at one half of the conventional adult dose. facilitates the inhibitory actions of GABA, but it lacks
Which statement about the use of triazolam in this anticonvulsant or muscle-relaxing properties and has
elderly patient is accurate? minimal effect on sleep architecture. Its actions are
(A) Ambulatory dysfunction is unlikely to occur in antagonized by flumazenil.
elderly patients taking one half of the conventional (A) Buspirone
adult dose (B) Chlordiazepoxide
(B) Hypertension is a common adverse effect of (C) Eszopiclone
benzodiazepines in elderly patients (D) Ramelteon
(C) Over-the-counter cold medications may (E) Phenobarbital
antagonize the hypnotic effects of the drug ANSWERS
(D) The patient may experience amnesia, especially 1.
if she also consumes alcoholic beverages
Benzodiazepines and barbiturates are Decreased blood flow to vital organs, including the
contraindicated in breathing-related sleep disorders liver and kidney, occurs during the aging process.
because they further compromise ventilation. In These changes may contribute to cumulative effects
obstructive sleep apnea (pickwickian syndrome), of sedative-hypnotic drugs. However, this does not
obesity is a major risk factor. The best prescription explain the enhanced sensitivity of the elderly
you can give this patient is to lose weight. The patient to a single dose of a central depressant,
answer is E. which appears to be due to changes in brain function
2. that accompany aging. The answer is A.
Withdrawal symptoms from use of the shorter- 7.
acting barbiturate secobarbital are more severe than Alprazolam and clonazepam (not listed) are the most
with phenobarbital. The dose-response curve for effective of the benzodiazepines for the treatment
benzodiazepines is flatter than that for barbiturates. of panic disorders. Eszopiclone and flumazenil are
Induction of liver drug-metabolizing enzymes occurs hypnotics. Propranolol is commonly used to
with barbiturates and may lead to decreases in half- attenuate excessive sympathomimetic activity in
life of other drugs. Flumazenil is an antagonist at BZ persons who suffer from performance anxiety
receptors and is used to reverse CNS depressant (“stage fright”). The answer is A.
effects of benzodiazepines. As a weak acid (pKa 7), 8.
phenobarbital will be more ionized (nonprotonated) Clonazepam and phenobarbital are both used in
in the urine at alkaline pH and less reabsorbed in the seizure disorders. Chronic administration of
renal tubule. The answer is B. phenobarbital (but not clonazepam) increases the
3. activity of hepatic drug-metabolizing enzymes,
Buspirone is a selective anxiolytic with including several cytochrome P450 isozymes. This
pharmacologic characteristics different from those can increase the rate of metabolism of drugs
of sedative-hypnotics. Buspirone has minimal effects administered concomitantly, resulting in decreases
on cognition or memory; it is not additive with in the intensity and duration of their effects. The
ethanol in terms of CNS depression; tolerance is answer is D.
minimal; and it has no dependence liability. 9.
Buspirone is not effective in acute anxiety because it The elimination of most benzodiazepines involves
has a slow onset of action. The answer is D. their metabolism by liver enzymes, including
4. cytochrome P450 isozymes. In a patient with liver
Benzodiazepines exert most of their CNS effects by dysfunction, lorazepam and oxazepam, which are
increasing the inhibitory effects of GABA, interacting metabolized extrahepatically, are less likely to cause
with components of the GABAA receptor-chloride excessive CNS depression. Benzodiazepines are not
ion channel macromolecular complex to increase the eliminated via the kidneys or lungs. Flumazenil is
frequency of chloride ion channel opening. used to reverse excessive CNS depression caused by
Benzodiazepines do not affect GABA metabolism or benzodiazepines. The answer is B.
release, and they are not GABA receptor agonists 10.
because they do not interact directly with the Only two of the drugs listed are used for insomnia,
binding site for GABA. The answer is C. eszopiclone and ramelteon. Eszopiclone, zaleplon,
5. and zolpidem are related hypnotics that, though
In elderly patients taking benzodiazepines, structurally different from benzodiazepines, appear
hypotension is far more likely than an increase in to have a similar mechanism of action. However,
blood pressure. Elderly patients are more prone to unlike benzodiazepines, these drugs are not used in
the CNS depressant effects of hypnotics; a dose seizures or in muscle spasticity states. Compared
reduction of 50% may still cause excessive sedation with benzodiazepines, the newer hypnotics are less
with possible ambulatory impairment. Additive CNS likely to alter sleep patterns. Ramelteon activates
depression occurs commonly with drugs used in melatonin receptors in the suprachiasmatic nuclei.
over-the-counter cold medications, and rebound Buspirone is not a hypnotic! The answer is C.
insomnia can occur with abrupt discontinuance of QUESTIONS
benzodiazepines used as sleeping pills. Alcohol 1. A 45-year-old moderately obese man has been
enhances psychomotor depression and the amnestic drinking heavily for 72 h. This level of drinking is
effects of the benzodiazepines. The answer is D. much higher than his regular habit of drinking 1
6. alcoholic drink per day. His only significant medical
problem is mild hypertension, which is adequately (E) Methanol
controlled by metoprolol. With this history, this man 6. After assessing and stabilizing the patient’s
is at significant risk for airway, respiration, and circulatory status,
(A) Bacterial pneumonia fomepizole was administered intravenously. Which
(B) Cardiac arrhythmias of the following most accurately describes the
(C) Hyperthermia therapeutic purpose of the fomepizole
(D) Tonic-clonic seizures administration?
(E) Wernicke-Korsakoff syndrome (A) Accelerate the rate of elimination of the toxic
2. A 42-year-old man with a history of alcoholism is liquid that he consumed
brought to the emergency department in a confused (B) Combat acidosis
and delirious state. He has truncal ataxia and (C) Inhibit the metabolic production of toxic
ophthalmoplegia. The most appropriate immediate metabolites
course of action is to administer diazepam plus (D) Prevent alcohol withdrawal seizures
(A) Chlordiazepoxide (E) Sedate the patient
(B) Disulfiram 7. The regular ingestion of moderate or heavy
(C) Folic acid amounts of alcohol predisposes to hepatic damage
(D) Glucosamine after overdose of acetaminophen because chronic
(E) Thiamine ethanol ingestion
3. The cytochrome P450-dependent microsomal (A) Blocks acetaminophen metabolism
ethanol oxidizing system (MEOS) pathway of ethanol (B) Causes thiamine deficiency
metabolism is most likely to be maximally activated (C) Displaces acetaminophen from plasma proteins
under the condition of low concentrations of (D) Induces hepatic drug-metabolizing enzymes
(A) Acetaldehyde
(B) Ethanol 8. A 23-year-old pregnant woman with alcoholism
(C) NAD+ presented to the emergency department in the early
(D) NADPH stages of labor. She had consumed large amounts of
(E) Oxygen alcohol throughout her pregnancy. This patient’s
4. A freshman student (weight 70 kg) attends a infant is at high risk of a syndrome that includes
college party where he rapidly consumes a quantity (A) Ambiguous genitalia in a male fetus and normal
of an alcoholic beverage that results in a blood level genitalia in a female fetus
of 500 mg/dL. Assuming that this young man has not (B) Failure of closure of the atrial septum or
had an opportunity to develop tolerance to ethanol, ventricular septum
his present condition is best characterized as (C) Limb or digit malformation
(A) Able to walk, but not in a straight line (D) Mental retardation and craniofacial
(B) Alert and competent to drive a car abnormalities
(C) Comatose and near death (E) Underdevelopment of the lungs
(D) Sedated with increased reaction times 9. The combination of ethanol and disulfiram results
(E) Slightly inebriated in nausea and hypotension as a result of the
Questions 5 and 6. A homeless middle-aged male accumulation of
patient presents in the emergency department in a (A) Acetaldehyde
state of intoxication. You note that he is behaviorally (B) Acetate
disinhibited and rowdy. He tells you that he has (C) Methanol
recently consumed about a pint of a red-colored (D) NADH
liquid that his friends were using to “get high.” He (E) Pyruvate
complains that his vision is blurred and that it is “like 10. The intense craving experienced by those who
being in a snowstorm.” His breath smells a bit like are trying to recover from chronic alcohol abuse can
formaldehyde. He is acidotic. be ameliorated by a drug that is an
5. Which of the following is the most likely cause of (A) Agonist of α1 adrenoceptors
this patient’s intoxicated state? (B) Agonist of serotonin receptors
(A) Ethanol (C) Antagonist of β2 adrenoceptors
(B) Ethylene glycol (D) Antagonist of opioid receptors
(C) Isopropanol (E) Inhibitor of cyclooxygenase
(D) Hexane ANSWERS
1. Chronic use of ethanol induces a CYP2E1 isozyme
This man’s regular rate of alcohol consumption is not that converts acetaminophen to a cytotoxic
high enough to put him at risk of long-term metabolite. This appears to be the explanation for
consequences such as Wernicke-Korsakoff the increased susceptibility to acetaminophen-
syndrome, increased susceptibility to bacterial induced hepatotoxicity found in individuals who
pneumonia, or alcohol withdrawal seizures. This regularly ingest alcohol. The answer is D.
pattern of “binge drinking” does put him at 8.
increased risk of cardiac arrhythmia. The answer is B. This woman’s infant is at risk for fetal alcohol
2. syndrome, a syndrome associated with mental
This patient has symptoms of Wernicke’s retardation, abnormalities of the head and face, and
encephalopathy, including delirium, gait growth deficiency. This syndrome is a leading cause
disturbances, and paralysis of the external eye of mental retardation. The answer is D.
muscles. The condition results from thiamine 9.
deficiency but is rarely seen in the absence of The nausea, hypotension, and ill feeling that result
alcoholism. The diazepam is administered to prevent from drinking ethanol while also taking disulfiram
the alcohol withdrawal syndrome. Glucosamine is stems from acetaldehyde accumulation. Disulfiram
primarily used for pain associated with arthritis. The inhibits acetaldehyde dehydrogenase, the enzyme
answer is E. that converts acetaldehyde to acetate. The answer is
3. A.
The microsomal ethanol-oxidizing system (MEOS) 10.
contributes most to ethanol metabolism at relatively Naltrexone, a competitive inhibitor of opioid
high blood alcohol concentrations (>100 mg/dL), receptors, decreases the craving for alcohol in
when the alcohol dehydrogenase pathway is patients who are recovering from alcoholism. The
saturated due to depletion of NAD+. So, the MEOS answer is D.
system contributes most when the NAD+
concentration is low. NADPH and oxygen are QUESTIONS
cofactors for MEOS reactions. The concentration of 1. A 9-year-old child is having learning difficulties at
acetaldehyde does not appear to affect the rate of school. He has brief lapses of awareness with eyelid
either the ADH or the MEOS reactions. The answer is fluttering that occur every 5–10 min.
C. Electroencephalogram (EEG) studies reveal brief 3-
4. Hz spike and wave discharges appearing
The blood level of ethanol achieved in this individual synchronously in all leads. Which drug would be
is extremely high and likely to result in coma and effective in this child without the disadvantages of
possibly death due to respiratory arrest in a person excessive sedation or tolerance development?
who lacks tolerance to ethanol. The answer is C. (A) Clonazepam
5. (B) Diazepam
Behavioral disinhibition is a feature of early (C) Ethosuximide
intoxication from ethanol and most other alcohols (D) Gabapentin
but not the solvent, hexane. Ocular dysfunction, (E) Phenobarbital
including horizontal nystagmus and diplopia, is also a 2. Which statement concerning the proposed
common finding in poisoning with alcohols, but the mechanisms of action of anticonvulsant drugs is
complaint of “flickering white spots before the eyes” inaccurate?
or “being in a snowstorm” is highly suggestive of (A) Benzodiazepines facilitate GABA-mediated
methanol intoxication. In some cases, the odor of inhibitory actions
formaldehyde may be present on the breath. In this (B) Ethosuximide selectively blocks potassium ion
patient, blood methanol levels should be (K+) channels in thalamic neurons
determined as soon as possible. The answer is E. (C) Phenobarbital has multiple actions, including
6. enhancement of the effects of GABA, antagonism of
In patients with suspected methanol intoxication, glutamate receptors, and blockade of sodium ion
fomepizole is given intravenously to inhibit the ADH- (Na+) channels
catalyzed formation of toxic metabolites. The answer (D) Phenytoin prolongs the inactivated state of the
is C. Na+ channel
7. (E) Zonisamide blocks voltage-gated Na+ channels
3. Which drug used in management of seizure (B) Clonazepam
disorders is most likely to elevate the plasma (C) Ethosuximide
concentration of other drugs administered (D) Phenobarbital
concomitantly? (E) Phenytoin
(A) Carbamazepine 8. The mechanism of antiseizure activity of
(B) Clonazepam carbamazepine is
(C) Phenobarbital (A) Block of sodium ion channels
(D) Phenytoin (B) Block of calcium ion channels
(E) Valproic acid (C) Facilitation of GABA actions on chloride ion
4. A young female patient suffers from absence channels
seizures. Which of the following statements about (D) Glutamate receptor antagonism
her proposed drug management is NOT accurate? (E) Inhibition of GABA transaminase
(A) Ethosuximide and valproic acid are preferred 9. Which statement about phenytoin is accurate?
drugs (A) Displaces sulfonamides from plasma proteins
(B) Gastrointestinal side effects are common with (B) Drug of choice in myoclonic seizures
ethosuximide (C) Half-life is increased if used with phenobarbital
(C) The patient should be examined every 2 or 3 mo (D) Isoniazid (INH) decreases steady-state blood
for deep tendon reflex activity levels of phenytoin
(D) The use of valproic acid in pregnancy may cause (E) Toxic effects may occur with only small
congenital malformations increments in dose
(E) Weight gain is common in patients on valproic 10. A young male patient suffers from a seizure
acid disorder characterized by tonic rigidity of the
5. Which statement concerning the extremities followed in 15–30 s of tremor
pharmacokinetics of antiseizure drugs is accurate? progressing to massive jerking of the body. This
(A) Administration of phenytoin to patients in clonic phase lasts for 1 or 2 min, leaving the patient
methadone maintenance programs has led to in a stuporous state. Of the following drugs, which is
symptoms of opioid overdose, including respiratory most suitable for long-term management of this
depression patient?
(B) To reduce gastrointestinal toxicity, ethosuximide (A) Clonazepam
is usually taken twice a day (B) Ethosuximide
(C) At high doses, phenytoin elimination follows first- (C) Felbamate
order kinetics (D) Phenytoin
(D) The administration of phenytoin to patients in (E) Pregabalin
methadone maintenance programs has led to ANSWERS
symptoms of opioid overdose, including respiratory 1.
depression This child suffers from absence seizures, and 2 of the
(E) Treatment with vigabatrin reduces the drugs listed are effective in this seizure disorder.
effectiveness of oral contraceptives Clonazepam is effective but exerts troublesome CNS-
(F) Valproic acid may increase the activity of hepatic depressant effects, and tolerance develops with
ALA synthase and the synthesis of porphyrins chronic use. Ethosuximide is not excessively
6. With chronic use in seizure states, the adverse sedating, and tolerance does not develop to its
effects of this drug include coarsening of facial antiseizure activity. Valproic acid (not listed) is also
features, hirsutism, and gingival hyperplasia. used in absence seizures. The answer is C.
(A) Carbamazepine 2.
(B) Ethosuximide The mechanism of action of phenylsuccinimides such
(C) Phenytoin as ethosuximide involves blockade of T-type Ca2+
(D) Tiagabine channels in thalamic neurons. Ethosuximide does
(E) Zonisamide not block K+ channels, which in any case would be
7. Abrupt withdrawal of antiseizure drugs can result likely to result in an increase (rather than a
in increases in seizure frequency and severity. decrease) in neuronal excitability. The answer is B.
Withdrawal is most easily accomplished if the 3.
patient is treated with With chronic use, carbamazepine, phenobarbital,
(A) Carbamazepine and phenytoin can induce the synthesis of hepatic
drug-metabolizing enzymes. This action may lead to inhibitory actions of GABA; topiramate may block
a decrease in the plasma concentration of other glutamate receptors; and vigabatrin inhibits GABA
drugs used concomitantly. Valproic acid, an inhibitor metabolism. The answer is A.
of drug metabolism, can increase the plasma levels 9.
of many drugs, including those used in seizure Sulfonamides can displace phenytoin from its
disorders such as carbamazepine, lamotrigine, binding sites, increasing the plasma-free fraction of
phenobarbital, and phenytoin. Benzodiazepines the drug. Induction of liver drug-metabolizing
(including clonazepam and diazepam) as well as enzymes by phenobarbital results in a decreased
gabapentin and vigabatrin have no major effects on half-life of phenytoin, and isoniazid increases plasma
the metabolism of other drugs. The answer is E. levels of phenytoin by inhibiting its metabolism.
4. Because of the dose-dependent elimination kinetics
Ethosuximide and valproic acid are preferred drugs of phenytoin, some toxicity may occur with only
in absence seizures because they cause minimal small increments in dose. The answer is E.
sedation. However, valproic acid causes 10.
gastrointestinal distress and weight gain and is This patient is suffering from generalized tonic-clonic
potentially hepatotoxic. In addition, its use in seizures. For many years, the drugs of choice in this
pregnancy has been associated with teratogenicity seizure disorder have been carbamazepine or
(neural tube defects). Peripheral neuropathy, phenytoin or valproic acid. However, many newer
including diminished deep tendon reflexes in the drugs are also effective, including gabapentin,
lower extremities, occurs with the chronic use of lamotrigine, levetiracetem, topiramate, and
phenytoin, not valproic acid. The answer is C. zonisamide. Clonazepam and ethosuximide are not
5. effective in this type of seizure disorder. Pregabalin
The enzyme-inducing activity of phenytoin has led to is approved for use only in partial seizures. The
symptoms of opioid withdrawal, presumably answer is D.
because of an increase in the rate of metabolism of
methadone. Monitoring of plasma concentration of QUESTIONS
phenytoin may be critical is establishing and 1. A new halogenated gas anesthetic has a blood:gas
effective dosage because of nonlinear elimination partition coefficient of 0.5 and a MAC value of 1%.
kinetics at high doses. Valproic acid has no effect on Which prediction about this agent is most accurate?
porphyrin synthesis. Vigabatrin does not affect the (Refer to Table 25–1 for comparison of agents.)
metabolism of oral contraceptives. Twice-daily (A) Equilibrium between arterial and venous gas
dosage of ethosuximide reduces the severity of tension will be achieved very slowly
adverse gastrointestinal effects. The answer is B. (B) It will be metabolized by the liver to release
6. fluoride ions
Common adverse effects of phenytoin include (C) It will be more soluble in the blood than
nystagmus, diplopia, and ataxia. With chronic use, isoflurane
abnormalities of vitamin D metabolism, coarsening (D) Speed of onset will be similar to that of nitrous
of facial features, gingival overgrowth and hirsutism oxide
may also occur. A major adverse effect of tiagabine (E) The new agent will be more potent than
and zonisamide is CNS depression. The answer is C. halothane
7. 2. Which statement concerning the effects of
Dose tapering is an important principle in antiseizure anesthetic agents is false?
drug withdrawal. As a rule, withdrawal from drugs (A) Bronchiolar smooth muscle relaxation occurs
used for absence seizures such as ethosuximide is during halothane anesthesia
easier than withdrawal from drugs used for partial (B) Chest muscle rigidity often follows the
and tonic-clonic seizures. Withdrawal is most administration of fentanyl
difficult in patients who have been treated with (C) Mild, generalized muscle twitching occurs at high
barbiturates and benzodiazepines. The answer is C. doses of enflurane
8. (D) Severe hepatitis has been reported after the use
The mechanism of action of carbamazepine is similar of methoxyflurane
to that of phenytoin, blocking sodium ion channels. (E) The use of midazolam with inhalation anesthetics
Ethosuximide blocks calcium channels; may prolong postanesthesia recovery
benzodiazepines and barbiturates facilitate the
3. A 23-year-old man has a pheochromocytoma, rapidly developed tachycardia and became
blood pressure of 190/120 mm Hg, and hematocrit hypertensive. Generalized skeletal muscle rigidity
of 50%. Pulmonary function and renal function are was accompanied by marked hyperthermia.
normal. His catecholamines are elevated, and he has Laboratory values revealed hyperkalemia and
a well-defined abdominal tumor on MRI. He has acidosis.
been scheduled for surgery. Which one of the 7. This unusual complication of anesthesia is most
following agents should be avoided in the anesthesia likely to be caused by
protocol? (A) Acetylcholine release from somatic nerve
(A) Desflurane endings at skeletal muscle
(B) Fentanyl (B) Activation of brain dopamine receptors by
(C) Isoflurane halothane
(D) Midazolam (C) Antagonism of autonomic ganglia by
(E) Sevoflurane tubocurarine
4. Which statement concerning nitrous oxide is (D) Calcium released within skeletal muscle
accurate? (E) Toxic metabolites of nitrous oxide
(A) A useful component of anesthesia protocols 8. The patient should be treated immediately with
because it lacks cardiovascular depression (A) Atropine
(B) Anemia is a common adverse effect in patients (B) Baclofen
exposed to nitrous oxide for periods longer than 2 h (C) Dantrolene
(C) It is the most potent of the inhaled anesthetics (D) Edrophonium
(D) There is a direct association between the use of (E) Flumazenil
nitrous oxide and malignant hyperthermia 9. If ketamine is used as the sole anesthetic in the
(E) Up to 50% of nitrous oxide is eliminated via attempted reduction of a dislocated shoulder joint,
hepatic metabolism its actions will include
5. Which statement concerning anesthetic MAC (A) Analgesia
(minimum anesthetic concentration) value is (B) Bradycardia
accurate? (C) Hypotension
(A) Anesthetics with low MAC value have low (D) Muscle rigidity
potency (E) Respiratory depression
(B) MAC values increase in elderly patients 10. Postoperative vomiting is uncommon with this
(C) MAC values give information about the slope of intravenous agent, and patients are often able to
the dose–response curve ambulate sooner than those who receive other
(D) Methoxyflurane has an extremely low MAC value anesthetics.
(E) Simultaneous use of opioid analgesics increases (A) Enflurane
the MAC for inhaled anesthetics (B) Etomidate
6. Total intravenous anesthesia with fentanyl has (C) Midazolam
been selected for a frail elderly woman about to (D) Propofol
undergo cardiac surgery. Which statement about (E) Thiopental
this anesthesia protocol is accurate? ANSWERS
(A) Fentanyl will control the hypertensive response 1.
to surgical stimulation The partition coefficient of an inhaled anesthetic is a
(B) Marked relaxation of skeletal muscles is determinant of its kinetic characteristics. Agents
anticipated with low blood:gas solubility have a fast onset of
(C) Opioids such as fentanyl provide useful action and a short duration of recovery. The new
cardiostimulatory effects agent described here resembles nitrous oxide but is
(D) Patient awareness may occur during surgery, more potent, as indicated by its low MAC value. Not
with recall after recovery all halogenated anesthetics undergo significant
(E) The patient is likely to experience pain during hepatic metabolism or release fluoride ions. The
surgery answer is D.
Questions 7 and 8. A 20-year-old male patient 2.
scheduled for hernia surgery was anesthetized with Hepatitis after general anesthesia has been linked to
halothane and nitrous oxide; tubocurarine was use of halothane, although the incidence is very low
provided for skeletal muscle relaxation. The patient (1 in 20,00–35,000). Hepatotoxicity has not been
reported after administration of methoxyflurane or Predisposing genetic factors include clinical
other inhaled anesthetics. However, fluoride release myopathy associated with mutations in the gene loci
from prolonged use of methoxyflurane has caused for the skeletal muscle ryanodine receptor or L-type
renal insufficiency. The answer is D. calcium receptors. Nitrous oxide is not metabolized!
3. The answer is D.
Isoflurane sensitizes the myocardium to 8.
catecholamines, as does halothane (not listed). The drug of choice in malignant hyperthermia is
Arrhythmias may occur in patients with cardiac dantrolene, which prevents release of calcium from
disease who have high circulating levels of the sarcoplasmic reticulum of skeletal muscle cells.
epinephrine and norepinephrine (eg, patients with Appropriate measures must be taken to lower body
pheochromocytoma). Other newer inhaled temperature, control hypertension, and restore acid-
anesthetics are considerably less arrhythmogenic. base and electrolyte balance. The answer is C.
The answer is C. 9.
4. Ketamine is a cardiovascular stimulant, increasing
Anemia has not been reported in patients exposed heart rate and blood pressure. This results in part
to nitrous oxide anesthesia for periods as long as 6 h. from central sympathetic stimulation and from
Nitrous oxide is the least potent of the inhaled inhibition of norepinephrine reuptake at
anesthetics, and the compound has not been sympathetic nerve endings. Analgesia and amnesia
implicated in malignant hyperthermia. More than occur, with preservation of muscle tone and minimal
98% of the gas is eliminated via exhalation. The depression of respiration. The answer is A.
answer is A. 10.
5. Propofol is used extensively in anesthesia protocols,
MAC value is inversely related to potency; a low including those for day surgery. The favorable
MAC means high potency. MAC gives no information properties of the drug include an antiemetic effect
about the slope of the dose–response curve. Use of and recovery more rapid than that after use of other
opioid analgesics or other CNS depressants with intravenous drugs. Propofol does not cause
inhaled anesthetics lowers the MAC value. As with cumulative effects, possibly because of its short half-
most CNS depressants, the elderly patient is more life (2–8 min) in the body. The drug is also used for
sensitive, so MAC values are lower. Methoxyflurane prolonged sedation in critical care settings. The
has the lowest MAC value of the inhaled anesthetics. answer is D.
The answer is D. QUESTIONS
6. 1. Characteristic properties of local anesthetics
Intravenous opioids (eg, fentanyl) are widely used in include all of the following EXCEPT
anesthesia for cardiac surgery because they provide (A) An increase in membrane refractory period
full analgesia and cause less cardiac depression than (B) Blockade of voltage-dependent sodium channels
inhaled anesthetic agents. The opioids are not (C) Effects on vascular tone
cardiac stimulants, and fentanyl is more likely to (D) Preferential binding to resting channels
cause skeletal muscle rigidity than relaxation. (E) Slowing of axonal impulse conduction
Disadvantages of this technique are patient recall 2. The pKa of lidocaine is 7.7. In infected tissue,
(which can be decreased by concomitant use of a which can be acidic, for example, at pH 6.7, the
benzodiazepine) and the occurrence of hypertensive percentage of the drug in the nonionized form will
responses to surgical stimulation. The addition of be
vasodilators (eg, nitroprusside) or a β blocker (eg, (A) 1%
esmolol) may be needed to prevent intraoperative (B) 10%
hypertension. The answer is D. (C) 50%
7. (D) 90%
Malignant hyperthermia is a rare but life-threatening (E) 99%
reaction that may occur during general anesthesia 3. Which statement about the speed of onset of
with halogenated anesthetics and skeletal muscle nerve blockade with local anesthetics is correct?
relaxants, particularly succinylcholine and (A) Faster in hypercalcemia
tubocurarine. Release of calcium from skeletal (B) Faster in myelinated fibers
sarcoplasmic reticulum leads to muscle spasms, (C) Faster in tissues that are infected
hyperthermia and autonomic instability. (D) Slower in hyperkalemia
(E) Slower in the periphery of a nerve bundle than in (D) Mepivacaine
the center of a bundle (E) Tetracaine
4. The most important effect of inadvertent 10. Prilocaine is relatively contraindicated in patients
intravenous administration of a large dose of with cardiovascular or pulmonary disease because
lidocaine is the drug
(A) Bronchoconstriction (A) Acts as an agonist at β adrenoceptors in the
(B) Methemoglobinemia heart and the lung
(C) Renal failure (B) Causes decompensation through formation of
(D) Seizures methemo-
(E) Tachycardia globin
5. All of the following factors influence the action of (C) Inhibits cyclooxygenase in cardiac and pulmonary
local anesthetics EXCEPT cells
(A) Acetylcholinesterase activity in the region of the (D) Is a potent bronchoconstrictor
injection site (E) None of the above
(B) Blood flow through the tissue in which the ANSWERS
injection is made 1.
(C) Dose of local anesthetic injected Local anesthetics bind preferentially to sodium
(D) The use of vasoconstrictors channels in the open and inactivated states.
(E) Tissue pH Recovery from drug-induced block is 10–1000 times
6. You have a vial containing 10 mL of a 2% solution slower than recovery of channels from normal
of lidocaine. How much lidocaine is present in 1 mL? inactivation. Resting channels have a lower affinity
(A) 2 mg for local anesthetics. The answer is D.
(B) 5 mg 2.
(C) 10 mg Because the drug is a weak base, it is more ionized
(D) 20 mg (protonated) at pH values lower than its pKa.
(E) 50 mg Because the pH given is 1 log unit lower (more acid)
7. Which statement about the toxicity of local than the pKa, the ratio of ionized to nonionized drug
anesthetics is correct? will be approximately 90:10. The answer is B. (Recall
(A) Bupivacaine is the safest local anesthetic to use from Chapter 1 that at a pH equal to pKa, the ratio is
in patients at risk for cardiac arrhythmias. 1:1; at 1 log unit difference, the ratio is
(B) In overdosage, hyperventilation (with oxygen) is approximately 90:10; at 2 log units difference, 99:1;
helpful to correct acidosis and lower extracellular and so on.)
potassium 3.
(C) Intravenous injection of local anesthetics may Myelinated nerve fibers are blocked by local
stimulate ectopic cardiac pacemaker activity anesthetics more readily than unmyelinated ones.
(D) Most local anesthetics cause vasoconstriction See the Skill Keeper answer for an explanation of the
(E) Serious cardiovascular reactions are more likely effects of hypocalcemia and hyperkalemia on nerve
to occur with tetracaine than with bupivacaine blockade by local anesthetics. The answer is B.
8. A vasoconstrictor added to a solution of lidocaine 4.
for a peripheral nerve block will Of the effects listed, the most important in local
(A) Decrease the risk of a seizure anesthetic overdose (of both amide and ester types)
(B) Increase the duration of anesthetic action of the concern the CNS. Such effects can include sedation
local anesthetic or restlessness, nystagmus, coma, respiratory
(C) Both A and B depression, and seizures. Intravenous diazepam is
(D) Neither A nor B commonly used for seizures caused by local
9. A child requires multiple minor surgical anesthetics. Methemoglobinemia is caused by a
procedures involving the nasopharynx. Which drug prilocaine metabolite. The answer is D.
has high surface local anesthetic activity and intrinsic 5.
vasoconstrictor actions that reduce bleeding in Local anesthetics are poor substrates for
mucous membranes? acetylcholinesterase, and the activity of this enzyme
(A) Bupivacaine does not play a part in terminating the actions of
(B) Cocaine local anesthetics. Ester-type local anesthetics are
(C) Lidocaine hydrolyzed by plasma (and tissue) pseudo-
cholinesterases. Persons with genetically based (C) Muscle fasciculations only in the later stages of
defects in pseudocholinesterase activity are block
unusually sensitive to procaine and other esters. The (D) Reversibility by acetylcholinesterase (AChE)
answer is A. inhibitors
6. (E) Sustained tension during a period of tetanic
The fact that you have 10 mL of the solution of stimulation
lidocaine is irrelevant. A 2% solution of any drug Questions 2 and 3. A patient underwent a surgical
contains 2 g/100 mL. The amount of lidocaine in 1 procedure of 2 h. Anesthesia was provided by
mL of a 2% solution is thus 0.02 g, or 20 mg. The isoflurane, supplemented by intravenous midazolam
answer is D. and a nondepolarizing muscle relaxant. At the end of
7. the procedure, a low dose of atropine was
Acidosis resulting from tissue hypoxia favors local administered followed by pyridostigmine.
anesthetic toxicity because these drugs bind more 2. The main reason for administering atropine was to
avidly (or dissociate more slowly) from the sodium (A) Block cardiac muscarinic receptors
channel binding site when they are in the charged (B) Enhance the action of pyridostigmine
state. (Note that onset of therapeutic effect may be (C) Prevent spasm of gastrointestinal smooth muscle
slower because charged local anesthetics penetrate (D) Provide postoperative analgesia
the membrane less rapidly; see text.) Hyperkalemia (E) Reverse the effects of the muscle relaxant
depolarizes the membrane, which also favors local 3. A muscarinic receptor antagonist would probably
anesthetic binding. Oxygenation reduces both not be needed for reversal of the skeletal muscle
acidosis and hyperkalemia. Bupivacaine may cause relaxant actions of a nondepolarizing drug if the
severe cardiotoxicity including arrhythmias. The agent used was
answer is B. (A) Cisatracurium
8. (B) Mivacurium
Epinephrine increases the duration of a nerve block (C) Pancuronium
when it is administered with short- and medium- (D) Tubocurarine
duration local anesthetics. As a result of the (E) Vecuronium
vasoconstriction that prolongs the duration of this 4. Which of the following drugs is the most effective
block, less local anesthetic is required, so the risk of in the emergency management of malignant
toxicity (eg, a seizure) is reduced. The answer is C. hyperthermia?
9. (A) Atropine
Cocaine is the only local anesthetic with intrinsic (B) Dantrolene
vasoconstrictor activity owing to its action to block (C) Haloperidol
the reuptake of norepinephrine released from (D) Succinylcholine
sympathetic nerve endings (Chapter 9). Cocaine also (E) Vecuronium
has significant surface local anesthetic activity and is 5. The clinical use of succinylcholine, especially in
favored for head, neck, and pharyngeal surgery. The patients with diabetes, is associated with
answer is B. (A) Antagonism by pyridostigmine during the early
10. phase of blockade
Large doses of prilocaine may cause accumulation of (B) Aspiration of gastric contents
o-toluidine, a metabolite that converts hemoglobin (C) Decreased intragastric pressure
to methemoglobin. Patients may become cyanotic (D) Histamine release in a genetically determined
with blood “chocolate colored.” High blood levels of population
methemoglobin have resulted in decompensation in (E) Metabolism at the neuromuscular junction by
patients who have cardiac or pulmonary diseases. acetylcholinesterase
The answer is B. 6. Which drug (related to clonidine) is most often
QUESTIONS associated with hypotension?
1. Characteristics of phase I depolarizing (A) Baclofen
neuromuscular blockade due to succinylcholine (B) Pancuronium
include (C) Succinylcholine
(A) Easy reversibility with nicotinic receptor (D) Tizanidine
antagonists (E) Vecuronium
(B) Marked muscarinic blockade
7. Regarding the spasmolytic drugs, which of the including bradycardia, a muscarinic blocking agent is
following statements is not accurate? used concomitantly. Although atropine is effective,
(A) Baclofen acts on GABA receptors in the spinal glycopyrollate is usually preferred because it lacks
cord to increase chloride ion conductance CNS effects. The answer is A.
(B) Cyclobenzaprine decreases both oropharyngeal 3.
secretions and gut motility One of the distinctive characteristics of pancuronium
(C) Dantrolene has no significant effect on the is that it can block muscarinic receptors, especially
release of calcium from cardiac muscle those in the heart. It has sometimes caused
(D) Diazepam causes sedation at doses commonly tachycardia and hypertension and may cause
used to reduce muscle spasms dysrhythmias in predisposed individuals. The answer
(E) Intrathecal use of baclofen is effective in some is C.
refractory cases of muscle spasticity 4.
8. Which drug is most likely to cause hyperkalemia Prompt treatment is essential in malignant
leading to cardiac arrest in patients with spinal cord hyperthermia to control body temperature, correct
injuries? acidosis, and prevent calcium release. Dantrolene
(A) Baclofen interacts with the RyR1 channel to block the release
(B) Dantrolene of activator calcium from the sarcoplasmic
(C) Pancuronium reticulum, which prevents the tension-generating
(D) Succinylcholine interaction of actin with myosin. The answer is B.
(E) Vecuronium 5.
9. Which drug has spasmolytic activity and could also Fasciculations associated with succinylcholine may
be used in the management of seizures caused by increase intragastric pressure with possible
overdose of a local anesthetic? complications of regurgitation and aspiration of
(A) Baclofen gastric contents. The complication is more likely in
(B) Cyclobenzaprine patients with delayed gastric emptying such as those
(C) Diazepam with esophageal dysfunction or diabetes. Histamine
(D) Gabapentin release resulting from succinylcholine is not
(E) Tizanidine genetically determined. The answer is B.
10. Myalgias are a common postoperative complaint 6.
of patients who receive large doses of Tizanidine causes hypotension via α2-adrenoceptor
succinylcholine, possibly the result of muscle activation, like its congener clonidine. Hypotension
fasciculations caused by depolarization. Which drug may occur with tubocurarine (not listed) due partly
administered in the operating room can be used to to histamine release and to ganglionic blockade. The
prevent postoperative pain caused by answer is D.
succinylcholine? 7.
(A) Atracurium Baclofen activates GABAB receptors in the spinal
(B) Baclofen cord. However, these receptors are coupled to K+
(C) Dantrolene channels (see Chapter 21). GABAA receptors in the
(D) Diazepam CNS modulate chloride ion channels, an action
(E) Lidocaine facilitated by diazepam and other benzodiazepines.
ANSWERS The answer is A.
1. 8.
Phase I depolarizing blockade caused by Skeletal muscle depolarization by succinylcholine
succinylcholine is not associated with antagonism at releases potassium from the cells, and the ensuing
muscarinic receptors, nor is it reversible with hyperkalemia can be life-threatening in terms of
cholinesterase inhibitors. Muscle fasciculations occur cardiac arrest. Patients most susceptible include
at the start of the action of succinylcholine. The those with extensive burns, spinal cord injuries,
answer is E. neurologic dysfunction, or intra-abdominal infection.
2. The answer is D.
Acetylcholinesterase inhibitors used for reversing 9.
the effects of nondepolarizing muscle relaxants Diazepam is both an effective antiseizure drug and a
cause increases in ACh at all sites where it acts as a spasmolytic. The spasmolytic action of diazepam is
neurotransmitter. To offset the resulting side effects, thought to be exerted partly in the spinal cord
because it reduces spasm of skeletal muscle in limbs. Which of the following statements about
patients with cord transection. Cyclobenzaprine is these symptoms is accurate?
used for acute local spasm and has no antiseizure (A) Coadministration of muscarinic blockers prevents
activity. The answer is C. the occurrence of dyskinesias during treatment with
10. levodopa
The depolarizing action of succinylcholine at the (B) Drugs that activate dopamine receptors can
skeletal muscle end plate can be antagonized by exacerbate dyskinesias in a patient taking levodopa
small doses of nondepolarizing blockers. To prevent (C) Dyskinesias are less likely to occur if levodopa is
skeletal muscle fasciculations and the resulting administered with carbidopa
postoperative pain caused by succinylcholine, a (D) Symptoms are likely to be alleviated by
small nonparalyzing dose of a nondepolarizing drug continued treatment with levodopa
(eg, atracurium) is often given immediately before (E) The symptoms are usually reduced if the dose of
succinylcholine. The answer is A. levodopa is increased
QUESTIONS 5. A 51-year-old patient with parkinsonism is being
Questions 1 and 2. Bradykinesia has made drug maintained on levodopa-carbidopa with adjunctive
treatment necessary in a 60-year-old male patient use of low doses of tolcapone but continues to have
with Parkinson’s disease, and therapy is to be off-periods of alkinesia. The most appropriate drug
initiated with levodopa. to “rescue” the patient but that will only provide
1. Regarding the anticipated actions of levodopa, the temporary relief is
patient would not be informed that (A) Apomorphine
(A) Dizziness may occur, especially when standing (B) Benztropine
(B) He should take the drug in divided doses to avoid (C) Carbidopa
nausea (D) Pramipexole
(C) Livedo reticularis is a possible side effect (E) Selegiline
(D) The drug will probably improve his symptoms for 6. Concerning the drugs used in parkinsonism, which
a period of time but not indefinitely statement is accurate?
(E) Uncontrollable muscle jerks may occur (A) Dopamine receptor agonists should never be
2. The prescribing physician will (or should) know used in Parkinson’s disease before a trial of levodopa
that levodopa (B) Levodopa causes mydriasis and may precipitate
(A) Causes fewer CNS side effects if given together an acute attack of glaucoma
with a drug that inhibits hepatic dopa decarboxylase (C) Selegiline is a selective inhibitor of COMT
(B) Fluctuates in its effectiveness with increasing (D) The primary benefit of antimuscarinic drugs in
frequency as treatment continues parkinsonism is their ability to relieve bradykinesia
(C) Prevents extrapyramidal adverse effects of (E) Therapeutic effects of amantadine continue for
antipsychotic drugs several years
(D) Protects against cancer in patients with 7. A previously healthy 40-year-old woman begins to
melanoma suffer from slowed mentation, lack of coordination,
(E) Has toxic effects, which include pulmonary and brief writhing movements of her hands that are
infiltrates not rhythmic. In addition, she has delusions of being
3. Which statement about pramipexole is accurate? persecuted. The woman has no history of psychiatric
(A) Effectiveness in Parkinson’s disease requires its or neurologic disorders. Although further diagnostic
metabolic conversion to an active metabolite assessment should be made, it is very likely that the
(B) It should not be administered to patients taking most appropriate drug for treatment will be
antimuscarinic drugs (A) Amantadine
(C) Pramipexole causes less mental disturbances (B) Bromocriptine
than levodopa (C) Diazepam
(D) The drug selectively activates the dopamine D3 (D) Haloperidol
receptor subtype (E) Levodopa
(E) Warfarin may enhance the actions of
pramipexole ANSWERS
4. A patient with parkinsonism is being treated with 1.
levodopa. He suffers from irregular, involuntary In prescribing levodopa, the patient should be
muscle jerks that affect the proximal muscles of the informed about adverse effects, including
gastrointestinal distress, postural hypotension, and closure glaucoma. Antimuscarinic drugs may
dyskinesias. It is reasonable to advise the patient improve the tremor and rigidity of parkinsonism but
that therapeutic benefits cannot be expected to have little effect on bradykinesia. Selegiline is a
continue indefinitely. Livedo reticularis (a netlike selective inhibitor of MAO type B. Amantadine is
rash) is an adverse effect of treatment with effective for only a few weeks. The answer is B.
amantadine. The answer is C. 7.
2. Although further diagnosis is desirable,
Levodopa causes less peripheral toxicity but more choreoathetosis with decreased mental abilities and
CNS or behavioral side effects when its conversion to psychosis (paranoia) suggests that this patient has
dopamine is inhibited outside the CNS. The drug is the symptoms of Huntington’s disease. Drugs that
not effective in antagonizing the akinesia, rigidity, are partly ameliorative include agents that deplete
and tremor caused by treatment with antipsychotic dopamine (eg, tetrabenazine) or that block
agents. Levodopa is a precursor of melanin and may dopaminergic receptors (eg, haloperidol). The
activate malignant melanoma. Use of levodopa is answer is D.
not associated with pulmonary dysfunction. The 8.
answer is B. Pramipexole is a non-ergot agonist at dopamine
3. receptors and has greater selectivity for D3
Pramipexole is a dopamine D3 receptor activator receptors in the striatum. Pramipexole (or the D2
and does not require bioactivation. It is excreted in receptor antagonist ropinirole) is often chosen for
unchanged form. Confusion, delusions, and monotherapy of mild parkinsonism, and these drugs
hallucinations occur more frequently with dopamine sometimes have value in patients who have become
receptor activators than with levodopa. The use of refractory to levodopa. Adverse effects of these
dopaminergic agents in combination with drugs include dyskinesias, postural hypotension, and
antimuscarinic drugs is common in the treatment of somnolence. The answer is E.
parkinsonism. Warfarin may enhance the action of 9.
ropinirole, another dopamine receptor agonist. The Tolcapone and entacapone are inhibitors of COMT
answer is D. used adjunctively in patients treated with levodopa-
4. carbidopa. The drugs decrease the formation of 3-O-
The form and severity of dyskinesias resulting from methyldopa (3-OMD) from levodopa. This improves
levodopa may vary widely in individual patients. patient response by increasing levodopa levels and
Dyskinesias occur in up to 80% of patients receiving by decreasing competition between 3-OMD and
levodopa for long periods. With continued levodopa for active transport into the brain by l-
treatment, dyskinesias may develop at a dose of amino acid carrier mechanism. The answer is B.
levodopa that was previously well tolerated. 10.
Muscarinic receptor blockers do not prevent their Increased activation of β adrenoceptors has been
occurrence. They occur more commonly in patients implicated in essential tremor, and management
treated with levodopa in combination with commonly involves administration of propranolol.
carbidopa or with other dopamine receptor agonists. However, the more selective β1 blocker metoprolol
The answer is B. is equally effective and is more suitable in a patient
5. with pulmonary disease. The answer is C.
Apomorphine, via subcutaneous injection, is used for
temporary relief of off-periods of akinesia (rescue) in QUESTIONS
parkinsonian patients on dopaminergic drug 1. Which statement about the pathophysiologic
therapy. Pretreatment with the antiemetic basis of schizophrenia is most accurate?
trimethobenzamide for 3 days is essential to prevent (A) All clinically effective antipsychotic drugs have
severe nausea. The answer is A. high affinity for dopamine D2 receptors
6. (B) Dopamine receptor-blocking drugs are used to
The non-ergot dopamine agonists (pramipexole, alleviate psychotic symptoms in parkinsonism
ropinirole) are commonly used prior to levodopa in (C) Drug-induced psychosis can occur without
mild parkinsonism. The mydriatic action of levodopa activation of brain dopamine receptors
may increase intraocular pressure; the drug should (D) Serotonin receptors are present at lower than
be used cautiously in patients with open-angle normal levels in the brains of untreated
glaucoma and is contraindicated in those with angle- schizophrenics
(E) The clinical potency of olanzapine correlates well (B) Clozapine
with its dopamine receptor-blocking activity (C) Lithium
2. Trifluoperazine was prescribed for a young male (D) Risperidone
patient diagnosed as suffering from schizophrenia. (E) Valproic acid
He complains about the side effects of his 7. A young male patient recently diagnosed as
medication. Which of the following is not likely to be schizophrenic develops severe muscle cramps with
on his list? torticollis a short time after drug therapy is initiated
(A) Constipation with haloperidol. The best course of action would be
(B) Decreased libido to
(C) Excessive salivation (A) Add risperidone to the drug regimen
(D) Postural hypotension (B) Discontinue haloperidol and observe the patient
3. Which statement concerning the adverse effects (C) Give oral diphenhydramine
of antipsychotic drugs is accurate? (D) Inject benztropine
(A) Acute dystonic reactions occur commonly with (E) Switch the patient to fluphenazine
olanzapine 8. Which of the following drugs is established to be
(B) Akathisias due to antipsychotic drugs are both effective and safe to use in a pregnant patient
managed by increasing the drug dose suffering from bipolar disorder?
(C) Blurring of vision and urinary retention are (A) Carbamazepine
common adverse effects of haloperidol (B) Fluphenazine
(D) Retinal pigmentation is a dose-dependent toxic (C) Lithium
effect of thioridazine (D) Olanzapine
(E) The late-occurring choreoathetoid movements (E) Valproic acid
caused by conventional antipsychotic drugs are 9. In comparing the characteristics of thioridazine
alleviated by atropine with other older antipsychotic drugs, which of the
4. Haloperidol is not an appropriate drug for following statements is accurate?
management of (A) Most likely to cause extrapyramidal dysfunction
(A) Acute mania (B) Least likely to cause urinary retention
(B) Amenorrhea-galactorrhea syndrome (C) Most likely to be safe in patients with history of
(C) Phencyclidine intoxication cardiac arrhythmias
(D) Schizoaffective disorders (D) Most likely to cause ocular dysfunction
(E) Tourette’s syndrome (E) The safest antipsychotic drug in overdose
5. Which statement concerning the use of lithium in 10. Which of the following drugs has a high affinity
the treatment of bipolar affective disorder is for 5-HT2 receptors in the brain, does not cause
accurate? extrapyramidal dysfunction or hematotoxicity, but is
(A) Ingestion of foods with high salt content reported to increase the risk of significant QT
enhances the toxicity of lithium prolongation?
(B) Lithium usually alleviates the manic phase of (A) Clozapine
bipolar disorder within 12 h (B) Haloperidol
(C) Lithium dosage may need to be decreased in (C) Olanzapine
patients taking thiazides (D) Valproic acid
(D) Since lithium does not cross the placental barrier, (E) Ziprasidone
it is safe in pregnancy
(E) The elimination rate of lithium is equivalent to ANSWERS
that of creatinine 1.
6. A 30-year-old male patient is on drug therapy for a Although most older antipsychotic drugs block D2
psychiatric problem. He complains that he feels receptors, this action is not a requirement for
“flat” and that he gets confused at times. He has antipsychotic action. Aripiprazole, clozapine, and
been gaining weight and has lost his sex drive. As he most newer second-generation drugs have a very
moves his hands, you notice a slight tremor. He tells low affinity for such receptors, but a high affinity for
you that since he has been on medication he is serotonin 5-HT2 receptors. There are no reports of
always thirsty and frequently has to urinate. The decreased serotonin receptors in the brains of
drug he is most likely to be taking is schizophrenics. The CNS effects of phencyclidine
(A) Carbamazepine (PCP) closely parallel an acute schizophrenic episode,
but PCP has no actions on brain dopamine receptors. of lithium that may occur at blood levels within the
Dopamine receptor blockers cause extrapyramidal therapeutic range. The answer is C.
dysfunction. The answer is C. 7.
2. Acute dystonic reactions are usually very painful and
Phenothiazines such as trifluoperazine cause should be treated immediately with parenteral
sedation and are antagonists at muscarinic and α administration of a drug that blocks muscarinic
adrenoceptors. Postural hypotension, blurring of receptors such as benztropine. Adding risperidone is
vision, and dry mouth are common autonomic not protective, and fluphenazine is as likely as
adverse effects, as is constipation. Effects on the haloperidol to cause acute dystonia. Oral
male libido may result from increased prolactin or administration of diphenhydramine is a possibility,
from increased peripheral conversion of androgens but the patient may find it difficult to swallow and it
to estrogens. The answer is C. would take a longer time to act. The answer is D.
3. 8.
Olanzapine has minimal dopamine receptor– Carbamazepine and valproic acid are effective in
blocking action and is unlikely to cause acute bipolar disorder but are contraindicated in the
dystonias. Muscarinic blockers such as atropine pregnant patient because of possible effects on fetal
exacerbate tardive dyskinesias. Akathisias development. Although the potential for
(uncontrollable restlessness) resulting from dysmorphogenesis due to lithium is probably low,
antipsychotic drugs may be relieved by a reduction in the most conservative approach would be to treat
dosage. Retinal pigmentation may occur from the patient with quetiapine or olanzapine.
treatment with thioridazine. The answer is D. Fluphenazine has no proven efficacy in bipolar
4. disorder. The answer is D.
In addition to its use in schizophrenia and acute 9.
mania, haloperidol has been used in the Atropine-like side effects are more prominent with
management of intoxication due to phencyclidine thioridazine than with other phenothiazines, but the
(PCP) and in Tourette’s syndrome. drug is less likely to cause extrapyramidal
Hyperprolactinemia and the amenorrhea- dysfunction. The drug has quinidine-like actions on
galactorrhea syndrome may occur as adverse effects the heart and, in overdose, may cause arrhythmias
during treatment with antipsychotic drugs, especially and cardiac conduction block with fatality. At high
those like haloperidol that strongly antagonize doses, thioridazine causes retinal deposits, which in
dopamine receptors in the tuberoinfundibular tract. advanced cases resemble retinitis pigmentosa. The
The answer is B. patient may complain of browning of vision. The
5. answer is D.
Clinical effects of lithium are slow in onset and may
not be apparent before 1 or 2 weeks of daily 10.
treatment. High urinary levels of sodium inhibit renal Many of the newer antipsychotic drugs have a
tubular reabsorption of lithium, thus decreasing its greater affinity for 5-HT2 receptors than dopamine
plasma levels. Lithium clearance is decreased by receptors. However, because clozapine is
distal tubule diuretics (eg, thiazides) because hematotoxic, the choice comes down to olanzapine
natriuresis stimulates a reflex increase in the and ziprasidone, both of which block 5-HT receptors.
proximal tubule reabsorption of both lithium and Of the currently available atypical antipsychotic
sodium. Any drug that can cross the blood-brain drugs, ziprasidone carries the greatest risk of QT
barrier can cross the placental barrier! Teratogenic prolongation. The answer is E.
risk is low, but use of lithium during pregnancy may
contribute to low Apgar score in the neonate. The QUESTIONS
answer is C. 1. A 36-year-old woman presents with symptoms of
6. major depression that are unrelated to a general
Confusion, mood changes, decreased sexual interest, medical condition, bereavement, or substance
and weight gain are symptoms that may be abuse. She is not currently taking any prescription or
unrelated to drug administration. On the other hand, over-the-counter medications. Drug treatment is to
psychiatric drugs are often responsible for such be initiated with sertraline. In your information to
symptoms. Tremor and symptoms of nephrogenic the patient, you would tell her that
diabetes insipidus are characteristic adverse effects
(A) Sertraline may take 2 wk or more to become (A) Administer bicarbonate and potassium chloride
effective (to correct acidosis and hypokalemia)
(B) It is preferable that she take the drug in the (B) Administer lidocaine (to control cardiac
morning arrhythmias)
(C) Muscle cramps and twitches can occur (C) Initiate hemodialysis (to hasten drug elimination)
(D) She should notify you if she anticipates using (D) Maintain heart rhythm by electrical pacing
other prescription drugs (E) Use intravenous diazepam to control seizures
(E) All of the above 6. Which drug is an antagonist at 5-HT2 receptors
2. Concerning the proposed mechanisms of action of and widely used for the management of insomnia?
antidepressant drugs, which statement is accurate? (A) Estazolam
(A) Bupropion inhibits NE reuptake into nerve (B) Flurazepam
endings in the CNS (C) Trazodone
(B) Chronic treatment with tricyclic antidepressants (D) Triazolam
leads to downregulation of adrenoceptors in the CNS (E) Zolpidem
(C) Decreased levels of NE and 5-HT in cerebrospinal 7. A recently widowed 76-year-old female patient
fluid is a characteristic of depressed patients before was treated with a benzodiazepine for several weeks
drug therapy after the death of her husband, but she did not like
(D) Nefazodone activates 5-HT receptors in the CNS the daytime sedation it caused even at low dosage.
(E) Selegiline selectively decreases the metabolism Living independently, she has no major medical
of serotonin problems but appears rather infirm for her age and
3. A 34-year-old male patient who was prescribed has poor eyesight. Because her depressive
citalopram for depression has decided he wants to symptoms are not abating, you decide on a trial of
stop taking the drug. When questioned, he said that an antidepressant medication. Which of the
it was affecting his sexual performance. You following drugs would be the most appropriate
ascertain that he is also trying to overcome his choice for this patient?
dependency on tobacco products. If you decide to (A) Amitriptyline
reinstitute drug therapy in this patient, the best (B) Citalopram
choice would be (C) Mirtazapine
(A) Amitriptyline (D) Phenelzine
(B) Bupropion (E) Trazodone
(C) Fluoxetine 8. SSRIs are much less effective than tricyclic
(D) Imipramine antidepressants in the management of
(E) Venlafaxine (A) Bulimia
4. Regarding the clinical use of antidepressant drugs, (B) Chronic pain of neuropathic origin
which statement is accurate? (C) Generalized anxiety disorder
(A) Chronic use of serotonin-norepinephrine (D) Obsessive-compulsive disorder
reuptake inhibitors (SNRIs) increases the activity of (E) Premenstrual dysphoric disorder
hepatic drug-metabolizing enzymes 9. Which of the following drugs is most likely to be of
(B) In the treatment of major depressive disorders, value in obsessive-compulsive disorders?
citalopram is usually more effective than paroxetine (A) Amitriptyline
(C) Tricyclics are highly effective in depressions with (B) Bupropion
attendant anxiety, phobic features, and (C) Clomipramine
hypochondriasis (D) Trazodone
(D) Weight gain often occurs during the first few (E) Venlafaxine
months in patients taking SSRIs 10. To be effective in breast cancer, tamoxifen must
(E) When selecting an appropriate drug for be converted to an active form by CYP2D6. Cases of
treatment of depression, the history of patient inadequate treatment of breast cancer have
response to specific drugs is a valuable guide occurred when tamoxifen was administered to
5. A patient under treatment for a major depressive patients who were being treated with
disorder is brought to the emergency department (A) Amitriptyline
after ingesting 30 times the normal daily therapeutic (B) Bupropion
dose of imipramine. Which of the following would be (C) Fluoxetine
least useful? (D) Mirtazapine
(E) Phenelzine the least effect on cardiac conductivity (eg,
ANSWERS lidocaine). Hemodialysis does not increase the rate
1. of elimination of tricyclic antidepressants in
All the statements are appropriate regarding the overdose. The answer is C.
initiation of treatment with sertraline or other SSRI 6.
in a depressed patient. The SSRIs have CNS- All of the drugs listed are effective hypnotic drugs,
stimulating effects and may cause agitation, anxiety, but only trazodone is an antagonist at 5-HT2
“the jitters,” and insomnia, especially early in receptors. Trazodone has wide use as a sleeping aid,
treatment. Consequently, the evening is not the best especially in patients with symptoms of affective
time to take SSRI drugs. The answer is E. disorder. The answer is C.
2. 7.
The mechanism of action of bupropion is unknown, Older patients are more likely to be sensitive to
but the drug does not inhibit either NE or 5-HT antidepressant drugs that cause sedation, atropine-
transporters. Levels of NE and 5-HT metabolites in like adverse effects, or postural hypotension.
the cerebrospinal fluid of depressed patients before Tricyclics and MAO inhibitors cause many autonomic
drug treatment are not higher than normal. side effects; mirtazapine and trazodone are highly
Selegiline is a selective inhibitor of MAO-B, the sedating. Citalopram (or another SSRI) is often the
enzyme form that metabolizes dopamine (see best choice in such patients. The answer is B.
Chapter 28). Nefazodone is a highly selective 8.
antagonist at the 5-HT2 receptor subtype. The SSRIs are not effective in chronic pain of
Downregulation of adrenoceptors appears to be a neuropathic origin. All the other uses of SSRIs are
common feature of chronic treatment of depression approved indications with clinical effectiveness
with tricyclic drugs such as amitriptyline. The answer equivalent or superior to that of tricyclic drugs. In
is B. addition to treatment of chronic pain states and
3. depression the tricyclics are also used to treat
The SSRIs (eg, fluoxetine) and venlafaxine (an SNRI) enuresis and attention deficit hyperkinetic disorder.
can cause sexual dysfunction with decreased libido, The answer is B.
erectile dysfunction, and anorgasmia. TCAs may also 9.
decrease libido or prevent ejaculation. Bupropion is Clomipramine, a tricyclic agent, is a more selective
the least likely antidepressant to affect sexual inhibitor of 5-HT reuptake than other drugs in its
performance. The drug is also purportedly useful in class. This activity appears to be important in the
withdrawal from nicotine dependence, which could treatment of obsessive-compulsive disorder.
be helpful in this patient. The answer is B. However, the SSRIs have now become the drugs of
4. choice for this disorder because they are safer in
No antidepressant has been shown to increase overdose than tricyclics. The answer is C.
hepatic drug metabolism. MAO inhibitors (not TCAs), 10.
though now used infrequently, are the drugs most Fluoxetine is an inhibitor of hepatic cytochrome
likely to be effective in depression with attendant P450s especially CYP2D6, and to a lesser extent
anxiety, phobic features, and hypochondriasis. SSRIs CYP3A4. Dosages of several drugs may need to be
are usually associated with weight loss, at least reduced if given concomitantly with fluoxetine. In
during the first 6 months of treatment. There is no the case of tamoxifen, however, its antineoplastic
evidence that any SSRI is more effective than action is dependent on its conversion to an active
another, or more effective overall than a tricyclic metabolite by CYP2D6. The answer is C.
drug, in treatment of major depressive disorder. The QUESTIONS
answer is E. Questions 1 and 2. A 63-year-old man is undergoing
5. radiation treatment as an outpatient for metastatic
Overdose with imipramine or any other tricyclic bone cancer. His pain has been treated with a fixed
antidepressant drug is a medical emergency. The “3 combination of oxycodone plus acetaminophen
Cs”—coma, convulsions, and cardiac problems—are taken orally. Despite increasing doses of the
the most common causes of death. Widening of the analgesic combination, the pain is getting worse.
QRS complex on the ECG is a major diagnostic 1. The most appropriate oral medication for his
feature of cardiac toxicity. Arrhythmias resulting increasing pain is
from cardiac toxicity require the use of drugs with (A) Buprenorphine
(B) Codeine plus aspirin and hyperthermia. The attending physician notes
(C) Hydromorphone that his pupil size is larger than normal.
(D) Pentazocine 6. What is the most likely cause of these signs and
(E) Tramadol symptoms?
2. It is possible that this patient will have to increase (A) The patient had injected dextroamphetamine
the dose of the analgesic as his condition progresses (B) The patient has hepatitis B
as a result of developing tolerance. However, (C) The patient has overdosed with an opioid
tolerance will not develop to a significant extent (D) The signs and symptoms are those of the opioid
with respect to abstinence syndrome
(A) Biliary smooth muscle (E) These are early signs of toxicity due to
(B) Emesis contaminants in “street heroin”
(C) Pupillary constriction 7. Which drug will be most effective in alleviating the
(D) Sedation symptoms experienced by this patient?
(E) Urinary retention (A) Buprenorphine
3. You are on your way to take an examination and (B) Codeine
you suddenly get an attack of diarrhea. If you stop at (C) Methadone
a nearby drugstore for an over-the-counter opioid (D) Naltrexone
with antidiarrheal action, you will be asking for (E) Tramadol
(A) Codeine 8. Which statement about nalbuphine is accurate?
(B) Dextromethorphan (A) Activates μ receptors
(C) Diphenoxylate (B) Does not cause respiratory depression
(D) Loperamide (C) Is a nonsedating opioid
(E) Nalbuphine (D) Pain-relieving action is not superior to that of
4. An emergency department patient with severe codeine
pain thought to be of gastrointestinal origin received (E) Response to naloxone in overdose may be
80 mg of meperidine. He subsequently developed a unreliable
severe reaction characterized by tachycardia, 9. Which drug does not activate opioid receptors,
hypertension, hyperpyrexia, and seizures. has been proposed as a maintenance drug in
Questioning revealed that the patient had been treatment programs for opioid addicts, and with a
taking a drug for a psychiatric condition. Which drug single oral dose, will block the effects of injected
is most likely to be responsible for this untoward heroin for up to 48 h?
interaction with meperidine? (A) Fentanyl
(A) Alprazolam (B) Nalbuphine
(B) Bupropion (C) Naloxone
(C) Lithium (D) Naltrexone
(D) Phenelzine (E) Propoxyphene
(E) Mirtazapine 10. Which drug is a full agonist at opioid receptors
5. Genetic polymorphisms in certain hepatic with analgesic activity equivalent to morphine, a
enzymes involved in drug metabolism are longer duration of action, and fewer withdrawal
established to be responsible for variations in signs on abrupt discontinuance than morphine?
analgesic response to (A) Fentanyl
(A) Buprenorphine (B) Hydromorphone
(B) Codeine (C) Methadone
(C) Fentanyl (D) Nalbuphine
(D) Methadone (E) Oxycodone
(E) Tramadol
Questions 6 and 7. A young male patient is brought ANSWERS
to the emergency department in an anxious and 1.
agitated state. He informs the attending physician In most situations, pain associated with metastatic
that he uses “street drugs” and that he gave himself carcinoma ultimately necessitates the use of an
an intravenous “fix” approximately 12 h ago. He now opioid analgesic that is equivalent in strength to
has chills and muscle aches and has also been morphine, so hydromorphone, oxymorphone, or
vomiting. His symptoms include hyperventilation levorphanol would be indicated. Pentazocine or the
combination of codeine plus salicylate would not be They usually start within 6–10 h after the last dose;
as effective as the original drug combination. their intensity depends on the degree of physical
Propoxyphene is even less active than codeine dependence, and peak effects usually occur at 36–48
alone. Buprenorphine, a mixed agonist-antagonist, is h. Mydriasis is a prominent feature of the abstinence
not usually recommended for cancer-associated pain syndrome; other symptoms include rhinorrhea,
because it has a limited maximum analgesic effect lacrimation, piloerection, muscle jerks, and yawning.
(“ceiling”) and because of possible dysphoric and The answer is D.
psychotomimetic effects. The answer is C. 7.
2. Prevention of signs and symptoms of withdrawal
Chronic use of strong opioid analgesics leads to the after chronic use of a strong opiate like heroin
development of tolerance to their analgesic, requires replacement with another strong opioid
euphoric, and sedative actions. Tolerance also analgesic drug. Methadone is most commonly used,
develops to their emetic effects and to effects on but other strong μ-receptor agonists would also be
some smooth muscle, including the biliary and the effective. Acetaminophen and codeine will not be
urethral sphincter muscles. However, tolerance does effective. Beneficial effects of diazepam are
not develop significantly to the constipating effects restricted to relief of anxiety and agitation. The
or the miotic actions of the opioid analgesics. The antagonist drug naltrexone may exacerbate
answer is C. withdrawal symptoms. The answer is C.
3. 8.
Codeine and nalbuphine could decrease Nalbuphine and butorphanol are κ agonists, with
gastrointestinal peristalsis, but not without marked weak μ-receptor antagonist activity. They have
side effects (and a prescription). Dextromethorphan analgesic efficacy superior to that of codeine, but it
is a cough suppressant. The other 2 drugs listed are is not equivalent to that of strong opioid receptor
opioids with antidiarrheal actions. Diphenoxylate is agonists. Although these mixed agonist-antagonist
not available over the counter because it is a drugs are less likely to cause respiratory depression
constituent of a proprietary combination that than strong μ activators, if depression does occur,
includes atropine sulfate (Lomotil). The answer is D. reversal with opioid antagonists such as naloxone is
4. unpredictable. Sedation is common. The answer is E.
Concomitant administration of meperidine and 9.
monoamine oxidase inhibitors such as isocarboxazid The opioid antagonist naltrexone has a much longer
or phenelzine has resulted in life-threatening half-life than naloxone, and its effects may last 2 d. A
hyperpyrexic reactions that may culminate in high degree of client compliance would be required
seizures or coma. Such reactions have occurred even for naltrexone to be of value in opioid dependence
when the MAO inhibitor was administered more treatment programs. The same reservation is
than a week after a patient had been treated with applicable to the use of naltrexone in alcoholism.
meperidine. Note that concomitant use of selective The answer is D.
serotonin reuptake inhibitors and meperidine has 10.
resulted in the serotonin syndrome, another life- Fentanyl, hydromorphone, and methadone are full
threatening drug interaction (see Chapter 16). The agonists with analgesic efficacy similar to that of
answer is D. morphine. When given intravenously, fentanyl has a
5. duration of action of just 60–90 min.
Codeine, hydrocodone, and oxycodone are Hydromorphone has poor oral bioavailability.
metabolized by the cytochrome P450 isoform Methadone has the greatest bioavailability of the
CYP2D6, and variations in analgesic response to drugs used orally, and its effects are more
these drugs have been attributed to genotypic prolonged. Tolerance and physical dependence
polymorphisms in this isozyme. In the case of develop, and dissipate, more slowly with methadone
codeine, this may be especially important since the than with morphine. These properties underlie the
drug is demethylated by CYP2D6 to form the active use of methadone for detoxification and
metabolite, morphine (see Chapter 5). The answer is maintenance programs. The answer is C.
B.
6. QUESTIONS
The signs and symptoms are those of withdrawal in a Questions 1 and 2. A 42-year-old homemaker suffers
patient physically dependent on an opioid agonist. from anxiety with phobic symptoms and occasional
panic attacks. She uses over-the-counter after its intravenous administration. His symptoms
antihistamines for allergic rhinitis and claims that are not likely to include
ethanol use is “just 1 or 2 glasses of wine with (A) Agitation
dinner.” Alprazolam, a benzodiazepine, is prescribed, (B) Bradycardia
and the patient is maintained on the drug for 3 yr, (C) Hyperthermia
with several dose increments over that time period. (D) Myocardial infarct
Her family notices that she does not seem to be (E) Seizures
improving and that her speech is often slurred in the 6. Which statement about hallucinogens is accurate?
evenings. She is finally hospitalized with severe (A) Dilated pupils and tachycardia are characteristic
withdrawal signs on one weekend while attempting effects of scopolamine
to end her dependence on drugs. (B) LSD is unique among hallucinogens in that
1. Which statement about the use of alprazolam is animals will self-administer it
accurate? (C) Mescaline and psilocybin exert their CNS actions
(A) Abrupt discontinuance of alprazolam after 4 wk through dopaminergic systems in the brain
of treatment may elicit withdrawal signs (D) Phencyclidine is a known teratogen
(B) Additive CNS depression occurs with ethanol (E) Withdrawal signs characteristic of dependence
(C) Benzodiazepines are Schedule IV-controlled occur with abrupt discontinuance of ketamine
drugs 7. Which statement about inhalants is accurate?
(D) Tolerance can occur with chronic use of any (A) Euphoria, numbness, and tingling sensations with
benzodiazepine visual and auditory disturbances occur in most
(E) All of the above statements are accurate persons who inhale organic nitrites
2. The main reason for hospitalization of this patient (B) Methemoglobinemia is a common toxicologic
was to be able to effectively control problem after repetitive inhalation of industrial
(A) Cardiac arrhythmias solvents
(B) Delirium (C) Nitrous oxide is the most commonly abused drug
(C) Hepatic dysfunction by medical personnel working in hospitals
(D) Seizures (D) Solvent inhalation is mainly a drug abuse
(E) None of the above problem in petroleum industry workers
3. Which drug, a partial agonist at nicotinic (E) The inhalation of isobutyl nitrite is likely to cause
acetycholine receptors, is used in smoking cessation headache, hypotension, and flushing
programs but may cause seizures in overdose? 8. Which sign or symptom is likely to occur with
(A) Acamprosate marijuana?
(B) Buprenorphine (A) Bradycardia
(C) Nalbuphine (B) Conjunctival reddening
(D) Rimonabant (C) Hypertension
(E) Varenicline (D) Increased psychomotor performance
4. Which statement about abuse of the opioid (E) Mydriasis
analgesics is false? Questions 9 and 10. A college student is brought to
(A) Lacrimation, rhinorrhea, yawning, and sweating the emergency department by friends. The physician
are early signs of withdrawal from opioid analgesics is informed that the student had taken a drug and
(B) In withdrawal from opioids, clonidine may be then “went crazy.” The patient is agitated and
useful in reducing symptoms caused by sympathetic delirious. Several persons are required to hold him
overactivity down. His skin is warm and sweaty, and his pupils
(C) Methadone alleviates most of the symptoms of are dilated. Bowel sounds are normal. Signs and
heroin withdrawal symptoms include tachycardia, marked
(D) Most patients experiencing withdrawal from hypertension, hyperthermia, increased muscle tone,
heroin are free of the symptoms of abstinence in 6– and both horizontal and vertical nystagmus.
8d 9. The most likely cause of these signs and
(E) Naloxone may precipitate a severe withdrawal symptoms is intoxication from
state in abusers of opioid analgesics with symptoms (A) Hashish
starting in less than 15–30 min (B) LSD
5. A young male patient is brought to the emergency (C) Mescaline
department suffering from an overdose of cocaine (D) Methamphetamine
(E) Phencyclidine Overdoses with amphetamines or cocaine have
10. The management of this patient is likely to many signs and symptoms in common. However, the
include ability of cocaine to block the reuptake of
(A) Administration of epinephrine norepinephrine at sympathetic nerve terminals
(B) Alkalinization of the urine to increase drug results in greater cardiotoxicity. Tachycardia is the
elimination rule, with the possibility of an arrhythmia, infarct, or
(C) Amitriptyline if psychosis ensues stroke. The answer is B.
(D) Atropine to control hyperthermia 6.
(E) None of the above Psilocybin, mescaline, and LSD have similar central
ANSWERS (via serotonergic systems) and peripheral
1. (sympathomimetic) effects, but no actions on
Therapeutic doses of benzodiazepines may lead to dopaminergic receptors in the CNS. None of the
dependence with withdrawal symptoms including hallucinogenic drugs have been shown to have
anxiety and agitation observable on abrupt teratogenic potential. Unlike most hallucinogens,
discontinuance after a few weeks of treatment. Like PCP (not LSD) acts as a positive reinforcer of self-
most sedative-hypnotics, benzodiazepines are administration in animals. Emergence reactions can
schedule-controlled, exhibiting dependence liability occur after use of ketamine, but they are not signs of
and the development of tolerance. Additive withdrawal. Scopolamine blocks muscarinic
depression occurs with ethanol and many other CNS receptors. The answer is A.
drugs. The answer is E. 7.
2. Male preteens are most likely to “experiment” with
This patient is probably withdrawing from solvent inhalation. This can result in central and
dependence on both alprazolam and alcohol use. In peripheral neurotoxicity, liver and kidney damage,
addition to the symptoms described previously, and pulmonary disease. Opioids, including fentanyl
abrupt withdrawal from sedative-hypnotic and meperidine, are the most widely abused by
dependence may include hyperreflexia progressing medical personnel working in hospitals. Industrial
to seizures, with ensuing coma and possibly death. solvents rarely cause methemoglobinemia, but this
The risk of a seizure is increased if the patient (and headaches, flushing, and hypotension) may
abruptly withdraws from ethanol use at the same occur after excessive use of nitrites. The answer is E.
time. Depending on severity of symptoms, initial 8.
management may require parenteral diazepam or Two of the most characteristic signs of marijuana
lorazepam, with the latter drug often favored in use are increased pulse rate and reddening of the
hepatic dysfunction. The answer is D. conjunctiva. Decreases in blood pressure and in
3. psychomotor performance occur. Pupil size is not
Acamprosate is an antagonist of NMDA glutamate changed by marijuana. The answer is B.
receptors used together with counseling in alcohol 9.
treatment programs. Varenicline blocks the The signs and symptoms point to PCP intoxication.
rewarding effects of nicotine and is used in smoking The presence of both horizontal and vertical
cessation programs. However, the drug may cause nystagmus is pathognomonic. The answer is E.
psychiatric changes and in overdose has caused 10.
seizures. The answer is E. Management of phencyclidine (PCP) overdose
4. involves ventilatory support and control of seizures
Symptoms of opioid withdrawal usually begin within (with a benzodiazepine), hypertension, and
6–8 h, and the acute course may last 6–8 d. hyperthermia. Antipsychotic drugs (eg, haloperidol)
However, a secondary phase of heroin withdrawal, may also be useful for psychosis. None of the drugs
characterized by bradycardia, hypotension, listed are of value. Atropine may cause
hypothermia, and mydriasis, may last 26–30 wk. hyperthermia! Phencyclidine is a weak base, and its
Methadone is commonly used in detoxification of renal elimination may be accelerated by urinary
the heroin addict because it is a strong agonist, has acidification, not alkalinization! A large percentage
high oral bioavailability, and has a relatively long of phencyclidine is secreted into the stomach, so
half-life. The answer is D. removal of the drug may be hastened by activated
5. charcoal or nasogastric suction. The answer is E.
QUESTIONS
Questions 1–4. A 23-year-old pregnant woman is (A) X = intrinsic factor; Y = folic acid.
referred by her obstetrician for evaluation of (B) X = intrinsic factor; Y = vitamin B12
anemia. She is in her fourth month of pregnancy and (C) X = extrinsic factor; Y = parenteral iron
has no history of anemia; her grandfather had (D) X = extrinsic factor; Y = sargramostim
pernicious anemia. Her hemoglobin is 10 g/dL 6. Which of the following is most likely to be
(normal, 12–16 g/dL). required by a 5-year-old boy with chronic renal
1. If this woman has macrocytic anemia, an insufficiency?
increased serum concentration of transferrin, and a (A) Cyanocobalamin
normal serum concentration of vitamin B12, the (B) Deferoxamine
most likely cause of her anemia is deficiency of (C) Erythropoietin
which of the following? (D) Filgrastim (G-CSF)
(A) Cobalamin (E) Oprelvekin (IL-11)
(B) Erythropoietin 7. In a patient who requires filgrastim (G-CSF) after
(C) Folic acid being treated with anticancer drugs, the therapeutic
(D) Intrinsic factor objective is to prevent which of the following?
(E) Iron (A) Allergic reactions
2. The laboratory data for your pregnant patient (B) Cancer recurrence
indicate that she does not have macrocytic anemia (C) Excessive bleeding
but rather microcytic anemia. Optimal treatment of (D) Hypoxia
normocytic or mild microcytic anemia associated (E) Systemic infection
with pregnancy uses which of the following? 8. The megaloblastic anemia that results from
(A) A high-fiber diet vitamin B12 deficiency is due to inadequate supplies
(B) Erythropoietin injections of which of the following?
(C) Ferrous sulfate tablets (A) Cobalamin
(D) Folic acid supplements (B) dTMP
(E) Hydroxocobalamin injections (C) Folic acid
3. If this patient has a young child at home and is (D) Homocysteine
taking iron-containing prenatal supplements, she (E) N 5-methyltetrahydrofolate
should be warned that they are a common source of Questions 9 and 10. After undergoing surgery for
accidental poisoning in young children and advised breast cancer, a 53-year-old woman is scheduled to
to make a special effort to keep these pills out of her receive 4 cycles of cancer chemotherapy. The cycles
child’s reach. Toxicity associated with acute iron are to be administered every 3–5 wk. Her first cycle
poisoning usually includes which of the following? was complicated by severe chemotherapy-induced
(A) Dizziness, hypertension, and cerebral thrombocytopenia.
hemorrhage 9. During the second cycle of chemotherapy, it
(B) Hyperthermia, delirium, and coma would be appropriate to consider treating this
(C) Hypotension, cardiac arrhythmias, and seizures patient with which of the following?
(D) Necrotizing gastroenteritis, shock, and metabolic (A) Darbepoetin alpha
acidosis (B) Filgrastim (G-CSF)
(E) Severe hepatic injury, encephalitis, and coma (C) Iron dextran
4. The child in the previous question did ingest the (D) Oprelvekin (IL-11)
iron-containing supplements. What immediate (E) Vitamin B12
treatment is necessary? Correction of acid-base and 10. Twenty months after finishing her
electrolyte abnormalities and chemotherapy, the woman had a relapse of breast
(A) Activated charcoal cancer. The cancer was now unresponsive to
(B) Oral deferasirox standard doses of chemotherapy. The decision was
(C) Parenteral deferoxamine made to treat the patient with high-dose
(D) Parenteral dantrolene chemotherapy followed by autologous stem cell
5. A 45-year-old male stomach cancer patient transplantation. Which of the following drugs is most
underwent tumor removal surgery. After surgery, he likely to be used to mobilize the peripheral blood
developed megaloblastic anemia. His anemia is stem cells needed for the patient’s autologous stem
caused by a deficiency of X and can be treated with cell transplantation?
Y. (A) Erythropoietin
(B) Filgrastim (G-CSF) Homocysteine and N 5-methyltetrahydrofolate
(C) Folic acid accumulate. The answer is B.
(D) Intrinsic factor 9.
(E) Oprelvekin (interleukin-11) Oprelvekin (IL-11) stimulates platelet production and
ANSWERS decreases the number of platelet transfusions
1. required by patients undergoing bone marrow
Deficiencies of folic acid or vitamin B12 are the most suppression therapy for cancer. The answer is D.
common causes of megaloblastic anemia. If a patient 10.
with this type of anemia has a normal serum vitamin The success of transplantation with peripheral blood
B12 concentration, folate deficiency is the most stem cells depends on infusion of adequate numbers
likely cause of the anemia. The answer is C. of hematopoietic stem cells. Administration of G-CSF
2. to the donor (in the case of autologous
Iron deficiency microcytic anemia is the anemia that transplantation, the patient who also will be the
is most commonly associated with pregnancy. In this recipient of the transplantation) greatly increases
condition, oral iron supplementation is indicated. the number of hematopoietic stem cells harvested
The answer is C. from the donor’s blood. The answer is B.
3. QUESTIONS
Acute iron poisoning often causes severe Questions 1–3. A 55-year-old lawyer is brought to
gastrointestinal damage resulting from direct the emergency department 2 h after the onset of
corrosive effects, shock from fluid loss in the severe chest pain during a stressful meeting. He has
gastrointestinal tract, and metabolic acidosis from a history of poorly controlled mild hypertension and
cellular dysfunction. The answer is D. elevated blood cholesterol but does not smoke. ECG
4. changes (ST elevation) and cardiac enzymes confirm
Activated charcoal does not bind iron and thus is the diagnosis of myocardial infarction. The decision
ineffective. Oral deferasirox is effective for chronic is made to attempt to open his occluded artery.
iron toxicity. Dantrolene inhibits Ca2+ release from 1. Which of the following drugs accelerates the
the sarcoplasmic reticulum and is an antidote for conversion of plasminogen to plasmin?
malignant hyperthermia induced by inhaled (A) Aminocaproic acid
anesthetics. The answer is C. (B) Heparin
5. (C) Argatroban
Resection of the stomach does lead to loss of (D) Reteplase
intrinsic factor and the patient will be deficient in (E) Warfarin
vitamin B12. Prevention or treatment of iron 2. If a fibrinolytic drug is used for treatment of this
deficiency anemia (microcytic cell size) is the only man’s acute myocardial infarction, which of the
indication for iron administration. Sargramostim is a following adverse drug effects is most likely to
GM-CSF and is used to stimulate the production of occur?
neutrophils and other myeloid and megakaryocyte (A) Acute renal failure
progenitors. The answer is B. (B) Development of antiplatelet antibodies
6. (C) Encephalitis secondary to liver dysfunction
The kidney produces erythropoietin; patients with (D) Hemorrhagic stroke
chronic renal insufficiency often require exogenous (E) Neutropenia
erythropoietin to avoid chronic anemia. The answer 3. If this patient undergoes a percutaneous coronary
is C. angiography procedure and placement of a stent in a
7. coronary blood vessel, he will need to be on dual
Filgrastim (G-CSF) stimulates the production and antiplatelet therapy. eg, aspirin and clopidogrel for
function of neutrophils, important cellular mediators at least a year. Which of the following most
of the innate immune system that serve as the first accurately describes the mechanism of action of
line of defense against infection. The answer is E. clopidogrel?
8. (A) Clopidogrel directly binds to the platelet ADP
Deficiency of vitamin B12 (cobalamin) leads to a receptors
deficiency in tetrahydrofolate and subsequently a (B) Clopidogrel irreversibly inhibits cyclooxygenase
deficiency of the dTMP required for DNA synthesis. (C) Clopidogrel facilitates the action of antithrombin
III
(D) The active metabolite of clopidogrel binds to the (A) Can be used without monitoring the patient’s
platelet ADP receptors aPTT
(E) The active metabolite of clopidogrel binds to the (B) Has a shorter duration of action
platelet glycoprotein IIb/IIIa receptors (C) Is less likely to have a teratogenic effect
Questions 5–7. A 58-year-old woman with chronic (D) Is more likely to be given intravenously
hypertension and diabetes mellitus was recently (E) Is more likely to cause thrombosis and
admitted to the hospital for congestive heart failure thrombocytopenia
and new onset atrial fibrillation. She is now seeing 9. During the next week, the patient was started on
you after discharge and, though feeling better, is still warfarin and her enoxaparin was discontinued. Two
in atrial fibrillation. An echocardiogram shows an months later, she returned after a severe nosebleed.
ejection fraction of 40%; there are no valvular Laboratory analysis revealed an INR (international
abnormalities. An ECG reveals only atrial fibrillation. normalized ratio) of 7.0 (INR value in such a
You calculate her risk using the CHADS(2) system and warfarin-treated patient should be 2.0–3.0). To
the score indicates that she requires anticoagulation prevent severe hemorrhage, the warfarin should be
rather than antiplatelet therapy. discontinued and this patient should be treated
5. You are discussing the risks and benefits of immediately with which of the following?
anticoagulation therapy with her, including the (A) Aminocaproic acid
option of using direct thrombin inhibitors. Which of (B) Desmopressin
the following anticoagulants is a direct inhibitor of (C) Factor VIII
thrombin? (D) Protamine
(A) Abciximab (E) Vitamin K1
(B) Dabigatran 10. A patient develops severe thrombocytopenia in
(C) Rivaroxaban response to treatment with unfractionated heparin
(D) Warfarin and still requires parenteral anticoagulation. The
6. She tells you that her main reason for not wanting patient is most likely to be treated with which of the
oral anticoagulation is that she does not want to following?
come to clinic for frequent blood draws. You agree (A) Abciximab
on an oral alternative and start her on apixaban. You (B) Bivalirudin
counsel her extensively on the importance of taking (C) Tirofiban
the medication each day, as suddenly stopping can (D) Plasminogen
lead to ANSWERS
(A) Anaphylaxis 1.
(B) Excess bleeding Reteplase is the only thrombolytic drug listed.
(C) Increase in INR Heparin and warfarin are anticoagulants. Argatroban
(D) Stroke is a direct inhibitor of thrombin, and aminocaproic
(E) Thrombocytopenia acid is an inhibitor, not an activator, of the
7. She is excited about not having to come in for conversion of plasminogen to plasmin. The answer is
blood tests but wonders if there is a test, just in case D.
the doctors need to know. Which of the following 2.
tests would provide accurate information about the The most common serious adverse effect of the
coagulation status of a patient taking apixaban? fibrinolytics is bleeding, especially in the cerebral
(A) aPTT circulation. The fibrinolytics do not usually have
(B) Factor X test serious effects on the renal, hepatic, or hematologic
(C) INR systems. Unlike heparin, they do not induce
(D) PT test antiplatelet antibodies. The answer is D.
Questions 8 and 9. A 67-year-old woman presents 3.
with pain in her left thigh muscle. Duplex Clopidogrel is a prodrug that is activated by CYP2C9
ultrasonography indicates the presence of deep vein and CYP2C19. It irreversibly binds to the ADP
thrombosis (DVT) in the affected limb. receptor on the surface of platelets that serves as a
8. The decision was made to treat this woman with key role in platelet aggregation. Aspirin and
enoxaparin. Relative to unfractionated heparin, clopidogrel help prevent platelet-induced occlusion
enoxaparin of coronary stents. The answer is D.
4.
A drug that increases metabolism (clearance) of the 1. PJ is a 4.5-year-old boy. At his checkup, the
anticoagulant lowers the steady-state plasma pediatrician notices cutaneous xanthomas and
concentration (both free and bound forms), whereas orders a lipid panel. Repeated measures confirm
one that displaces the anticoagulant increases the that the patient’s serum cholesterol levels are high
plasma level of the free form only until elimination (936 mg/dL). Further testing confirms a diagnosis of
of the drug has again lowered it to the steady-state homozygous familial hypercholesterolemia. Which of
level. The answer is C. the following interventions will be least effective in
5. this patient?
Abciximab is an antiplatelet agent that binds to and (A) Atorvastatin
inhibits GPIIb/IIIa. Rivaroxaban is an oral factor X (B) Ezetimibe
inhibitor and warfarin inhibits vitamin K epoxide (C) Lomitapide
reductase (VKOR). The answer is B. (D) Mipomersen
6. (E) Niacin
Due to the shorter half-life of the oral factor X and 2. A 46-year-old woman with a history of
thrombin inhibitors, the anticoagulant status of the hyperlipidemia was treated with a drug. The chart
patient changes rapidly. Sudden cessation of short- below shows the results of the patient’s fasting lipid
acting oral anticoagulants can lead to stroke. Excess panel before treatment and 6 mo after initiating
bleeding is associated with taking any of the drug therapy. Normal values are also shown. Which
anticoagulants not with stopping them. An increase of the following drugs is most likely to be the one
in INR reflects increased anticoagulation by warfarin. that this patient received?
Thrombocytopenia is a risk associated with heparin. (A) Colestipol
The answer is D. (B) Ezetimibe
7. (C) Gemfibrozil
INR (measured as PT test) reflects changes due to (D) Lovastatin
warfarin and to some extent the thrombin inhibitors. (E) Niacin
Factor X inhibition is not reliably measured by the Questions 3–6. A 35-year-old woman appears to
aPTT (used for unfractionated heparin) or PT test. have familial combined hyperlipidemia. Her serum
The answer is B. concentrations of total cholesterol, LDL cholesterol,
8. and triglyceride are elevated. Her serum
Enoxaparin is an LMW heparin. LMW heparins have concentration of HDL cholesterol is somewhat
a longer half-life than standard heparin and a more reduced.
consistent relationship between dose and 3. Which of the following drugs is most likely to
therapeutic effect. Enoxaparin is given increase this patient’s triglyceride and VLDL
subcutaneously, not intravenously. It is less, not cholesterol concentrations when used as
more, likely to cause thrombosis and monotherapy?
thrombocytopenia. Neither LMW heparins nor (A) Atorvastatin
standard heparin are teratogenic. The aPTT is not (B) Cholestyramine
useful for monitoring the effects of LMW heparins. (C) Ezetimibe
The answer is A. (D) Gemfibrozil
9. (E) Niacin
The elevated INR indicates excessive anticoagulation 4. If this patient is pregnant, which of the following
with a high risk of hemorrhage. Warfarin should be drugs should be avoided because of a risk of harming
discontinued and vitamin K1 administered to the fetus?
accelerate formation of vitamin K-dependent (A) Cholestyramine
factors. The answer is E. (B) Ezetimibe
10. (C) Fenofibrate
Direct thrombin inhibitors such as bivalirudin and (D) Niacin
argatroban provide parenteral anticoagulation (E) Pravastatin
similar to that achieved with heparin, but the direct 5. The patient is started on gemfibrozil. Which of the
thrombin inhibitors do not induce formation of following is a major mechanism of gemfibrozil’s
antiplatelet antibodies. The answer is B. action?
QUESTIONS (A) Increased excretion of bile acid salts
(B) Increased expression of high-affinity LDL 10. Six months after beginning atorvastatin, the
receptors patient’s total and LDL cholesterol concentrations
(C) Increased secretion of VLDL by the liver remained above normal, and he continued to have
(D) Increased triglyceride hydrolysis by lipoprotein anginal attacks despite good adherence to his
lipase antianginal medications. His physician decided to
(E) Reduced uptake of dietary cholesterol add ezetimibe. Which of the following is the most
6. Which of the following is a major toxicity accurate description of ezetimibe’s mechanism of an
associated with gemfibrozil therapy? action?
(A) Bloating and constipation (A) Decreased lipid synthesis in adipose tissue
(B) Cholelithiasis (B) Decreased secretion of VLDL by the liver
(C) Hyperuricemia (C) Decreased gastrointestinal absorption of
(D) Liver damage cholesterol
(E) Severe cardiac arrhythmia (D) Increased endocytosis of HDL by the liver
Questions 7–10. A 43-year-old man has (E) Increased lipid hydrolysis by lipoprotein lipase
heterozygous familial hypercholesterolemia. His ANSWERS
serum concentrations of total cholesterol and LDL 1.
are markedly elevated. His serum concentration of Homozygous familial hypercholesterolemia is caused
HDL cholesterol, VLDL cholesterol, and triglycerides by mutations leading to dysfunctional LDL receptors
are normal or slightly elevated. The patient’s mother incapable of taking up LDL from the bloodstream.
and older brother died of myocardial infarctions Options B–E would have a cholesterol-lowering
before the age of 50. This patient recently effect. Lomitapide and mipomersen are specifically
experienced mild chest pain when walking upstairs indicated for patients with familial
and has been diagnosed as having angina of effort. hypercholesterolemia. Reductase inhibitors such as
The patient is somewhat overweight. He drinks atorvastatin rely on functional LDL receptors to
alcohol most evenings and smokes about 1 pack of achieve a LDL-lowering effect and thus will not work
cigarettes per week. in patients with homozygous familial
7. Consumption of alcohol is associated with which hypercholesterolemia. The answer is A.
of the following changes in serum lipid 2.
concentrations? This patient presents with striking
(A) Decreased chylomicrons hypertriglyceridemia, elevated VLDL cholesterol, and
(B) Decreased HDL cholesterol depressed HDL cholesterol. Six months after drug
(C) Decreased VLDL cholesterol treatment was initiated, her triglyceride and VLDL
(D) Increased LDL cholesterol cholesterol have dropped dramatically and her HDL
(E) Increased triglyceride cholesterol level has doubled. The drug that is most
likely to have achieved all of these desirable
8. If the patient has a history of gout, which of the changes, particularly the large increase in HDL
following drugs is most likely to exacerbate this cholesterol, is niacin. Although gemfibrozil lowers
condition? triglyceride and VLDL concentrations, it does not
(A) Colestipol cause such large increases in HDL cholesterol and
(B) Ezetimibe decreases in LDL cholesterol. The answer is E.
(C) Gemfibrozil 3.
(D) Niacin In some patients with familial combined
(E) Simvastatin hyperlipidemia and elevated VLDL, the resins
9. After being counseled about lifestyle and dietary increase VLDL and triglyceride concentrations even
changes, the patient was started on atorvastatin. though they also lower LDL cholesterol. The answer
During his treatment with atorvastatin, it is is B.
important to routinely monitor serum 4.
concentrations of which of the following? The HMG-CoA reductase inhibitors are
(A) Blood urea nitrogen contraindicated in pregnancy because of the risk of
(B) Alanine and aspartate aminotransferase teratogenic effects. The answer is E.
(C) Platelets 5.
(D) Red blood cells A major mechanism recognized for gemfibrozil is
(E) Uric acid increased activity of the lipoprotein lipase associated
with capillary endothelial cells. Gemfibrozil and (A) Bone marrow suppression and possibly aplastic
other fibrates decrease VLDL secretion, presumably anemia
by stimulating hepatic fatty acid oxidation. The (B) Fever, hepatic dysfunction, and encephalopathy
answer is D. (C) Hyperthermia, metabolic acidosis, and coma
6. (D) Rapid, fulminant hepatic failure
A major toxicity of the fibrates is increased risk of (E) Rash, interstitial nephritis, and acute renal failure
gallstone formation, which may be due to enhanced 4. Which of the following drugs is most likely to
biliary excretion of cholesterol. The answer is B. increase serum concentrations of conventional
7. doses of methotrexate, a weak acid that is primarily
Chronic ethanol ingestion can increase serum cleared in the urine?
concentrations of VLDL and triglycerides. This is one (A) Acetaminophen
of the factors that places patients with alcoholism at (B) Allopurinol
risk of pancreatitis. Chronic ethanol ingestion also (C) Colchicine
has the possibly beneficial effect of raising, not (D) Hydroxychloroquine
decreasing, serum HDL concentrations. The answer (E) Probenecid
is E. 5. The main advantage of ketorolac over aspirin is
8. that ketorolac
Niacin can exacerbate both hyperuricemia and (A) Can be combined more safely with an opioid
glucose intolerance. The answer is D. such as codeine
9. (B) Can be obtained as an over-the-counter agent
The 2 primary adverse effects of the HMG-CoA (C) Does not prolong the bleeding time
reductase inhibitors are hepatotoxicity and (D) Is available in a parenteral formulation that can
myopathy. Patients taking these drugs should have be injected intramuscularly or intravenously
liver function tests performed before starting (E) Is less likely to cause acute renal failure in
therapy, and at regular intervals as needed during patients with some preexisting degree of renal
therapy. Serum concentrations of alanine and impairment
aspartate aminotransferase are used as markers of 6. An 18-month-old boy dies from an accidental
hepatocellular toxicity. The answer is B. overdose of acetaminophen. Which of the following
10. is the most likely cause of this patient’s death?
The major recognized effect of ezetimibe is (A) Arrhythmia
inhibition of absorption of cholesterol in the (B) Hemorrhagic stroke
intestine. The answer is C. (C) Liver failure
QUESTIONS (D) Noncardiogenic pulmonary edema
1. Among NSAIDs, aspirin is unique because it (E) Ventilatory failure
(A) Irreversibly inhibits its target enzyme Questions 7 and 8. A 52-year-old woman presented
(B) Prevents episodes of gouty arthritis with long- with intense pain, warmth, and redness in the first
term use toe on her left foot. Examination of fluid withdrawn
(C) Reduces fever from the inflamed joint revealed crystals of uric acid.
(D) Reduces the risk of colon cancer 7. In the treatment of this woman’s acute attack of
(E) Selectively inhibits the COX-2 enzyme gout, a high dose of colchicine will reduce the pain
2. Which of the following is an analgesic and and inflammation. However, many physicians prefer
antipyretic drug that lacks an anti-inflammatory to treat acute gout with a corticosteroid or
action? indomethacin because high doses of colchicine are
(A) Acetaminophen likely to cause
(B) Celecoxib (A) Behavioral changes that include psychosis
(C) Colchicine (B) High blood pressure
(D) Indomethacin (C) Rash
(E) Probenecid (D) Severe diarrhea
3. A 16-year-old girl comes to the emergency (E) Sudden gastrointestinal bleeding
department suffering from the effects of an aspirin 8. Over the next 7 mo, the patient had 2 more
overdose. Which of the following syndromes is this attacks of acute gout. Her serum concentration of
patient most likely to exhibit as a result of this drug uric acid was elevated. The decision was made to put
overdose? her on chronic drug therapy to try to prevent
subsequent attacks. Which of the following drugs 4.
could be used to decrease this woman’s rate of Like other weak acids, methotrexate depends on
production of uric acid? active tubular excretion in the proximal tubule for
(A) Allopurinol efficient elimination. Probenecid competes with
(B) Aspirin methotrexate for binding to the proximal tubule
(C) Colchicine transporter and thereby decreases the rate of
(D) Hydroxychloroquine clearance of methotrexate. The answer is E.
(E) Probenecid 5.
Questions 9 and 10. A 54-year-old woman presented Ketorolac exerts typical NSAID effects. It prolongs
with signs and symptoms consistent with an early the bleeding time and can impair renal function,
stage of rheumatoid arthritis. The decision was made especially in a patient with preexisting renal disease.
to initiate NSAID therapy. Its primary use is as a parenteral agent for pain
9. Which of the following patient characteristics is management, especially for treatment of
the most compelling reason for avoiding celecoxib in postoperative patients. The answer is D.
the treatment of her arthritis? 6.
(A) History of alcohol abuse In overdose, acetaminophen causes fulminant liver
(B) History of gout failure as a result of its conversion by hepatic
(C) History of myocardial infarction cytochrome P450 enzymes to a highly reactive
(D) History of osteoporosis metabolite. The answer is C.
(E) History of peptic ulcer disease 7.
10. Although the patient’s disease was adequately At doses needed to treat acute gout, colchicine
controlled with an NSAID and methotrexate for frequently causes significant diarrhea. Such
some time, her symptoms began to worsen and gastrointestinal effects are less likely with the lower
radiologic studies of her hands indicated progressive doses used in chronic gout. The answer is D.
destruction in the joints of several fingers. 8.
Treatment with another second-line agent for Allopurinol is the only drug listed that decreases
rheumatoid arthritis was considered. Which of the production of uric acid. Probenecid increases uric
following is a parenterally administered DMARD acid excretion. Colchicine and hydroxychloroquine
whose mechanism of anti-inflammatory action is do not affect uric acid metabolism. Aspirin actually
antagonism of tumor necrosis factor? slows renal secretion of uric acid and raises uric acid
(A) Cyclosporine blood levels. It should not be used in gout. The
(B) Etanercept answer is A.
(C) Penicillamine 9.
(D) Phenylbutazone Celecoxib is a COX-2-selective inhibitor. Although the
(E) Sulfasalazine COX-2 inhibitors have the advantage over
ANSWERS nonselective NSAIDs of reduced gastrointestinal
1. toxicity, clinical data suggest that they are more
Aspirin differs from other NSAIDs by irreversibly likely to cause arterial thrombotic events. A history
inhibiting cyclooxygenase. The answer is A. of myocardial infarction would be a compelling
2. reason to avoid a COX-2 inhibitor. The answer is C.
Acetaminophen is the only drug that fits this 10.
description. Indomethacin is a nonselective COX Etanercept is a recombinant protein that binds to
inhibitor and celecoxib is a COX-2 inhibitor; both tumor necrosis factor and prevents its inflammatory
have analgesic, antipyretic, and anti-inflammatory effects. The answer is B.
effects. Colchicine is a drug used for gout that also QUESTIONS
has an anti-inflammatory action. Probenecid is a 1. A young couple (25-year-old male, 23-year-old
uricosuric drug that promotes the excretion of uric female) wants to start a family. They have not
acid. The answer is A. conceived after 1 yr of unprotected intercourse.
3. Infertility evaluation revealed no abnormalities in
Salicylate intoxication is associated with metabolic the female partner and low sperm count in the male.
acidosis, dehydration, and hyperthermia. If these Which of the following is a drug that is purified from
problems are not corrected, coma and death ensue. the urine of postmenopausal women and is used to
The answer is C. promote spermatogenesis in infertile men?
(A) Desmopressin deviations) and appears to have loose skin on her
(B) Gonadorelin neck. Cytogenetic testing reveals an XO karyotype.
(C) Goserelin Which of the following drugs will allow her to
(D) Somatropin achieve a higher adult height?
(E) Urofollitropin (A) Adrenocorticotropin (ACTH)
2. A 29-year-old woman in her 41st wk of gestation (B) Corticotropin-releasing hormone (CRH)
had been in labor for 12 h. Although her uterine (C) Growth hormone-releasing hormone (GHRH)
contractions had been strong and regular initially, (D) Gonadotropin-releasing hormone (GnRH)
they had diminished in force during the past hour. (E) Somatropin
Which of the following agents would be used to 7. A 3-year-old girl presented with hirsutism, breast
facilitate this woman’s labor and delivery? enlargement, and a height and bone age that was
(A) Dopamine consistent with an age of 9. Diagnostic testing
(B) Leuprolide revealed precocious puberty. Which of the following
(C) Oxytocin is the most appropriate drug for treatment of this
(D) Prolactin patient’s precocious puberty?
(E) Vasopressin (A) Atosiban
3. A 3-year-old boy with failure to thrive and (B) Follitropin
metabolic disturbances was found to have an (C) Leuprolide
inactivating mutation in the gene that encodes the (D) Octreotide
growth hormone receptor. Which of the following (E) Pegvisomant
drugs is most likely to improve his metabolic 8. A 47-year-old man exhibited signs and symptoms
function and promote his growth? of acromegaly. Radiologic studies indicated the
(A) Atosiban presence of a large pituitary tumor. Surgical
(B) Bromocriptine treatment of the tumor was only partially effective
(C) Mecasermin in controlling his disease. At this point, which of the
(D) Octreotide following drugs is most likely to be used as
(E) Somatropin pharmacologic therapy?
4. An important difference between leuprolide and (A) Cosyntropin
ganirelix is that ganirelix (B) Desmopressin
(A) Can be administered as an oral formulation (C) Leuprolide
(B) Can be used alone to restore fertility to (D) Octreotide
hypogonadal men and women (E) Somatropin
(C) Immediately reduces gonadotropin secretion 9. A 37-year-old woman with infertility due to
(D) Initially stimulates pituitary production of LH and obstructed fallopian tubes was undergoing ovulation
FSH induction in preparation for in vitro fertilization.
(E) Must be administered in a pulsatile fashion After 10 d of treatment with leuprolide, the next
5. A 27-year-old woman with amenorrhea, infertility, step in the procedure is most likely to involve 10–14
and galactorrhea was treated with a drug that d of treatment with which of the following?
successfully restored ovulation and menstruation. (A) Bromocriptine
Before being given the drug, the woman was (B) Follitropin
carefully questioned about previous mental health (C) Gonadorelin
problems, which she did not have. She was advised (D) hCG
to take the drug orally. Which of the following is (E) Pergolide
most likely to be the drug that was used to treat this 10. A 7-year-old boy underwent successful
patient? chemotherapy and cranial radiation for treatment of
(A) Bromocriptine acute lymphocytic leukemia. One month after the
(B) Desmopressin completion of therapy, the patient presented with
(C) Human gonadotropin hormone excessive thirst and urination plus hypernatremia.
(D) Leuprolide Laboratory testing revealed pituitary diabetes
(E) Octreotide insipidus. To correct these problems, this patient is
6. A 3-year-old girl was referred to the genetic likely to be treated with which of the following?
counselor by her pediatrician. She presents with (A) Corticotropin
short stature (height is 85 cm, –3 standard (B) Desmopressin
(C) hCG suppressing gonadotropin secretion with continuous
(D) Menotropins administration of a long-acting GnRH agonist such as
(E) Thyrotropin leuprolide. The answer is C.
ANSWERS 8.
1. Octreotide, a somatostatin analog, has some efficacy
Spermatogenesis in males requires the action of FSH in reducing the excess GH production that causes
and LH. Urofollitropin, which is purified from the acromegaly. The answer is D.
urine of postmenopausal women, is used clinically to 9.
provide FSH activity. The answer is E. Once the patient’s endogenous gonadotropin
2. production has been inhibited through continuous
Oxytocin is an effective stimulant of uterine administration of the GnRH agonist leuprolide, the
contraction that is routinely used to augment labor. next step in ovulation induction is the administration
The answer is C. of a drug with FSH activity to stimulate follicle
3. maturation. Follitropin is recombinant FSH. The only
This child’s condition is due to the inability of GH to other drug listed that is used in ovulation induction
stimulate the production of insulin-like growth is hCG, but this is an LH analog. The answer is B.
factors, the ultimate mediators of GH effects. 10.
Mecasermin, a combination of recombinant IGF-1 Pituitary diabetes insipidus results from deficiency in
and the binding protein that protects IGF-1 from vasopressin. It is treated with desmopressin, a
immediate destruction, will help correct the IGF peptide agonist of vasopressin V2 receptors. The
deficiency. Because of the inactive GH receptors, answer is B.
somatropin will not be effective. The answer is C. QUESTIONS
4. Questions 1–3. A 24-year-old woman was found to
Leuprolide is an agonist of GnRH receptors, whereas have mild hyperthyroidism due to Graves’ disease.
ganirelix is an antagonist. Although both drugs can She appears to be in good health otherwise.
be used to inhibit gonadotropin release, ganirelix 1. In Graves’ disease, the cause of the
does so immediately, whereas leuprolide does so hyperthyroidism is the production of an antibody
only after about 1 wk of sustained activity. The that does which of the following?
answer is C. (A) Activates the pituitary thyrotropin-releasing
5. hormone (TRH) receptor and stimulates TSH release
Bromocriptine, a dopamine receptor agonist, is used (B) Activates the thyroid gland TSH receptor and
to treat the amenorrhea-galactorrhea syndrome, stimulates thyroid hormone synthesis and release
which is a consequence of hyperprolactinemia. (C) Activates thyroid hormone receptors in
Because of its central dopaminergic effects, the drug peripheral tissues
should not be used in patients with a history of (D) Binds to thyroid gland thyroglobulin and
schizophrenia or other forms of psychotic illness. accelerates its proteolysis and the release of its
The answer is A. supply of T4 and T3
6. (E) Binds to thyroid-binding globulin (TBG) and
Adrenocorticotropin (ACTH) is used diagnostically in displaces bound T4 and T3
suspected adrenal insufficiency. Corticotropin- 2. The decision is made to begin treatment with
releasing hormone (CRH) is used to distinguish methimazole. Methimazole reduces serum
Cushing’s syndrome from ectopic ACTH secretion. concentration of T3 primarily by which of the
GHRH is rarely used as treatment. Its main use is as a following mechanisms?
diagnostic tool. GnRH can be used to treat infertility. (A) Accelerating the peripheral metabolism of T3
Somatropin, recombinant human GH, promotes (B) Inhibiting the proteolysis of thyroid-binding
growth in children with Turner’s syndrome (an XO globulin
genetic genotype) or chronic renal failure. It also (C) Inhibiting the secretion of TSH
helps combat the AIDS-associated wasting (D) Inhibiting the uptake of iodide by cells in the
syndrome. The answer is E. thyroid
7. (E) Preventing the addition of iodine to tyrosine
In precocious puberty, the hypothalamic-pituitary- residues on thyroglobulin
gonadal axis becomes prematurely active for reasons 3. Though rare, a serious toxicity associated with the
that are not understood. Treatment involves thioamides is which of the following?
(A) Agranulocytosis (B) T3
(B) Lupus erythematosus-like syndrome (C) T4
(C) Myopathy (D) TSH
(D) Torsades de pointes arrhythmia (E) FSH
(E) Thrombotic thrombocytic purpura (TTP) 9. A 62-year-old woman presents with complaints of
4. A 56-year-old woman presented to the emergency fatigue, sluggishness, and weight gain. She needs to
department with tachycardia, shortness of breath, nap several times a day, which is unusual for her.
and chest pain. She had had shortness of breath and She has been taking T4 for the past 15 yr without
diarrhea for the last 2 d and was sweating and significant problems regarding her energy level. Her
anxious. A relative reported that the patient had run recent history is significant for diagnosis of
out of methimazole 2 wk earlier. arrhythmia, and she is currently taking an
A TSH measurement revealed a value of <0.01 mIU/L antiarrhythmic drug. What is the most likely cause of
(normal 0.4–4.0 mIU/L). The diagnosis of thyroid her current condition?
storm was made. Which of the following is a drug (A) Amiodarone
that is a useful adjuvant in the treatment of thyroid (B) Lidocaine
storm? (C) Procainamide
(A) Amiodarone (D) Sotalol
(B) Betamethasone (E) Verapamil
(C) Epinephrine 10. A 25-year-old woman presents with insomnia
(D) Propranolol and fears she may have “something wrong with her
(E) Radioactive iodine heart.” She describes “her heart jumping out of her
5. A 65-year-old man with multinodular goiter is chest.” She feels healthy otherwise and reports she
scheduled for a near-total thyroidectomy. Which of has lots of energy. Lab tests confirm
the following drugs will be administered for 10–14 d hyperthyroidism. Which of the following is a drug
before surgery to reduce the vascularity of his that produces a permanent reduction in thyroid
thyroid gland? activity?
(A) Levothyroxine (A) 131I
(B) Liothyronine (B) Methimazole
(C) Lugol’s solution (C) Propylthiouracil
(D) Prednisone (D) Thiocyanate (SCN–)
(E) Radioactive iodine (E) Thyroglobulin
6. Which of the following is a sign or symptom that ANSWERS
would be expected to occur in the event of chronic 1.
overdose with exogenous T4? The antibodies produced in Graves’ disease activate
(A) Bradycardia thyroid gland TSH receptors. Their effects mimic
(B) Dry, puffy skin those of TSH. The answer is B.
(C) Large tongue and drooping of the eyelids 2.
(D) Lethargy, sleepiness The thioamides (methimazole and propylthiouracil)
(E) Weight loss act in thyroid cells to prevent conversion of tyrosine
7. When initiating T4 therapy for an elderly patient residues in thyroglobulin to MIT or DIT. The answer
with longstanding hypothyroidism, it is important to is E.
begin with small doses to avoid which of the 3.
following? Rarely, the thioamides cause severe adverse
(A) A flare-up of exophthalmos reactions that include agranulocytosis, vasculitis,
(B) Acute renal failure hepatic damage, and hypoprothrombinemia. The
(C) Hemolysis answer is A.
(D) Overstimulation of the heart 4.
(E) Seizures In thyroid storm, β blockers such as propranolol are
8. A 27-year-old woman underwent near total useful in controlling the tachycardia and other
thyroidectomy. She was started on levothyroxine. cardiac abnormalities, and propranolol also inhibits
What hormone is produced in the peripheral tissues peripheral conversion of T4 to T3. The answer is D.
when levothyroxine is administered? 5.
(A) Methimazole
Iodides inhibit the synthesis and release of thyroid albuterol (salbutamol) (5 mg) and ipratropium (250
hormone and decrease the size and vascularity of μg), as well as intravenous methyl prednisolone (40
the hyperplastic gland. Lugol’s solution contains a mg) were administered. Which of the following is a
mixture of potassium iodide and iodine. The answer pharmacologic effect of exogenous glucocorticoids?
is C. (A) Increased muscle mass
6. (B) Hypoglycemia
In hyperthyroidism, the metabolic rate increases, (C) Inhibition of leukotriene synthesis
and even though there is increased appetite, weight (D) Improved wound healing
loss often occurs. The other choices are symptoms (E) Increased excretion of salt and water
seen in hypothyroidism. The answer is E. 2. A 34-year-old woman with ulcerative colitis has
7. required long-term treatment with pharmacologic
Patients with longstanding hypothyroidism, doses of a glucocorticoid agonist. Which of the
especially those who are elderly, are highly sensitive following is a toxic effect associated with long-term
to the stimulatory effects of T4 on cardiac function. glucocorticoid treatment?
Administration of regular doses can cause (A) A lupus-like syndrome
overstimulation of the heart and cardiac collapse. (B) Adrenal gland neoplasm
The answer is D. (C) Hepatotoxicity
8. (D) Osteoporosis
The thioamides (methimazole and propylthiouracil) (E) Precocious puberty in children
act in thyroid cells to prevent conversion of tyrosine 3. A 46-year-old male patient has Cushing’s
residues in thyroglobulin to MIT or DIT. syndrome due to an adrenal tumor. Which of the
Levothyroxine (T4) is converted into T3 in the following drugs would be expected to reduce the
periphery. FSH is follicle-stimulating hormone. The signs and symptoms of this man’s disease?
answer is B. (A) Betamethasone
9. (B) Cortisol
Amiodarone is an iodine-containing antiarrhythmic (C) Fludrocortisone
drug with complex effects on the thyroid gland and (D) Ketoconazole
thyroid hormones. One of its actions is to inhibit (E) Triamcinolone
peripheral conversion of T4 to T3. Note that 4. A newborn girl exhibited ambiguous genitalia,
propranolol also reduces conversion of T4 to T3. hyponatremia, hyperkalemia, and hypotension as a
Procainamide (class 1A), lidocaine (class 1B), sotalol result of genetic deficiency of 21α-hydroxylase
(class III), and verapamil (class IV) are activity. Treatment consisted of fluid and salt
antiarrhythmics and have no effect on T4 replacement and hydrocortisone administration. In
conversion. The answer is A. this type of adrenal hyperplasia in which there is
10. excess production of cortisol precursors, which of
Propylthiouracil and, to a much lesser extent, the following describes the primary therapeutic
methimazole inhibit peripheral conversion of T4 to effect of glucocorticoid administration?
T3. Thyroglobulin is not a drug. Radioactive iodine is (A) Increased adrenal estrogen synthesis
the only medical therapy that produces a permanent (B) Inhibition of adrenal aldosterone synthesis
reduction of thyroid activity. Anions such as (C) Prevention of hypoglycemia
thiocyanate (SCN–) and perchlorate (ClO4–) block (D) Recovery of normal immune function
the uptake of iodide by the thyroid gland through (E) Suppression of ACTH secretion
competitive inhibition of the iodide transporter. 5. Which of the following best describes a
Their effectiveness is unpredictable and ClO4– can glucocorticoid response element?
cause aplastic anemia, so these drugs are rarely (A) A protein regulator that controls the interaction
used. The answer is A. between an activated steroid receptor and DNA
QUESTIONS (B) A short DNA sequence that binds tightly to RNA
1. A 50-year-old woman, a known asthmatic for the polymerase
past 30 years, presented to the emergency (C) A small protein that binds to an unoccupied
department with a 2-d history of worsening steroid receptor protein and prevents it from
breathlessness and cough. Chest auscultation becoming denatured
revealed bilateral polyphonic inspiratory and (D) A specific nucleotide sequence that is recognized
expiratory wheeze. Supplemental oxygen, nebulized by a steroid hormone receptor-hormone complex
(E) The portion of the steroid receptor that binds to Glucocorticoids inhibit the production of both
DNA leukotrienes and prostaglandins via inhibition of
6. Glucocorticoids have proved useful in the phospholipase A2. This is a key component of their
treatment of which of the following medical anti-inflammatory action. The answer is C.
conditions? 2.
(A) Chemotherapy-induced vomiting One of the adverse metabolic effects of long-term
(B) Essential hypertension glucocorticoid therapy is a net loss of bone, which
(C) Hyperprolactinemia can result in osteoporosis. The answer is D.
(D) Parkinson’s disease 3.
(E) Type II diabetes Ketoconazole inhibits many types of cytochrome
7. A 56-year-old woman with systemic lupus P450 enzymes. It can be used to reduce the
erythematosus had been maintained on a moderate unregulated overproduction of corticosteroids by
daily dose of prednisone for 9 months. Her disease adrenal tumors. The answer is D.
has finally gone into remission and she now wishes 4.
to gradually taper and then discontinue the A 21α-hydroxylase deficiency prevents normal
prednisone. Gradual tapering of a glucocorticoid is synthesis of cortisol and aldosterone, and causes
required for recovery of which of the following? accumulation of cortisol precursors (Figure 39–2).
(A) Depressed release of insulin from pancreatic B The hypothalamic-pituitary system responds to the
cells abnormally low levels of cortisol by increasing ACTH
(B) Hematopoiesis in the bone marrow release. High levels of ACTH induce adrenal
(C) Normal osteoblast function hyperplasia and excess production of adrenal
(D) The control by vasopressin of water excretion androgens, which can cause virilization of females
(E) The hypothalamic-pituitary-adrenal system and prepubertal males. Glucocorticoid is
8. A patient presents with pain and stiffness in his administered to replace the missing
wrists and knees. The stiffness is worse first thing in mineralocorticoid and glucocorticoid activity and to
the morning. A blood test confirms rheumatoid suppress ACTH release, which removes the stimulus
arthritis. You advise a short course of steroids. for excess adrenal androgen production. The answer
Which one of the following is the most potent anti- is E.
inflammatory steroid? 5.
(A) Cortisol Activated steroid hormone receptors mediate their
(B) Dexamethasone effects on gene expression by binding to hormone
(C) Fludrocortisone response elements, which are short sequences of
(D) Prednisone DNA located near steroid-regulated genes. The
(E) Triamcinolone answer is D.
9. A 54-year-old man with advanced tuberculosis has 6.
developed signs of severe acute adrenal Glucocorticoids are used in combination with other
insufficiency. The patient should be treated antiemetics to prevent chemotherapy-induced
immediately. Which of the following combinations is nausea and vomiting, which are commonly
most rational? associated with anticancer drugs. The answer is A.
(A) Aldosterone and fludrocortisone 7.
(B) Cortisol and fludrocortisone Exogenous glucocorticoids act at the hypothalamus
(C) Dexamethasone and metyrapone and pituitary to suppress the production of CRF and
(D) Fludrocortisone and metyrapone ACTH. As a result, adrenal production of endogenous
(E) Triamcinolone and dexamethasone corticosteroids is suppressed. On discontinuance,
10. Which of the following is a drug that, in high the recovery of normal hypothalamic-pituitary-
doses, blocks the glucocorticoid receptor? adrenal function occurs slowly. Glucocorticoid doses
(A) Aminoglutethimide must be tapered slowly, over several months, to
(B) Beclomethasone prevent adrenal insufficiency. The answer is E.
(C) Ketoconazole 8.
(D) Mifepristone Of the drugs listed, cortisol has the lowest and
(E) Spironolactone dexamethasone the highest anti-inflammatory
ANSWERS activity. The answer is B.
1. 9.
In acute adrenal insufficiency, there is loss of salt progesterone receptors. After this procedure, she
and water that is primarily due to reduced will probably receive which of the following drugs?
production of aldosterone. The loss of salt and water (A) Danazol
can lead to dehydration. A rational combination of (B) Flutamide
drugs should include agents with complementary (C) Leuprolide
effects (ie, a glucocorticoid and a mineralocorticoid). (D) Mifepristone
The combination with these characteristics is cortisol (E) Tamoxifen
and fludrocortisone. (Note that although 5. A 60-year-old man is found to have a prostate
fludrocortisone may have sufficient glucocorticoid lump and an elevated prostate-specific antigen (PSA)
activity for a patient with mild disease, a patient in blood test. Magnetic resonance imaging suggests
severe acute adrenal insufficiency needs a full several enlarged lymph nodes in the lower
glucocorticoid such as cortisol.) The answer is B. abdomen, and an x-ray reveals 2 radiolucent lesions
10. in the bony pelvis. This patient is likely to be treated
Mifepristone is a competitive antagonist of with which of the following drugs?
glucocorticoid and progesterone receptors. (A) Anastrozole
Ketoconazole and aminoglutethimide also (B) Desogestrel
antagonize corticosteroids; however, they act by (C) Leuprolide
inhibiting steroid hormone synthesis. The answer is (D) Methyltestosterone
D. (E) Oxandrolone
QUESTIONS 6. A young woman complains of abdominal pain at
1. A teenager seeks postcoital contraception. Which the time of menstruation. Careful evaluation
of the following preparations will be effective for indicates the presence of significant endometrial
this purpose? deposits on the pelvic peritoneum. Which of the
(A) Clomiphene following is the most appropriate medical therapy
(B) Ethinyl estradiol for this patient?
(C) Diethylstilbestrol (DES) (A) Flutamide, orally
(D) Mifepristone (B) Medroxyprogesterone acetate by intramuscular
(E) Norgestrel injection
2. A 23-year-old woman desires a combined oral (C) Norgestrel as an IUD
contraceptive for pregnancy protection. Which of (D) Oxandrolone by intramuscular injection
the following patient factors would lead a health (E) Raloxifene orally
professional to recommend an alternative form of 7. Diethylstilbestrol (DES) should never be used in
contraception? pregnant women because it is associated with which
(A) Evidence of hirsutism of the following?
(B) History of gastroesophageal reflux disease and is (A) Deep vein thrombosis
currently taking omeprazole (B) Feminization of the external genitalia of male
(C) History of pelvic inflammatory disease offspring
(D) History of migraine headache that is well (C) Infertility and development of vaginal cancer in
controlled by sumatriptan female offspring
(E) She plans to use this contraceptive for about 1 yr (D) Miscarriages
and will then attempt to become pregnant (E) Virilization of the external genitalia of female
3. Men who use large doses of anabolic steroids are offspring
at increased risk of which of the following? 8. Which of the following is a unique property of
(A) Anemia SERMs?
(B) Cholestatic jaundice and elevation of aspartate (A) Act as agonists in some tissues and antagonists in
transaminase levels in the blood other tissues
(C) Hirsutism (B) Activate a unique plasma membrane-bound
(D) Hyperprolactinemia receptor
(E) Testicular enlargement (C) Have both estrogenic and progestational agonist
4. A 50-year-old woman with a positive activity
mammogram undergoes lumpectomy and a small (D) Inhibit the aromatase enzyme required for
carcinoma is removed. Biochemical analysis of the estrogen synthesis
cancer reveals the presence of estrogen and
(E) Produce estrogenic effects without binding to In endometriosis, suppression of ovarian function
estrogen receptors and production of gonadal steroids are useful.
9. Finasteride has efficacy in the prevention of male- Intramuscular injection of relatively large doses of
pattern baldness by virtue of its ability to do which medroxyprogesterone provides
of the following? 3 months of an ovarian suppressive effect because
(A) Competitively antagonize androgen receptors of inhibition of pituitary production of
(B) Decrease the release of gonadotropins gonadotropins. The answer is B.
(C) Increase the serum concentration of sex 7.
hormone-binding globulin Diethylstilbestrol (DES) is a nonsteroidal estrogen
(D) Inhibit the synthesis of testosterone agonist. Several decades ago, misguided use of the
(E) Reduce the production of dihydrotestosterone drug in pregnant women appears to have resulted in
10. A 52-year-old postmenopausal patient has fetal damage that predisposed female offspring to
evidence of low bone mineral density. She and her infertility and a rare form of vaginal cancer. For this
physician are considering therapy with raloxifene or reason, the drug should be avoided in pregnant
a combination of conjugated estrogens and women. Other estrogenic drugs do not appear to
medroxyprogesterone acetate. Which of the have these effects. Although estrogens do increase
following patient characteristics is most likely to lead the risk of deep vein thrombosis, this is not the
them to select raloxifene? reason why DES should be avoided. The answer is C.
(A) Previous hysterectomy 8.
(B) Recurrent vaginitis SERMs such as tamoxifen and raloxifene exhibit
(C) Rheumatoid arthritis tissue-specific estrogenic and antiestrogenic effects.
(D) Strong family history of breast cancer The answer is A.
(E) Troublesome hot flushes 9.
ANSWERS Finasteride inhibits 5α-reductase, the enzyme that
1. converts testosterone to DHT, the principal
Mifepristone, an antagonist at progesterone and androgen in androgen-sensitive hair follicles. The
glucocorticoid receptors, has a luteolytic effect and answer is E.
is effective as a postcoital contraceptive. When 10.
combined with a prostaglandin, it is also an effective Conjugated estrogens and raloxifene both improve
abortifacient. The answer is D. bone mineral density and protect against
2. osteoporosis. The
Estrogen-containing hormonal contraceptives 2 advantages of raloxifene over full estrogen
increase the risk of episodes of migraine headache. receptor agonists are that raloxifene has antagonist
The answer is D. effects in breast tissue and lacks an agonistic effect
3. in endometrium. If a patient’s uterus was removed
In men, large doses of anabolic steroids are by surgery, the difference in the endometrial effect
associated with liver impairment, including is moot. In patients with a strong family history of
cholestasis and elevation of serum concentrations of breast cancer, raloxifene may be a better choice
transaminases. The answer is B. than a full estrogen agonist because it will not
4. further increase the woman’s risk of breast cancer
Tamoxifen has proved useful in adjunctive therapy of and may even lower her risk. The answer is D.
breast cancer; the drug decreases the rate of QUESTIONS
recurrence of cancer. The answer is E. Questions 1 and 2. A 13-year-old boy with type 1
5. diabetes is brought to the hospital complaining of
Leuprolide is a GnRH agonist used in the treatment dizziness. Laboratory findings include severe
of men with prostate cancer. Continuous use leads hyperglycemia, ketoacidosis, and a blood pH of 7.15.
to downregulation of testosterone production. 1. Which of the following agents should be
Initially, the agonist action increases testosterone, administered to achieve rapid control of the severe
causing a tumor flare. To prevent this, flutamide, a ketoacidosis in this diabetic boy?
competitive antagonist of the androgen receptor, is (A) Crystalline zinc insulin
added until downregulation of testosterone is (B) Glyburide
complete. The answer is C. (C) Insulin glargine
6. (D) NPH insulin
(E) Tolbutamide (A) Acarbose
2. Which of the following is the most likely (B) Exenatide
complication of insulin therapy in this patient? (C) Glipizide
(A) Dilutional hyponatremia (D) Pioglitazone
(B) Hypoglycemia (E) Repaglinide
(C) Increased bleeding tendency 8. The PPAR-γ receptor that is activated by
(D) Pancreatitis thiazolidinediones increases tissue sensitivity to
(E) Severe hypertension insulin by which of the following mechanisms?
3. A 24-year-old woman with type 1 diabetes wishes (A) Activating adenylyl cyclase and increasing the
to try tight control of her diabetes to improve her intracellular concentration of cAMP
long-term prognosis. Which of the following (B) Inactivating a cellular inhibitor of the GLUT2
regimens is most appropriate? glucose transporter
(A) Morning injections of mixed insulin lispro and (C) Inhibiting acid glucosidase, a key enzyme in
insulin aspart glycogen breakdown pathways
(B) Evening injections of mixed regular insulin and (D) Regulating transcription of genes involved in
insulin glargine glucose utilization
(C) Morning and evening injections of regular insulin, (E) Stimulating the activity of a tyrosine kinase that
supplemented by small amounts of NPH insulin at phosphorylates the insulin receptor
mealtimes 9. Which of the following drugs is most likely to
(D) Morning injections of insulin glargine, cause hypoglycemia when used as monotherapy in
supplemented by small amounts of insulin lispro at the treatment of type 2 diabetes?
mealtimes (A) Acarbose
(E) Morning injection of NPH insulin and evening (B) Canagliflozin
injection of regular insulin (C) Glyburide
4. Which one of the following drugs promotes the (D) Metformin
release of endogenous insulin? (E) Miglitol
(A) Acarbose (F) Rosiglitazone
(B) Canagliflozin 10. Which of the following patients is most likely to
(C) Glipizide be treated with intravenous glucagon?
(D) Metformin (A) An 18-year-old woman who took an overdose of
(E) Miglitol cocaine and now has a blood pressure of 190/110
(F) Pioglitazone mm Hg
5. Which of the following is an important effect of (B) A 27-year-old woman with severe diarrhea
insulin? caused by a flare in her inflammatory bowel disease
(A) Increased conversion of amino acids into glucose (C) A 57-year-old woman with type 2 diabetes who
(B) Increased gluconeogenesis has not taken her glyburide for the last 3 d
(C) Increased glucose transport into cells (D) A 62-year-old man with severe bradycardia and
(D) Inhibition of lipoprotein lipase hypotension resulting from ingestion of an overdose
(E) Stimulation of glycogenolysis of atenolol
6. A 54-year-old obese patient with type 2 diabetes (E) A 74-year-old man with lactic acidosis as a
has a history of alcoholism. In this patient, complication of severe infection and shock
metformin should either be avoided or used with ANSWERS
extreme caution because the combination of 1.
metformin and ethanol increases the risk of which of Oral antidiabetic agents (listed in Table 41–1) are
the following? inappropriate in this patient because he has insulin-
(A) A disulfiram-like reaction dependent diabetes. He needs a rapid-acting insulin
(B) Excessive weight gain preparation that can be given intravenously (see
(C) Hypoglycemia Figure 41–1). The answer is A.
(D) Lactic acidosis 2.
(E) Serious hepatotoxicity Because of the risk of brain damage, the most
7. Which of the following drugs is taken during the important complication of insulin therapy is
first part of a meal for the purpose of delaying the hypoglycemia. The other choices are not common
absorption of dietary carbohydrates? effects of insulin. The answer is B.
3. of β-blocker-induced cardiac depression. The answer
Insulin regimens for tight control usually take the is D.
form of establishing a basal level of insulin with a QUESTIONS
small amount of a long-acting preparation (eg, 1. Which of the following drugs is routinely added to
insulin glargine) and supplementing the insulin calcium supplements and milk for the purpose of
levels, when called for by food intake, with short- preventing rickets in children and osteomalacia in
acting insulin lispro. Less tight control may be adults?
achieved with 2 injections of intermediate-acting (A) Cholecalciferol
insulin per day. Because intake of glucose is mainly (B) Calcitriol
during the day, long-acting insulins are usually given (C) Gallium nitrate
in the morning, not at night. The answer is D. (D) Sevelamer
4. (E) Plicamycin
Glipizide is a second-generation sulfonylurea that 2. Which of the following drugs is most useful for the
promotes insulin release by closing potassium treatment of hypercalcemia in Paget’s disease?
channels in pancreatic B cells. The answer is C. (A) Fluoride
5. (B) Hydrochlorothiazide
Insulin lowers serum glucose concentration in part (C) Pamidronate
by driving glucose into cells, particularly into muscle (D) Raloxifene
cells. The answer is C. (E) Teriparatide
6. 3. The active metabolites of vitamin D act through a
Biguanides, especially the older drug phenformin, nuclear receptor to produce which of the following
have been associated with lactic acidosis. Thus, effects?
metformin should be avoided or used with extreme (A) Decrease the absorption of calcium from bone
caution in patients with conditions that increase the (B) Increase PTH formation
risk of lactic acidosis, including acute ethanol (C) Increase renal production of erythropoietin
ingestion. The answer is D. (D) Increase the absorption of calcium from the
7. gastrointestinal tract
To be absorbed, carbohydrates must be converted (E) Lower the serum phosphate concentration
into monosaccharides by the action of α-glucosidase 4. A 59-year-old female was referred to your clinic
enzymes in the gastrointestinal tract. Acarbose for evaluation of osteopenia. She was diagnosed
inhibits α-glucosidase and, when present during with adult-onset cystic fibrosis (CF). She reported
digestion, delays the uptake of carbohydrates. The being treated with prednisone 2 times in the past for
answer is A. CF exacerbations. Since menopause at 52 years of
8. age, she had been treated with raloxifene for
The PPAR-γ receptor belongs to a family of nuclear osteoporosis prevention. She also was on daily
receptors. When activated, these receptors calcium and vitamin D supplementation. Her bone
translocate to the nucleus, where they regulate the mineral density test revealed a T score of –1.6 at the
transcription of genes encoding proteins involved in lumbar spine, –2.2 at the left femoral neck, and –1.6
the metabolism of carbohydrate and lipids. The at the total left hip. Which of the following drugs can
answer is D. be used to reduce the fracture risk by further
9. stimulating bone formation in this patient?
The insulin secretagogues, including the sulfonylurea (A) Cholecalciferol
glyburide, can cause hypoglycemia as a result of (B) Ergocalciferol
their ability to increase serum insulin levels. The (C) Furosemide
biguanides, thiazolidinediones, α-glucosidase (D) Tamoxifen
inhibitors, and canagliflozin are euglycemics that are (E) Teriparatide
unlikely to cause hypoglycemia when used alone. Questions 5–7. A 58-year-old postmenopausal
The answer is C. woman was sent for dual-energy x-ray
10. absorptiometry to evaluate the bone mineral density
Glucagon acts through cardiac glucagon receptors to of her lumbar spine, femoral neck, and total hip. The
stimulate the rate and force of contraction of the test results revealed significantly low bone mineral
heart. Because this bypasses cardiac β density in all sites.
adrenoceptors, glucagon is useful in the treatment
5. Chronic use of which of the following medications (C) Furosemide
is most likely to have contributed to this woman’s (D) Paricalcitol
osteoporosis? (E) Risedronate
(A) Lovastatin 10. In the treatment of patients like this with
(B) Metformin secondary hyperparathyroidism due to chronic
(C) Prednisone kidney disease, cinacalcet is an alternative to vitamin
(D) Propranolol D-based drugs. Cinacalcet lowers PTH by which of
(E) Thiazide diuretic the following mechanisms?
6. If this patient began oral therapy with (A) Activating a steroid receptor that inhibits
alendronate, she would be advised to drink large expression of the PTH gene
quantities of water with the tablets and remain in an (B) Activating the calcium-sensing receptor in
upright position for at least 30 min and until eating parathyroid cells
the first meal of the day. These instructions would (C) Activating transporters in the GI tract that are
be given to decrease the risk of which of the involved in calcium absorption
following? (D) Inducing the liver enzyme that converts vitamin
(A) Cholelithiasis D3 to 25-hydroxyvitamin D3
(B) Diarrhea (E) Inhibiting the farnesyl pyrophosphate synthase
(C) Constipation found in osteoclasts
(D) Erosive esophagitis ANSWERS
(E) Pernicious anemia 1.
7. The patient’s condition was not sufficiently The 2 forms of vitamin D—cholecalciferol and
controlled with alendronate, so she began therapy ergocalciferol—are commonly added to calcium
with a nasal spray containing a protein that inhibits supplements and dairy products. Calcitriol, the
bone resorption. The drug contained in the nasal active 1,25-dihydroxyvitamin D3 metabolite, would
spray was which of the following? prevent vitamin D deficiency and is available as an
(A) Calcitonin oral formulation. However, because it is not subject
(B) Calcitriol to the complex mechanisms that regulate
(C) Cinacalcet endogenous production of active vitamin D
(D) Cortisol metabolites, it is not suitable for widespread use.
(E) Teriparatide The answer is A.
Questions 8–10. A 67-year-old man with chronic 2.
kidney disease was found to have an elevated serum Paget’s disease is characterized by excessive bone
PTH concentration and a low serum concentration of resorption, poorly organized bone formation, and
25-hydroxyvitamin D. He was successfully treated hypercalcemia. Bisphosphonates and calcitonin are
with ergocalciferol. Unfortunately, his kidney disease first-line treatments. Pamidronate is a powerful
progressed so that he required dialysis and his bisphosphonate used parenterally to treat
serum PTH concentration became markedly hypercalcemia. The answer is C.
elevated. 3.
8. Which of the following drugs is most likely to The active metabolites of vitamin D increase serum
lower this patient’s serum PTH concentration? calcium and phosphate by promoting calcium and
(A) Calcitriol phosphate uptake from the gastrointestinal tract,
(B) Cholecalciferol increasing bone resorption, and decreasing renal
(C) Furosemide excretion of both electrolytes. They inhibit, rather
(D) Gallium nitrate than stimulate, PTH formation. The answer is D.
(E) Risedronate 4.
9. Although the drug therapy was effective at Cholecalciferol and ergocalciferol are precursors of
lowering serum PTH concentrations, the patient vitamin D. Furosemide is a loop diuretic, which
experienced several episodes of hypercalcemia. He causes increased calcium excretion; tamoxifen is a
was switched to a vitamin D analog that suppresses selective estrogen receptor modulator (SERM) but is
PTH with less risk of hypercalcemia. Which drug was less selective for bone compared with raloxifene.
the patient switched to? Teriparatide increases bone formation and bone
(A) Calcitriol resorption; during the first 6 months, it causes a net
(B) Cholecalciferol gain in bone. Teriparatide should not be used longer
than 2 yr due to risk of osteosarcoma. The answer is Questions 2 and 3. A 33-year-old man was seen in a
E. clinic with a complaint of dysuria and urethral
5. discharge of yellow pus. He had a painless clean-
Long-term therapy with glucocorticoids such as based ulcer on the penis and nontender
prednisone is associated with a reduction in bone enlargement of the regional lymph nodes. Gram
mineral density and an increased risk of fractures. stain of the urethral exudate showed gram-negative
The other drugs are not known to have significant diplococci within polymorphonucleocytes. The
effects on bone or serum calcium. The answer is C. patient informed the clinic staff that he was
6. unemployed and had not eaten a meal for 2 d.
Oral bisphosphonates such as alendronate can 2. The most appropriate treatment of gonorrhea in
irritate the esophagus and stomach. The risk of this this patient is
toxicity is reduced by drinking water and by (A) A single intramuscular dose of ceftriaxone
remaining in an upright position for 30 min after (B) Amoxicillin orally for 7 d
taking the medication. The answer is D. (C) Procaine penicillin G intramuscularly as a single
7. dose plus oral probenecid
Calcitonin is a peptide hormone that prevents bone (D) Meropenem orally for 7 d
resorption. Salmon calcitonin is available as a nasal (E) Vancomycin intramuscularly as a single dose
spray or a parenteral form for injection. The answer 3. Immunofluorescent microscopic examination of
is A. fluid expressed from the penile chancre of this
8. patient revealed treponemes. Because he appears to
In patients with chronic kidney disease that requires be infected with Treponema pallidum, the best
dialysis, the impaired production of active vitamin D course of action would be to
metabolites compounded with elevated serum (A) Administer a single oral dose of fosfomycin
phosphate due to renal impairment leads to (B) Give no other antibiotics because drug treatment
secondary hyperparathyroidism. Administration of of gonorrhea provides coverage for incubating
the active vitamin D metabolite calcitriol acts directly syphilis
on the parathyroid to inhibit PTH production. (C) Inject intramuscular benzathine penicillin G
Cholecalciferol, a form of vitamin D, is not effective (D) Treat with oral tetracycline for 7 d
in patients with advanced renal disease who cannot (E) Treat with vancomycin
form adequate amounts of active vitamin D 4. Which of the following statements about beta-
metabolites. The answer is A. lactam antibiotics is false?
9. (A) Cephalexin and other first-generation
Paricalcitol is an analog of 1,25-dihyroxyvitamin D3 cephalosporins do not cross the blood-brain barrier
(calcitriol) that lowers serum PTH at doses that only (B) Ceftriaxone and nafcillin are both eliminated
rarely precipitate hypercalcemia. The molecular mainly via biliary secretion
basis of this selective action is poorly understood but (C) Instability of penicillins in gastric acid can limit
is of value in the management of their oral absorption
hyperparathyroidism and psoriasis. The answer is D. (D) Renal tubular reabsorption of amoxicillin is
10. inhibited by probenecid
Cinacalcet is a member of a novel class of drugs that (E) Ticarcillin has activity against several gram
activate the calcium-sensing receptor in parathyroid negative rods
cells. When this receptor is activated by cinacalcet or 5. A 36-year-old woman recently treated for
free ionized calcium, it activates a signaling pathway leukemia is admitted to the hospital with malaise,
that suppresses PTH synthesis and release. The chills, and high fever. Gram stain of blood reveals the
answer is B. presence of gram-negative bacilli. The initial
QUESTIONS diagnosis is bacteremia, and parenteral antibiotics
1. The primary mechanism of antibacterial action of are indicated. The records of the patient reveal that
the penicillins involves inhibition of she had a severe urticarial rash, hypotension, and
(A) Beta-lactamases respiratory difficulty after oral penicillin V about 6
(B) Cell membrane synthesis mo ago. The most appropriate drug regimen for
(C) N-acetylmuramic acid synthesis empiric treatment is
(D) Peptidoglycan cross-linking (A) Aztreonam
(E) Transglycosylation (B) Ceftriaxone
(C) Meropenem (C) Ceftriaxone
(D) Oxacillin (D) Ticarcillin
(E) Ticarcillin plus clavulanic acid (E) Vancomycin
Questions 6–8. A 52-year-old man (weight 70 kg) is 10. Which statement about vancomycin is accurate?
brought to the hospital emergency department in a (A) Active against methicillin-resistant staphylococci
confused and delirious state. He has had an elevated (B) Bacteriostatic
temperature for more than (C) Binds to PBPs
24 h, during which time he had complained of a (D) Hepatic metabolism
severe headache and had suffered from nausea and (E) Oral bioavailability
vomiting. Lumbar puncture reveals an elevated ANSWERS
opening pressure, and cerebrospinal fluid findings 1.
include elevated protein, decreased glucose, and Penicillins (and cephalosporins) bind to PBPs acting
increased neutrophils. Gram stain of a smear of at the transpeptidation stage of cell wall synthesis
cerebrospinal fluid reveals gram-positive diplococci, (the final step) to inhibit peptidoglycan cross-linking.
and a preliminary diagnosis is made of purulent The beta-lactam antibiotics also activate autolysins,
meningitis. The microbiology report informs you that which break down the bacterial cell wall. Synthesis
for approximately 15% of S pneumoniae isolates in of N-acetylmuramic acid is inhibited by fosfomycin.
the community, the minimal inhibitory Vancomycin inhibits transglycolase, preventing
concentration for penicillin G is elongation of peptidoglycan chains. The answer is D.
20 mcg/mL. 2.
6. Treatment of this patient should be initiated Treatments of choice for gonorrhea include a single
immediately with intravenous administration of dose of ceftriaxone (intramuscularly). Because of the
(A) Amoxicillin high incidence of beta-lactamase-producing
(B) Cephalexin gonococci, the use of penicillin G or amoxicillin is no
(C) Ceftriaxone plus vancomycin longer appropriate for gonorrhea. Similarly, many
(D) Nafcillin strains of gonococci are resistant to tetracyclines.
(E) Piperacillin Alternative drugs (not listed) for gonorrhea include
7. Resistance of pneumococci to penicillin G is due to cefixime, azithromycin (see Chapter 44) or
(A) Alterations in porin structure spectinomycin (see Chapter 45). The answer is A.
(B) Beta-lactamase production 3.
(C) Changes in chemical structure of target penicillin- This patient with gonorrhea also has primary
binding proteins syphilis. The penile chancre, the enlarged nontender
(D) Changes in the d-Ala-d-Ala building block of lymph nodes, and the microscopic identification of
peptidoglycan precursor treponemes in fluid expressed from the lesion are
(E) Decreased intracellular accumulation of penicillin essentials of diagnosis. Although a single dose of
G ceftriaxone may cure incubating syphilis, it cannot
8. If this patient had been 82-years-old and the Gram be relied on for treating primary syphilis. The most
stain of the smear of cerebrospinal fluid had appropriate course of action in this patient is to
revealed gram-positive rods resembling administer a single intramuscular injection of 2.4
diphtheroids, the antibiotic regimen for empiric million units of benzathine penicillin G. For penicillin-
treatment would include allergic patients, oral doxycycline or tetracycline for
(A) Ampicillin 15 d (not 7 d) is effective in most cases (see Chapter
(B) Cefoxitin 44). However, lack of compliance may be a problem
(C) Ceftriaxone with oral therapy. Fosfomycin and vancomycin have
(D) Fosfomycin no significant activity against spirochetes. The
(E) Vancomycin answer is C.
9. A patient needs antibiotic treatment for native 4.
valve, culture-positive infective enterococcal First- and second-generation cephalosporins are not
endocarditis. His medical history includes a severe effective in meningitis because they do not readily
anaphylactic reaction to penicillin G during the last enter the cerebrospinal fluid. The elimination half-
year. The best approach would be treatment with lives of many beta-lactam antibiotics are prolonged
(A) Amoxicillin-clavulanate by probenecid, which inhibits their proximal tubular
(B) Aztreonam secretion. The answer is D.
5. vancomycin, often with an aminoglycoside (eg,
Each of the drugs listed has activity against some gentamicin) for synergistic activity against
gram-negative bacilli. All penicillins should be enterococci. The answer is E.
avoided in patients with a history of allergic 10.
reactions to any individual penicillin drug. Vancomycin is a bactericidal glycoprotein. It inhibits
Cephalosporins should also be avoided in patients cell wall synthesis but does not bind to PBPs and is
who have had anaphylaxis or other severe not susceptible to beta-lactamases. Vancomycin is
hypersensitivity reactions after use of a penicillin. not absorbed after oral administration and is used by
There is partial cross-reactivity between penicillins this route in the treatment of colitis caused by C
and the carbapenems such as imipenem and difficile and staphylococci. It undergoes renal
meropenem, but no cross-reactivity between the elimination. Vancomycin is commonly considered
penicillins and aztreonam. The answer is A. the drug of first choice for parenteral use against
6. methicillin-resistant staphylococci. The answer is A
Pneumococcal isolates with a minimal inhibitory QUESTIONS
concentration for penicillin G of greater than 2 1. A 4-year-old child is brought to the hospital after
mcg/mL are highly resistant. Such strains are not ingesting pills that a parent had used for bacterial
killed by the concentrations of penicillin G or dysentery when traveling outside the United States.
ampicillin that can be achieved in the cerebrospinal The child has been vomiting for more than 24 h and
fluid. Nafcillin has minimal activity against penicillin- has had diarrhea with green stools. She is now
resistant pneumococci and piperacillin is mainly used lethargic with an ashen color. Other signs and
for infections caused by gram-negative rods. symptoms include hypothermia, hypotension, and
Cefotaxime and ceftriaxone are the most active abdominal distention. The drug most likely to be the
cephalosporins against penicillin-resistant cause of this problem is
pneumococci, and the addition of vancomycin is (A) Ampicillin
recommended in the case of highly resistant strains. (B) Azithromycin
The answer is C. (C) Chloramphenicol
7. (D) Doxycycline
Pneumococcal resistance to penicillins is due to (E) Erythromycin
changes in the chemical structures of the target 2. The mechanism of antibacterial action of
penicillin-binding proteins located in the bacterial tetracycline involves
cytoplasmic membrane. A similar mechanism (A) Antagonism of bacterial translocase activity
underlies the resistance of staphylococci to (B) Binding to a component of the 50S ribosomal
methicillin (MRSA strains). A structural alteration in subunit
the d-Ala-d-Ala component of the pentapeptide side (C) Inhibition of DNA-dependent RNA polymerase
chains of peptidoglycans is the basis for a (D) Interference with binding of aminoacyl-tRNA to
mechanism of resistance to vancomycin. The answer bacterial ribosomes
is C. (E) Selective inhibition of ribosomal peptidyl
8. transferases
Diphtheroid-like gram-positive rods in the 3. Clarithromycin and erythromycin have very similar
cerebrospinal fluid smear of an elderly patient are spectra of antimicrobial activity. The major
indicative of L monocytogenes. Listeria infections are advantage of clarithromycin is that it
more common in neonates, elderly patients, and (A) Does not inhibit hepatic drug-metabolizing
those who have been treated with enzymes
immunosuppressive agents. Treatment consists of (B) Eradicates mycoplasmal infections in a single
ampicillin with or without an aminoglycoside such as dose
gentamicin. Trimethoprim-sulfamethoxazole can (C) Has greater activity against H pylori
also be used (see Chapter 46). The answer is A. (D) Is active against methicillin-resistant strains of
9. staphylococci
In patients who have had a severe reaction to a (E) Is active against strains of streptococci that are
penicillin, it is inadvisable to administer a resistant to erythromycin
cephalosporin or a carbapenem such as meropenem. 4. The primary mechanism of resistance of gram-
Aztreonam has no significant activity against gram- positive organisms to macrolide antibiotics including
positive cocci, so the logical treatment in this case is erythromycin is
(A) Changes in the 30S ribosomal subunit least 6 cups of caffeinated coffee per day. The
(B) Decreased drug permeability of the cytoplasmic physician makes an initial diagnosis of community-
membrane acquired pneumonia.
(C) Formation of drug-inactivating acetyltransferases 7. Regarding the treatment of this patient, which of
(D) Formation of esterases that hydrolyze the the following drugs is most suitable?
lactone ring (A) Ampicillin
(E) Methylation of binding sites on the 50S ribosomal (B) Clindamycin
subunit (C) Doxycycline
5. A 26-year-old woman was treated for a suspected (D) Linezolid
chlamydial infection at a neighborhood clinic. She (E) Vancomycin
was given a prescription for oral doxycycline to be 8. If this patient were to be treated with
taken for 14 d. Three weeks later, she returned to erythromycin, she should
the clinic with a mucopurulent cervicitis. On (A) Avoid exposure to sunlight
questioning she admitted not having the (B) Avoid taking supplementary iron tablets
prescription filled. The best course of action at this (C) Decrease her intake of caffeinated beverages
point would be to (D) Have her plasma urea nitrogen or creatinine
(A) Delay drug treatment until the infecting checked before treatment
organism is identified (E) Temporarily stop taking loratadine
(B) Rewrite the original prescription for oral 9. A 5-d course of treatment for community-
doxycycline acquired pneumonia would be effective in this
(C) Treat her in the clinic with a single oral dose of patient with little risk of drug interactions if the drug
azithromycin prescribed were
(D) Treat her in the clinic with an intravenous dose (A) Azithromycin
of amoxicillin (B) Clindamycin
(E) Write a prescription for oral erythromycin for 10 (C) Doxycycline
d (D) Erythromycin
6. A 55-year-old patient with a prosthetic heart valve (E) Vancomycin
is to undergo a periodontal procedure involving 10. Concerning quinupristin-dalfopristin, which
scaling and root planing. Several years ago, the statement is accurate?
patient had a severe allergic reaction to procaine (A) Active in treatment of infections caused by E
penicillin G. Regarding prophylaxis against bacterial faecalis
endocarditis, which one of the following drugs taken (B) An effective drug in treatment of multidrug-
orally is most appropriate? resistant streptococcal infections
(A) Amoxicillin 10 min before the procedure (C) Bacteriostatic
(B) Clindamycin 1 h before the procedure (D) Hepatotoxicity has led to FDA drug alerts
(C) Erythromycin 1 h before the procedure and 4 h (E) Increase the activity of hepatic drug-metabolizing
after the procedure enzymes
(D) Vancomycin 15 min before the procedure ANSWERS
(E) No prophylaxis is needed because this patient is 1.
in the negligible risk category Chloramphenicol is commonly used outside the
Questions 7–9. A 24-year-old woman comes to a United States for treatment of bacillary dysentery.
clinic with complaints of dry cough, headache, fever, The drug causes a dose-dependent (reversible)
and malaise, which have lasted 3 or 4 d. She appears suppression of erythropoiesis. Although the gray
to have some respiratory difficulty, and chest baby syndrome was initially described in neonates, a
examination reveals rales but no other obvious signs similar syndrome has occurred with overdosage of
of pulmonary involvement. However, extensive chloramphenicol in older children and adults,
patchy infiltrates are seen on chest x-ray film. Gram especially those with hepatic dysfunction. The
stain of expectorated sputum fails to reveal any answer is C.
bacterial pathogens. The patient mentions that a 2.
colleague at work had similar symptoms to those she Tetracyclines inhibit bacterial protein synthesis by
is experiencing. The patient has no history of serious interfering with the binding of aminoacyl-tRNA
medical problems. She takes loratadine for allergies molecules to bacterial ribosomes. Peptidyl
and supplementary iron tablets, and she drinks at
transferase is inhibited by chloramphenicol. The It is often difficult to establish a definite cause of
answer is D. community-acquired pneumonia (CAP). More than
3. 80% of cases are caused by typical pathogens such
Clarithromycin can be administered less frequently as S pneumoniae, H influenzae, or M catarrhalis, and
than erythromycin, but it is not effective in single 15% are due to the nonzoonotic atypial pathogens
doses against susceptible organisms. Organisms such as Legionella species, Mycoplasma species, or C
resistant to erythromycin, including pneumococci pneumoniae. Currently, monotherapy coverage of
and methicillin-resistant staphylococci, are also both typical and atypical pathogens in CAP is
resistant to other macrolides. Drug interactions have preferred to double-drug therapy. Preferred initial
occurred with clarithromycin through its ability to therapy includes a macrolide, doxycycline, or a
inhibit cytochrome P450. Clarithromycin is more quinolone active against respiratory pathogens
active than erythromycin against M avium complex, (Chapter 46). Ampicillin, clindamycin, and
T gondii, and H pylori. The answer is C. vancomycin have low activity against atypical
4. pathogens in CAP. The answer is C.
Methylase production and methylation of the 8.
receptor site are established mechanisms of The inhibition of liver cytochrome P450 by
resistance of gram-positive organisms to macrolide erythromycin has led to serious drug interactions.
antibiotics. Such enzymes may be inducible by Although erythromycin does not inhibit loratadine
macrolides or constitutive; in the latter case, cross- metabolism, it does inhibit the CYP1A2 form of
resistance occurs between macrolides and cytochrome P450, which metabolizes
clindamycin. Increased expression of efflux pumps is methylxanthines.. Consequently, cardiac and/or CNS
also a mechanism of macrolide resistance. Esterase toxicity may occur with excessive ingestion of
formation is a mechanism of macrolide resistance caffeine. Unlike the tetracyclines, the oral absorption
seen in coliforms. The answer is E. of erythromycin is not affected by cations and the
5. drug does not cause photosensitivity. Because
Cervicitis or urethritis is often caused by C erythromycin undergoes biliary excretion, there is
trachomatis. Such infections may develop slowly little reason to assess renal function before
because of the long incubation period of chlamydial treatment. The answer is C.
infection. Treatment with oral doxycycline for 14 d 9.
(as originally prescribed) would have eradicated C Azithromycin has a half-life of more than 70 h, which
trachomatis and most other organisms commonly allows for once-daily dosing and a 5-d course of
associated with nongonococcal cervicitis or treatment for community-acquired pneumonia.
urethritis. Given the limited compliance of this Unlike other macrolides, azithromycin does not
patient, the best course of action would be the inhibit cytochrome P450 enzymes involved in drug
administration (in the clinic) of a single oral dose of metabolism. The answer is A.
azithromycin. The answer is C. 10.
6. Quinupristin-dalfopristin is bactericidal against many
This patient is in the high-risk category for bacterial drug-resistant gram-positive cocci, including
endocarditis and should receive prophylactic multidrug-resistant streptococci, MRSA, and
antibiotics before many dental procedures. The vancomycin-resistant enterococci. The
American Heart Association recommends that streptogramins have activity against E faecium (not E
clindamycin be used in patients allergic to penicillins. faecalis). The drugs are potent inhibitors of CYP3A4
Oral erythromycin is not recommended because it is and interfere with the metabolism of many other
no more effective than clindamycin and causes more drugs. The streptogramins are not hepatotoxic. The
gastrointestinal side effects. Intravenous vancomycin answer is B.
(not oral), sometimes with gentamicin, is QUESTIONS
recommended for prophylaxis in high-risk penicillin- 1. Regarding the mechanism of action of
allergic patients undergoing genitourinary and lower aminoglycosides, the drugs
gastrointestinal surgical procedures. Complete cross- (A) Are bacteriostatic
allergenicity must be assumed between individual (B) Bind to the 50S ribosomal subunit
penicillins. The answer is B. (C) Cause misreading of the code on the mRNA
7. template
(D) Inhibit peptidyl transferase
(E) Stabilize polysomes (C) 400 mg
2. A 72-kg patient with creatinine clearance of 80 (D) 600 mg
mL/min has a gram-negative infection. Amikacin is (E) 800 mg
administered intramuscularly at a dose of 5 mg/kg 6. A 76-year-old man is seen in a hospital emergency
every 8 h, and the patient begins to respond. After 2 department complaining of pain in and behind the
d, creatinine clearance declines to 40 mL/min. right ear. Physical examination shows edema of the
Assuming that no information is available about external otic canal with purulent exudate and
amikacin plasma levels, what would be the most weakness of the muscles on the right side of the
reasonable approach to management of the patient face. The patient informs the physician that he is a
at this point? diabetic. Gram stain of the exudate from the ear
(A) Administer 5 mg/kg every 12 h shows many polymorphonucleocytes and gram-
(B) Decrease the dosage to daily total of 200 mg negative rods, and samples are sent to the
(C) Decrease the dosage to 180 mg every 8 h microbiology laboratory for culture and drug
(D) Discontinue amikacin and switch to gentamicin susceptibility testing. A preliminary diagnosis is
(E) Maintain the patient on the present dosage and made of external otitis. At this point, which of the
test auditory function following is most appropriate?
3. All of the following statements about the clinical (A) Amikacin should be injected intramuscularly and
uses of the aminoglycosides are accurate except the patient should be sent home
(A) Effective in the treatment of infections caused by (B) Analgesics should be prescribed, but antibiotics
anaerobes such as Bacteroides fragilis should be withheld pending microbiological results
(B) Gentamycin is used with ampicillin for synergistic (C) Oral cefaclor should be prescribed together with
effects in the treatment of enterococcal endocarditis analgesics, and the patient should be sent home
(C) In the treatment of a hospital-acquired infection (D) The patient should be hospitalized and treatment
caused by Serratia marcescens, netilmicin is more started with imipenem-cilastatin
effective than streptomycin (E) The patient should be hospitalized and treatment
(D) Often used with cephalosporins in the empiric started with gentamicin plus ticarcillin
treatment of life-threatening bacterial infections 7. Regarding the toxicity of aminoglycosides which
(E) Owing to their polar nature, aminoglycosides are statement is accurate?
not absorbed after oral administration (A) Gentamicin and tobramycin are the least likely to
4. Which statement is accurate regarding the cause renal damage
antibacterial action of the aminoglycoside amikacin? (B) Ototoxicity due to amikacin and gentamicin
(A) Antibacterial activity is often reduced by the includes vestibular dysfunction, which is often
presence of an inhibitor of cell wall synthesis irreversible
(B) Antibacterial action is not concentration- (C) Ototoxicity is reduced if loop diuretics are used
dependent to facilitate the renal excretion of aminoglycoside
(C) Antibacterial action is time-dependent antibiotics
(D) Efficacy is directly proportional to the duration of (D) Reduced blood creatinine is an early sign of
time that the plasma level is greater than the aminoglycoside nephrotoxicity
minimal inhibitory concentration (E) Skin reactions are very rare following topical use
(E) The drug continues to exert antibacterial effects of neomycin
even after plasma levels decrease below detectable 8. This drug has characteristics almost identical to
levels those of gentamicin but has much weaker activity in
5. An adult patient (weight 80 kg) has bacteremia combination with penicillin against enterococci.
suspected to be due to a gram-negative rod. (A) Amikacin
Tobramycin is to be administered using a once-daily (B) Erythromycin
dosing regimen, and the loading dose must be (C) Netilmicin
calculated to achieve a peak plasma level of 20 mg/L. (D) Spectinomycin
Assume that the patient has normal renal function. (E) Tobramycin
Pharmacokinetic parameters of tobramycin in this 9. Your 23-year-old female patient is pregnant and
patient are as follows: Vd = 30 L; t1/2 = 3 h; CL = 80 has gonorrhea. The medical history includes
mL/min. What loading dose should be given? anaphylaxis following exposure to amoxicillin. The
(A) 100 mg most appropriate drug to use is
(B) 200 mg (A) Azithromycin
(B) Cefixime The loading dose of any drug is calculated by
(C) Ceftriaxone multiplying the desired plasma concentration (mg/L)
(D) Ciprofloxacin by the volume of distribution (L). The answer is D.
(E) Doxycycline 6.
10. Which statement about “once-daily” dosing with The diabetic patient with external otitis is at special
aminoglycosides is not accurate? risk because of the danger of spread to the middle
(A) Dose adjustment is less important in renal ear and possibly the meninges, so hospitalization is
dysfunction advisable, especially in the elderly. Likely pathogens
(B) It is convenient for outpatient treatment include E coli and Pseudomonas aeruginosa, and
(C) Less nursing time is required for drug coverage must be provided for these and possibly
administration other gram-negative rods. The combination of an
(D) Often less side effects than multiple aminoglycoside plus a wider spectrum penicillin is
(conventional) dosing regimens most suitable in this case and is synergistic against
(E) Underdosing is less of a problem many pseudomonas strains. Imipenem-cilastatin is
ANSWERS also possible, but resistant strains of P aeruginosa
1. have emerged during treatment. Cefaclor lacks
Aminoglycosides are bactericidal inhibitors of antipseudomonal activity. The answer is E.
protein synthesis binding to specific components of 7.
the 30S ribosomal subunit. Their actions include Gentamicin and tobramycin are the most
block of the formation of the initiation complex, nephrotoxic aminoglycosides. The incidence of
miscoding, and polysomal breakup. Peptidyl nephrotoxic effects with gentamicin is 2 to 3 times
transferase is inhibited by chloramphenicol, not greater than the incidence of ototoxicity. With
aminoglycosides. The answer is C. traditional dosage regimens, the first indication of
2. potential nephrotoxicity is an increase in trough
Monitoring plasma drug levels is important when serum levels of aminoglycosides, which is followed
aminoglycosides are used. In this case, the patient by an increase in blood creatinine. Although
seems to be improving, so a decrease of the aminoglycoside ototoxicity usually involves
amikacin dose in proportion to decreased creatinine irreversible effects on vestibular function, hearing
clearance is most appropriate. Because creatinine loss can also occur. Ototoxicity is enhanced by loop
clearance is only one half of the starting value, a diuretics. Skin reactions are common with topical
dose reduction should be made to one half of that use of neomycin. The answer is B.
given initially. The answer is C. 8.
3. Tobramycin is almost identical to gentamicin in both
Aminoglycoside antibiotics act at the ribosomal level its pharmacodynamic and pharmacokinetic
and their intracellular accumulation by bacteria is properties. However, for reasons that are unclear, it
oxygen dependent. Anaerobic bacteria including B is much less active than either gentamicin or
fragilis are innately resistant. The answer is A. streptomycin when used in combination with a
4. penicillin in the treatment of enterococcal
The antibacterial action of aminoglycosides is endocarditis. The answer is E.
concentration dependent rather than time 9.
dependent. The activity of amikacin continues to All of the listed drugs have been used for the
increase as its plasma level rises above the minimal treatment of gonorrhea. Cephalosporins should be
inhibitory concentration (MIC). When theplasma avoided in patients with a history of severe
level falls below the MIC, aminoglycosides continue hypersensitivity to penicillins, and fluoroquinolones
to exert antibacterial effects for several hours, (see Chapter 46) should be avoided in pregnancy.
exerting a post-antibiotic effect. Inhibitors of Tetracyclines including doxycycline have been used
bacterial cell wall synthesis often exert synergistic in the past for gonorrhea, but not as single doses,
effects with aminoglycosides, possibly by increasing and they too should be avoided in pregnancy. The
the intracellular accumulation of the answer is A.
aminoglycoside. The answer is E. 10.
5. In “once-daily dosing” with aminoglycosides, the
selection of an appropriate dose is particularly
critical in patients with renal insufficiency. The
aminoglycosides are eliminated by the kidney in 5. Which statement about the fluoroquinolones is
proportion to creatinine clearance. Knowledge of the accurate?
degree of insufficiency, based on plasma creatinine (A) Antacids increase their oral bioavailability
(or BUN), is essential for estimation of the (B) Contraindicated in patients with hepatic
appropriate single daily dose of an aminoglycoside. dysfunction
The answer is A. (C) Fluoroquinolones are drugs of choice in a 6-year-
QUESTIONS old child with a urinary tract
1. Trimethoprim-sulfamethoxazole is established to (D) Gonococcal resistance to fluoroquinolones may
be effective against which of the following involve changes in DNA gyrase
opportunistic infections in the AIDS patient? (E) Modification of moxifloxacin dosage is required
(A) Cryptococcal meningitis in patients when creatinine clearance is less than 50
(B) Herpes simplex mL/min
(C) Oral candidiasis 6. A 40-year-old man complains of periodic bouts of
(D) Toxoplasmosis diarrhea with lower abdominal cramping and
(E) Tuberculosis intermittent rectal bleeding. Seen in the clinic, he
2. A 65-year-old woman has returned from a appears well nourished, with blood pressure in the
vacation abroad suffering from traveler’s diarrhea, normal range. Examination reveals moderate
and her problem has not responded to antidiarrheal abdominal pain and tenderness. His current
drugs. A pathogenic gram-negative bacillus is medications are limited to loperamide for his
suspected. Which drug is most likely to be effective diarrhea. Sigmoidoscopy reveals mucosal edema,
in the treatment of this patient? friability, and some pus. Laboratory findings include
(A) Ampicillin mild anemia and decreased serum albumin.
(B) Ofloxacin Microbiologic examination via stool cultures and
(C) Sulfadiazine mucosal biopsies do not reveal any evidence for
(D) Trimethoprim bacterial, amebic, or cytomegalovirus involvement.
(E) Vancomycin The most appropriate drug to use in this patient is
3. Which statement about the clinical use of (A) Ampicillin
sulfonamides is false? (B) Doxycycline
(A) Active against C trachomatis and can be used (C) Norfloxacin
topically for treatment of chlamydial infections of (D) Sulfasalazine
the eye (E) Trimethoprim-sulfamethoxazole
(B) Are not effective as sole agents in the treatment 7. Which adverse effect is most common with
of prostatitis sulfonamides?
(C) Effective in Rocky Mountain spotted fever (A) Fanconi’s aminoaciduria syndrome
(D) In some bacterial strains resistance occurs via (B) Hematuria
increased PABA formation (C) Kernicterus in the newborn
(E) Reduced intracellular uptake is a mechanism of (D) Neurologic dysfunction
sulfonamide resistance in some bacterial strains (E) Skin rash
4. A 31-year-old man has gonorrhea. He has no drug 8. Which drug is effective in the treatment of
allergies, but a few years ago acute hemolysis nocardiosis and, in combination with
followed use of an antimalarial drug. The physician is pyrimethamine, is prophylactic against Pneumocystis
concerned that the patient has an accompanying jirovecii infections in AIDS patients?
urethritis caused by C trachomatis, although no (A) Amoxicillin
cultures or enzyme tests have been performed. (B) Erythromycin
Which of the following drugs will be reliably effective (C) Levofloxacin
against both gonococci and C trachomatis and safe (D) Sulfadiazine
to use in this patient? (E) Trimethoprim
(A) Cefixime 9. Which statement about ciprofloxacin is accurate?
(B) Ciprofloxacin (A) Antagonism occurs if used with dihydrofolate
(C) Spectinomycin reductase inhibitors
(D) Sulfamethoxazole-trimethoprim (B) Ciprofloxacin is active against MRSA strains of
(E) None of the above staphylococci
(C) Most “first-time” urinary tract infections are resistance is now common. This patient could be
resistant to ciprofloxacin treated by single oral doses of cefixime plus
(D) Organisms that commonly cause ear infections azithromycin (not listed). Sulfamethoxazole or TMP-
are highly resistant SMZ would not be useful and may cause acute
(E) Tendinitis may occur during treatment hemolysis in this patient. The answer is E.
10. Supplementary folinic acid may prevent anemia 5.
in folate-deficient persons who use this drug; it is a Antacids can decrease oral bioavailability of
weak base achieving tissue levels similar to those in fluoroquinolones. Neither hepatic or renal
plasma dysfunction is a contraindication to the use of
(A) Ciprofloxacin fluoroquinolones. Most fluoroquinolones undergo
(B) Levofloxacin renal elimination and dosage should be modified
(C) Linezolid with creatinine clearance <50 mL/min. Moxifloxacin
(D) Sulfamethoxazole elimination occurs mainly via the liver. The
(E) Trimethoprim fluoroquinolones should not be used to treat
ANSWERS uncomplicated first-time urinary tract infections in
1. children because of possible effects on cartilage
Trimethoprim-sulfamethoxazole is not effective in development. Uncomplicated urinary tract infections
the treatment of infections caused by viruses, fungi, in children are usually due to a strain of E coli that is
or mycobacteria. However, the drug combination is sensitive to many other drugs, including beta-lactam
active against certain protozoans, including antibiotics. The answer is D.
Toxoplasma, and can be used for both prevention 6.
and treatment of toxoplasmosis in the severely In the absence of any evidence pointing toward a
immunocompromised AIDS patient. The answer is D. definite microbial cause for the colitis in this patient,
2. a drug that decreases inflammation is indicated.
The second-generation fluoroquinolones are very Sulfasalazine has significant anti-inflammatory
effective in diarrhea caused by bacterial gram- action, and its oral use results in symptomatic
negative pathogens, including E coli, Shigella, and improvement in 50–75% of patients suffering from
Salmonella. None of the other drugs listed would be ulcerative colitis. The drug is also used for its anti-
appropriate. Many coliforms are now resistant to inflammatory effects in rheumatoid arthritis. The
amoxicillin and ampicillin. Although trimethoprim is answer is D.
available as a single drug, resistance emerges rapidly 7.
during treatment unless it is used for urinary tract The most common adverse effect of the
infections, in which high concentrations can be sulfonamides is a skin rash caused by
achieved. Vancomycin has no activity against gram- hypersensitivity. Neurologic dysfunction and
negative bacilli. The answer is B. hematuria occur less frequently. Sulfonamides are
3. usually avoided in the third trimester of pregnancy
Sulfonamides have minimal therapeutic actions in or in neonates, so kernicterus is rare. Fanconi’s
rickettsial infections. Chloramphenicol may be used syndrome is associated with the use of outdated
for Rocky Mountain spotted fever in patients with tetracyclines. The answer is E.
established allergy or other contraindication to 8.
tetracyclines. All of the other statements about Sulfadiazine and TMP-SMZ are drugs of choice in
sulfonamide antimicrobial drugs are accurate. The nocardiosis. In combination with pyrimethamine (an
answer is C. effective dihydrofolate reductase inhibitor in
4. protozoa), sulfadiazine is effective in toxoplasmosis
Cefixime in a single oral dose is effective in and is prophylactic against pneumocystis pneumonia
gonorrhea (Chapter 43), but it has no activity against in the AIDS patient. However, TMP-SMZ is more
organisms causing nongonococcal urethritis. commonly used for the latter purpose. The answer is
Spectinomycin (Chapter 45) is active against most D.
gonococci, but does not eradicate a urogenital 9.
chlamydial infection. Although ciprofloxacin might Ciprofloxacin is commonly used for the treatment of
be effective in both gonorrhea and chlamydial urinary tract infections and is active against most
urethritis, it is no longer recommended for strains of common causative agents of otitis media,
treatment of gonorrhea in the United States, since including H influenzae and pneumococci. However,
up to 50% of strains of MRSA are now resistant to 2. At this point, the most appropriate course of
ciprofloxacin. No clinical antagonism has been action is to
reported between fluoroquinolones and inhibitors of (A) Hospitalize the patient and start treatment with
folic acid synthesis. Fluoroquinolones are not 4 antitubercular drugs
recommended for use in pregnancy or for children (B) Hospitalize the patient and start treatment with
less than 10 years of age because they may damage rifampin
growing cartilage. Tendonitis and tendon rupture are (C) Prescribe isoniazid for prophylaxis and send the
adverse effects of the fluoroquinolones. The answer patient home to await culture results
is E. (D) Provide no drugs and send the patient home to
10. await culture results
Trimethoprim is the only weak base listed (E) Treat the patient with isoniazid plus rifampin
(fluoroquinolones and sulfonamides are acidic 3. Which drug regimen should be initiated in this
compounds), and its high lipid solubility at blood pH patient when treatment is started?
allows penetration of the drug into prostatic and (A) Amikacin, isoniazid, pyrazinamide, streptomycin
vaginal fluid to reach levels similar to those in (B) Ciprofloxacin, cycloserine, isoniazid, PAS
plasma. Leukopenia and thrombocytopenia may (C) Ethambutol, isoniazid, pyrazinamide, rifampin
occur in folate deficiency when the drug is used (D) Isoniazid, pyrazinamide, rifampin, streptomycin
alone or in combination with sulfamethoxazole. (E) PAS, pyrazinamide, rifabutin, streptomycin
Fluoroquinolones and linezolid do not exacerbate 4. Which statement concerning the possible use of
symptoms of folic acid deficiency. The answer is E. isoniazid (INH) in this patient is false?
QUESTIONS (A) Dyspnea, flushing, palpitations, and sweating
1. The primary reason for the use of drug may occur after ingestion of tyramine-containing
combinations in the treatment of tuberculosis is to foods
(A) Delay or prevent the emergence of resistance (B) In patients from Southeast Asia, lower
(B) Ensure patient compliance with the drug regimen maintenance doses are necessary
(C) Increase antibacterial activity synergistically (C) Peripheral neuritis may occur during treatment
(D) Provide prophylaxis against other bacterial (D) The patient should take pyridoxine daily
infections (E) The risk of the patient developing hepatitis from
(E) Reduce the incidence of adverse effects INH is less than 2%
Questions 2–5. A 21-year-old woman from 5. On her release from the hospital, the patient is
Southeast Asia has been staying with family advised not to rely solely on oral contraceptives to
members in the United States for the last 3 mo and prevent pregnancy because they may be less
is looking after her sister’s preschool children during effective while she is being maintained on
the day. Because she has difficulty with the English antimycobacterial drugs. The agent most likely to
language, her sister escorts her to the emergency interfere with the action of oral contraceptives is
department of a local hospital. She tells the staff (A) Amikacin
that her sister has been feeling very tired for the last (B) Ethambutol
month, has a poor appetite, and has lost weight. The (C) Isoniazid
patient has been feeling somewhat better lately (D) Pyrazinamide
except for a cough that produces a greenish sputum, (E) Rifampin
sometimes specked with blood. With the exception 6. A patient with AIDS and a CD4 cell count of 100/μL
of rales in the left upper lobe, the physical has persistent fever and weight loss associated with
examination is unremarkable and she does not seem invasive pulmonary disease due to M avium complex
to be acutely ill. Laboratory values show a white (MAC). Optimal management of this patient is to
count of 12,000/μL and a hematocrit of 33%. Chest (A) Choose an antibiotic based on drug susceptibility
x-ray film reveals an infiltrate in the left upper lobe of the cultured organism
with a possible cavity. A Gram-stained smear of the (B) Initiate a two-drug regimen of INH and
sputum shows mixed flora with no dominance. An pyrazinamide
acid-fast stain reveals many thin rods of pinkish hue. (C) Prescribe rifabutin because it prevents the
A preliminary diagnosis is made of pulmonary development of MAC bacteremia
tuberculosis. Sputum is sent to the laboratory for (D) Start treatment with the combination of
culture. azithromycin, ethambutol, and rifabutin
(E) Treat with trimethoprim-sulfamethoxazole
7. A 10-year-old boy has uncomplicated pulmonary Despite the fact that this patient does not appear to
tuberculosis. After initial hospitalization, he is now be acutely ill, she would in most cases be treated
being treated at home with isoniazid, rifampin, and with 4 drugs that have activity against M
ethambutol. Which statement about this case is tuberculosis. This is because organisms infecting
accurate? patients from Southeast Asia are commonly INH-
(A) A baseline test of auditory function test is resistant, and coverage must be provided with 3
essential before drug treatment is initiated other antituberculosis drugs in addition to isoniazid.
(B) His mother, who takes care of him, does not This patient should be hospitalized for several
need INH prophylaxis reasons, including potential difficulties with
(C) His 3-year-old sibling should receive INH compliance regarding the drug regimen and the fact
prophylaxis that young children are in the home where she is
(D) Polyarthralgia is a potential adverse effect of the living. The answer is A.
drugs the boy is taking 3.
(E) The potential nephrotoxicity of the prescribed Sputum cultures will not be available for several
drugs warrants periodic assessment of renal function weeks, and no information is available regarding
8. Which statement about antitubercular drugs is drug susceptibility of the organism at this stage. For
accurate? optimum coverage, the initial regimen should
(A) Antimycobacterial actions of streptomycin include INH, rifampin, pyrazinamide, and
involve inhibition of arabinosyltransferases ethambutol. INH-resistant organisms are usually
(B) Cross-resistance of M tuberculosis to isoniazid sensitive to both rifampin and pyrazinamide.
and pyrazinamide is common Streptomycin is usually reserved for use in severe
(C) Ocular toxicity of ethambutol is prevented by forms of tuberculosis or for infections known to be
thiamine resistant to first-line drugs. Likewise, amikacin and
(D) Pyrazinamide treatment should be discontinued ciprofloxacin are possible agents for treatment of
immediately if hyperuricemia occurs multidrug-resistant strains of M tuberculosis.
(E) Resistance to ethambutol involves mutations in Cycloserine, PAS, and rifabutin are alternative
the emb gene second-line drugs that may be used in cases of failed
9. Once-weekly administration of which of the response to more conventional agents. The answer
following antibiotics has prophylactic activity against is C.
bacteremia caused by M avium complex in AIDS 4.
patients? Patients from Pacific Rim countries do not require
(A) Acedapsone lower doses of INH! Fast acetylators, including
(B) Azithromycin Native Americans, may require higher doses of the
(C) Clarithromycin drug than others. Peripheral neuropathy caused by
(D) Kanamycin INH is due to pyridoxine deficiency. It is more
(E) Rifabutin common in the diabetic, malnourished, or AIDS
10. Risk factors for multidrug-resistant tuberculosis patient and can be prevented by a daily dose of 25–
include 50 mg of pyridoxine. INH can inhibit monoamine
(A) A history of treatment of tuberculosis without oxidase type A and has caused tyramine reactions.
rifampin Hepatotoxicity is age-dependent, with an incidence
(B) Recent immigration from Asia and living in an of 0.3% in patients aged 21–35 yr and greater than
area of over 4% isoniazid resistance 2% in patients older than 50 yr. The answer is B.
(C) Recent immigration from Latin America 5.
(D) Residence in regions where isoniazid resistance is Rifampin induces the formation of several
known to exceed 4% microsomal drug-metabolizing enzymes, including
(E) All of the above cytochrome P450 isoforms. This action increases the
ANSWERS rate of elimination of a number of drugs, including
1. anticoagulants, ketoconazole, methadone, and
Although it is sometimes possible to achieve steroids that are present in oral contraceptives. The
synergistic effects against mycobacteria with drug pharmacologic activity of these drugs can be
combinations, the primary reason for their use is to reduced markedly in patients taking rifampin. The
delay the emergence of resistance. The answer is A. answer is E.
2. 6.
Combinations of antibiotics are essential for shown to be active in vitro against the infecting
suppression of disease caused by M avium complex strain. The appropriate duration of therapy has not
in the AIDS patient, and treatment should be started been established. The answer is E.
before culture results are available. Although QUESTIONS
rifabutin is prophylactic against MAC bacteremia 1. Interactions between this drug and cell membrane
when it is used as sole therapy in active disease, components can result in the formation of pores
resistant strains of the organism emerge rapidly. lined by hydrophilic groups present in the drug
MAC is much less susceptible than M tuberculosis to molecule.
conventional antimycobacterial drugs. Currently, the (A) Caspofungin
optimum regimen consists of azithromycin (or (B) Flucytosine
clarithromycin) with ethambutol and rifabutin. The (C) Griseofulvin
answer is D. (D) Nystatin
7. (E) Terbinafine
A baseline test of ocular (not auditory) function may 2. Which statement about fluconazole is accurate?
be useful before starting ethambutol. None of the (A) Does not penetrate the blood-brain barrier
drugs prescribed is associated with nephrotoxicity. (B) Drug of choice in treatment of aspergillosis
Polyarthralgia is a common adverse effect of (C) Induces hepatic drug-metabolizing enzymes
pyrazinamide that was not prescribed in this case. (D) Has the least effect of all azoles on drug
Periodic tests of liver function may be advisable in metabolism
younger patients who are treated with INH plus (E) Oral bioavailability is less than that of
rifampin, especially if higher doses of these drugs are ketoconazole
used. Prophylaxis with INH is advisable for all Questions 3–5. A 37-year-old woman with leukemia
household members and very close contacts of was undergoing chemotherapy with intravenous
patients with active tuberculosis, especially young antineoplastic drugs. During treatment, she
children. The answer is C. developed a systemic infection from an
8. opportunistic pathogen. There was no erythema or
Arabinosyltransferase is inhibited by ethambutol edema at the catheter insertion site. A white vaginal
(not streptomycin) and resistance involves discharge was observed. After appropriate
alterations in the emb gene. Ocular adverse effects specimens were obtained for culture, empiric
of ethambutol are dose-dependent and usually antibiotic therapy was started with gentamicin,
reversible when the drug is discontinued. Thiamine nafcillin, and ticarcillin intravenously. This regimen
is not protective. There is minimal cross-resistance was maintained for 72 h, during which time the
between pyrazinamide and other antimycobacterial patient’s condition did not improve significantly. Her
drugs. Pyrazinamide uniformly causes throat was sore, and white plaques had appeared in
hyperuricemia, but this is not a reason to halt her pharynx. On day 4, none of the cultures had
therapy even though the drug may provoke gouty shown any bacterial growth, but both the blood and
arthritis in susceptible persons. The answer is E. urine cultures grew out Candida albicans.
9. 3. At this point, the best course of action is to
Because of its long elimination half-life (3–4 d), (A) Continue current antibiotics and start
weekly administration of azithromycin has proved to griseofulvin
be equivalent to daily administration of (B) Continue current antibiotics and start
clarithromycin when used for prophylaxis against M amphotericin B
avium complex in AIDS patients. Acedapsone is a (C) Stop current antibiotics and start itraconazole
repository form of dapsone used in leprosy. The (D) Stop current antibiotics and start amphotericin B
answer is B. (E) Stop current antibiotics and start terbinafine
10. 4. If amphotericin B is administered, the patient
Multidrug-resistant tuberculosis (MDR-TB) is defined should be premedicated with
as resistance to 2 or more drugs. All the risk factors (A) Diphenhydramine
are relevent. In the case of resistance to both INH (B) Ibuprofen
and rifampin, initial regimens still include both (C) Prednisone
drugs, plus ethambutol, pyrazinamide, streptomycin (D) Any or all of the above
(or other aminoglycoside), and a fluoroquinolone. (E) None of the above
Continuation therapy should include at least 3 drugs
5. Candida is a major cause of nosocomial (B) Less expensive to use than conventional
bloodstream infection. The opportunistic fungal amphotericin B
infection in this patient could have been prevented (C) More effective in fungal infections because they
by administration of increase tissue uptake of amphotericin B
(A) Caspofungin (D) They decrease the nephrotoxicity of
(B) Flucytosine amphotericin B
(C) Nystatin (E) They have a wider spectrum of antifungal activity
(D) Voriconazole than conventional formulations of amphotericin B
(E) None of the above ANSWERS
Questions 6–7. A 28-year-old man living on the East 1.
Coast was transferred by his employer to California The polyene antifungal drugs amphotericin B and
for several months. On his return, he complains of nystatin are amphipathic molecules that can interact
having influenza-like symptoms with fever and a with ergosterol in fungal cell membranes to form
cough. He also has red, tender nodules on his shins. artificial pores. In these structures, the lipophilic
His physician suspects that these symptoms are due groups on the drug molecule are arranged on the
to coccidioidomycosis contracted during his stay in outside of the pore, and the hydrophilic regions are
California. located on the inside. The fungicidal action of the
6. This patient should be treated immediately with polyenes derives from this interaction, which results
(A) Amphotericin B in leakage of intracellular constituents. The answer is
(B) Caspofungin D.
(C) Ketoconazole 2.
(D) Terbinafine The azoles with activity against Aspergillus are
(E) None of these drugs itraconazole and voriconazole. Fluconazole is the
7. Which is the drug of choice if this patient is best absorbed member of the azole group by the
suffering from persistent lung lesions or oral route and the only one that readily penetrates
disseminated disease caused by Coccidioides into cerebrospinal fluid. Although fluconazole may
immitis? inhibit the metabolism of some drugs, it has the
(A) Amphotericin B least effect of all azoles on hepatic microsomal drug-
(B) Flucytosine metabolizing enzymes. The answer is D.
(C) Itraconazole 3.
(D) Micofungin The antibiotic regimen should be stopped
(E) Terbinafine immediately, since the condition of the patient did
8. Which drug is least likely to be effective in the not improve after 3 d of such treatment, the cultures
treatment of esophageal candidiasis if it is used by were negative for bacteria, and the clinical picture
the oral route? suggested that the patient had a fungal infection.
(A) Clotrimazole This was subsequently confirmed by blood culture.
(B) Griseofulvin The answer is D.
(C) Ketoconazole 4.
(D) Itraconazole Infusion-related adverse effects of amphotericin B
(E) Nystatin include chills and fevers (the “shake and bake”
9. Serious cardiac effects have occurred when this syndrome), muscle spasms, nausea, headache, and
drug was taken by patients using the antihistamines hypotension. Analgesic-antipyretics, antihistamines,
astemizole or terfenadine and glucocorticoids all have been shown to be
(A) Amphotericin B helpful. The administration of a 1-mg test dose of
(B) Griseofulvin amphotericin B is sometimes useful in predicting the
(C) Ketoconazole severity of infusion-related toxicity. The answer is D.
(D) Terbinafine 5.
(E) Voriconazole In the case of opportunistic candidal infections in the
10. Regarding the clinical use of liposomal immunocompromised patient, no prophylactic drugs
formulations of amphotericin B, which statement is have been shown to be clinically effective.
accurate? Prophylaxis against other fungi may be effective in
(A) Amphotericin B affinity for these lipids is greater some instances, including suppression of
than affinity for ergosterol cryptococcal meningitis in AIDS patients with
fluconazole. However, prophylactic use of azoles QUESTIONS
may contribute to the development of fungal 1. Which statement about the mechanisms of action
resistance. The answer is E. of antiviral drugs is accurate?
6. (A) Acyclovir has no requirement for activation by
A travel history can be important in the diagnosis of phosphorylation
fungal disease. If this patient has a fungal infection (B) Ganciclovir inhibits viral DNA polymerase but
of the lungs, it is probably due to C immitis, which is does not cause chain termination
endemic in dry regions of the western United States. (C) Increased activity of host cell ribonucleases that
Pulmonary symptoms of coccidioidomycosis are degrade viral mRNA is one of the actions of
usually self-limiting, and drug therapy is not interferon-α
commonly required in an otherwise healthy patient. (D) The initial step in activation of foscarnet in HSV-
Tender red nodules on extensor surfaces constitute infected cells is its phosphorylation by thymidine
a good prognostic sign. Erythema nodosum is a kinase
delayed hypersensitivity response to fungal antigens. (E) The reverse transcriptase of HIV is 30–50 times
No organisms are present in the lesions, and it is not more sensitive to inhibition by fosamprenavir than
a sign of disseminated disease. The answer is E. host cell DNA polymerases
7. Questions 2 and 3. A 30-year-old male patient who
In progressive or disseminated forms of is HIV-positive and symptomatic has a CD4 count of
coccidioidomycosis, systemic antifungal drug 250/μL and a viral RNA load of 15,000 copies/mL. His
treatment is needed. Until recently, amphotericin B treatment involves a 3-drug antiviral regimen
was the recommended therapy, but fluconazole or consisting of zidovudine, didanosine, and ritonavir.
itraconazole are now generally preferred. Note that The patient is taking acyclovir for a herpes infection
the risk of dissemination is much greater in African and ketoconazole for oral candidiasis. He now
Americans (10% incidence) and in pregnant women complains of anorexia, nausea and vomiting, and
during the third trimester. The answer is C. abdominal pain. His abdomen is tender in the
8. epigastric area. Laboratory results reveal an amylase
Griseofulvin has no activity against C albicans and is activity of 220 U/L, and a preliminary diagnosis is
not effective in the treatment of systemic or made of acute pancreatitis.
superficial infections caused by such organisms. 2. If this patient has acute pancreatitis, the drug
“Swish and swallow” formulations of clotrimazole most likely to be responsible is
and nystatin have been used commonly. Most of the (A) Acyclovir
azoles are effective in esophageal candidiasis. The (B) Didanosine
answer is B. (C) Ketoconazole
9. (D) Ritonavir
Ketoconazole was the first oral azole introduced into (E) Zidovudine
clinical use, but it has a greater propensity to inhibit 3. In the further treatment of this patient, the drug
human cytochrome P450 enzymes than other azoles causing the pancreatitis should be withdrawn and
and is no longer widely used in the United States. replaced by
Cardiotoxicity may occur when ketoconazole is used (A) Atazanavir
by patients taking astemizole or terfenadine as a (B) Cidofovir
result of the ability of ketoconazole to inhibit their (C) Foscarnet
metabolism via hepatic cytochromes P450. The (D) Lamivudine
answer is C. (E) Ribavirin
10. 4. In an accidental needlestick, an unknown quantity
Liposomal formulations of amphotericin B result in of blood from an AIDS patient is injected into a
decreased accumulation of the drug in tissues, resident physician. The most recent laboratory
including the kidney. As a result, nephrotoxicity is report on the AIDS patient shows a CD4 count of
decreased. With some lipid formulations, infusion- 20/μL and a viral RNA load of greater than 107
related toxicity may also be reduced. Lipid copies/mL. The most appropriate course of action
formulations do not have a wider antifungal regarding treatment of the resident is to
spectrum; their daily cost ranges from 10 to 40 times (A) Determine whether HIV transmission has
more than the conventional formulation of occurred by monitoring the patient’s blood
amphotericin B. The answer is D. (B) Treat with a single high dose of zidovudine
(C) Treat with full doses of zidovudine for 4 wk (A) At the start of treatment, most patients
(D) Treat with single doses of zidovudine and experience flu-like symptoms
indinavir (B) Indications include treatment of genital warts
(E) Treat with zidovudine plus lamivudine plus (C) It is used in the management of hepatitis B and C
ritonavir for 4 wk (D) Lamivudine interferes with its activity against
Questions 5 and 6. A patient with AIDS has a CD4 hepatitis B
count of 45/μL. He is being maintained on a 3-drug (E) Toxicity includes bone marrow suppression
regimen of indinavir, didanosine, and zidovudine. 10. More than 90% of this drug is excreted in the
For prophylaxis against opportunistic infections, he is urine in intact form. Because its urinary solubility is
also receiving cidofovir, fluconazole, rifabutin, and low, patients should be well hydrated to prevent
trimethoprim-sulfamethoxazole. nephrotoxicity. Which drug is described?
5. The drug most likely to suppress herpetic (A) Acyclovir
infections and provide prophylaxis against CMV (B) Efavirenz
retinitis in this patient is (C) Indinavir
(A) Fluconazole (D) Trifluridine
(B) Cidofovir (E) Zidovudine
(C) Indinavir ANSWERS
(D) Rifabutin 1.
(E) Trimethoprim-sulfamethoxazole Acyclovir is activated by host cell kinases. Like
6. The dose of indinavir in this patient may need to acyclovir, ganciclovir inhibits viral DNA polymerase
be increased above normal. This is because and causes chain termination. However, foscarnet
(A) Fluconazole slows gastric emptying inhibits viral DNA polymerase without requiring
(B) Ganciclovir increases the renal clearance of bioactivation. Fosamprenavir is the prodrug of
indinavir amprenavir, an inhibitor of HIV protease; it has no
(C) Gastric absorption is inhibited by fluconazole significant effect on reverse transcriptase. The
(D) Rifabutin increases hepatic drug metabolism answer is C.
(E) Sulfamethoxazole increases indinavir plasma 2.
protein binding Gastrointestinal problems occur with most antiviral
7. A 27-year-old nursing mother is diagnosed as drugs used in HIV-positive patients, and acute
suffering from genital herpes. She has a history of pancreatitis has been reported for several reverse
this viral infection. Previously, she responded to a transcriptase inhibitors. However, didanosine’s most
drug used topically. Apart from her current problem, characteristic adverse effect is a dose-limiting acute
she is in good health. Which drug to be used orally is pancreatitis. Other risk factors that are relative
most likely to be prescribed at this time? contraindications to didanosine are advanced AIDS,
(A) Amantadine hypertriglyceridemia, and alcoholism. The answer is
(B) Foscarnet B.
(C) Ritonavir 3.
(D) Trifluridine Symptomatic AIDS patients should be treated with a
(E) Valacyclovir HAART regimen regardless of a relatively high CD4
8. Oral formulations of this drug should not be used count or a relatively low HIV RNA load. Because
in a pregnant AIDS patient because they contain didanosine must be discontinued, lamivudine would
propylene glycol. One of the characteristic adverse be a good NRTI replacement. Use of a second
effects of the drug is hyperpigmentation on the protease inhibitor (eg, atazanavir) with a single
palms of the hands and soles of the feet, especially reverse transcriptase inhibitor could be as effective
in African-American patients. as regimens that include 2 reverse transcriptase
(A) Amprenavir inhibitors, although there may be an increased
(B) Emtricitabine possibility of drug interactions. Atazanavir use is
(C) Efavirenz associated with electrocardiograhic PR-interval
(D) Fosamprenavir prolongation, which may be exacerbated by other
(E) Zalcitabine causative agents such as the calcium channel blocker
9. Which of the following statements about verapamil which an older patient might be taking for
interferon-a is false? angina. The answer is D.
4.
The viral RNA titer in the blood from the AIDS Acyclovir is eliminated in the urine by glomerular
patient in this case is very high, and this needlestick filtration and by active tubular secretion, which is
must be considered as a high-risk situation. Although inhibited by probenecid. Nephrotoxic effects,
full doses of zidovudine for 4 wk has been shown to including hematuria and crystalluria, are enhanced in
have prophylactic value, in high-risk situations patients who are dehydrated or who have
combination regimens are favored. Optimal preexisting renal dysfunction. Adequate hydration is
prophylaxis in this case might best be provided by equally important in the case of indinavir because it
the combination of zidovudine with lamivudine causes nephrolithiasis. However, more than 80% of a
(basic regimen), plus the addition of protease dose of indinavir is eliminated via hepatic
inhibitors (expanded regimen). The answer is E. metabolism. Trifluridine is used topically to treat
5. herpes keratoconjunctivitis. The answer is A.
Ganciclovir (not listed) has been the most commonly QUESTIONS
used drug for prevention and treatment of CMV 1. Infections caused by gram-negative bacilli have
infections in the immunocompromised patient. occurred when this cationic surfactant has been
However, cidofovir is also very effective in CMV used as a skin antiseptic.
retinitis and has good activity against many strains of (A) Acetic acid
HSV, including those resistant to acyclovir. The (B) Benzalkonium chloride
answer is B. (C) Lindane
6. (D) Hexachlorophene
Drug interactions can be severe in the (E) Thimerosal
immunocompromised patient because many of the Questions 2 and 3. A young woman is brought to a
drugs administered can influence the hospital emergency department with intense
pharmacokinetic properties of other drugs. abdominal pain of 2 d duration. The pain has spread
Rifabutin, like rifampin, acts as an inducer of several to the right lower quadrant and is accompanied by
isoforms of hepatic cytochrome P450. This action nausea, vomiting, and fever. She arrives at the
can result in an increased clearance of other drugs, emergency department with a blood pressure of
including indinavir. The answer is D. 85/45, pulse 120/min, and temperature 40°C. Her
7. abdomen has a board-like rigidity with diffuse pain
Three of the drugs listed (foscarnet, trifluridine, to palpation. Laboratory values include the
valacyclovir) are active against strains of herpes following: WBC 20,000/μL and creatinine 1.5 mg/dL.
simplex virus. Foscarnet is not used in genital After abdominal x-ray films are taken, a preliminary
infections (HSV-2) because clinical efficacy has not diagnosis of abdominal sepsis is made, possibly
been established, it has poor oral bioavailability and resulting from bowel perforation. After appropriate
the drug causes many toxic effects. Trifluridine is samples are sent to the laboratory for culture, the
used topically but only for herpes patient is hospitalized, and antimicrobial therapy is
keratoconjunctivitis (HSV-1). Valacyclovir is started with intravenous ampicillin and gentamicin.
converted to acyclovir by first-pass metabolism in 2. Regarding the treatment of this patient, which
the intestine and liver. The answer is E. statement is accurate?
8. (A) A drug active against anaerobes should be
Three of the drugs listed should be avoided, or used included in the antimicrobial drug regimen
with extreme caution, in the pregnant patient. Oral (B) Cultures are pointless because this is probably a
forms of amprenavir and emtricitabine both contain mixed infection
propylene glycol, a potentially toxic compound. (C) Empiric antibiotic therapy of abdominal sepsis
Efavirenz has caused fetal abnormalities in pregnant should always include a third-generation
monkeys. However, one of the distinctive adverse cephalosporin
effects of emtricitabine is hyperpigmentation. The (D) Gram stain of the blood would provide positive
answer is B. identification of the specific organism involved in
9. this infection
Lamivudine is used in monotherapy of HBV (E) The combination of ampicillin and gentamicin
infections and does not oppose the beneficial effects provides good coverage for all likely pathogens
of interferon-α when both agents are used together 3. If the antibiotic regimen in this patient is modified
in the treatment of hepatitis B. The answer is D. to include metronidazole
10. (A) Ampicillin should be excluded from the regimen
(B) Coverage will be extended to methicillin-resistant contaminating the floor, which is made of porous
staphylococci material. The best way to deal with this is to
(C) Gentamicin should be excluded from the regimen (A) Clean the floor with a 10% solution of household
(D) Metronidazole should not be administered bleach
intravenously (B) Clean the floor with soap and water
(E) The patient should be monitored for candidiasis (C) Completely replace the contaminated part of the
4. Which compound is the safest drug to use floor
topically to treat scabies and pediculosis? (D) Neutralize the spill with a solution of potassium
(A) Benzoyl peroxide permanganate
(B) Chlorhexidine (E) Seal the room and decontaminate with ethylene
(C) Lindane oxide
(D) Permethrin 10. Neuropathies are more likely to occur with this
(E) Silver sulfadiazine agent when it is used in patients with renal
5. Methenamine salts are used as urinary dysfunction. The drug may cause acute hemolysis in
antiseptics. The reason they lack systemic patients with glucose-6-phosphate dehydrogenase
antibacterial action is that they are (G6PD) deficiency.
(A) Converted to formaldehyde only at low pH (A) Chlorhexidine
(B) Metabolized rapidly by hepatic drug- (B) Halazone
metabolizing enzymes (C) Methenamine
(C) More than 98% bound to plasma proteins (D) Metronidazole
(D) Not absorbed into the systemic circulation after (E) Nitrofurantoin
oral ingestion ANSWERS
(E) Substrates for active tubular secretion 1.
6. Which statement about the actions of Pseudomonas and other gram-negative bacteria
antimicrobial agents is false? have caused infections after the use of cationic
(A) Metronidazole has activity against C difficile. surfactants such as benzalkonium and
(B) Neonatal gonococcal ophthalmia can be cetylpyridinium chlorides, partly because they form
prevented by silver nitrate a film on the skin under which microorganisms can
(C) Polymyxins act as cationic detergents to disrupt survive. In addition, some gram-negative bacilli are
bacterial cell membranes able to grow in solutions containing benzalkonium
(D) Resistance to nitrofurans emerges rapidly, and salts. Bacterial growth may also occur in solutions of
there is cross-resistance with sulfonamides povidone-iodine. The answer is B.
(E) Salicylic acid has useful antidermatophytic 2.
activity when applied locally Abdominal sepsis is commonly a mixed infection; the
7. Which antiseptic promotes wound healing? most likely pathogens are Bacteroides fragilis,
(A) Cetylpyridinium chloride Enterobacteriaceae, and Enterococcus faecalis. An
(B) Chlorhexidine antibiotic regimen that includes only ampicillin and
(C) Hexachlorophene gentamicin does not control B fragilis. Empiric
(D) Phenol treatment in this case should include a drug active
(E) None of the above against this pathogen (eg, metronidazole, cefoxitin,
8. A 22-year-old man with gonorrhea is to be treated cefotetan, or clindamycin). The answer is A.
with cefixime and will need another drug to provide 3.
coverage for possible urethritis caused by C Fungal superinfections, especially from Candida
trachomatis. Which of the following drugs is least albicans, occur frequently during treatment with
likely to be effective in nongonococcal urethritis? metronidazole. In most cases of abdominal sepsis,
(A) Azithromycin metronidazole would be given by slow intravenous
(B) Ciprofloxacin infusion. Both ampicillin and gentamicin should be
(C) Erythromycin maintained until the infection is controlled, at which
(D) Nitrofurantoin time surgery is indicated. Metronidazole has no
(E) Tetracycline activity against aerobes. The combination of
9. A patient with AIDS has an extremely high viral ampicillin, gentamicin, and metronidazole does not
RNA titer. While blood is being drawn from this provide coverage for methicillin-resistant
patient, the syringe is accidentally dropped, staphylococci. The answer is E.
4. 1. Antimicrobial treatment of this severely immune-
Of the agents listed, both lindane and permethrin depressed patient should not be initiated before
are effective scabicides and pediculicides. However, (A) Antipyretic drugs have been given to reduce
there is some concern about systemic absorption of body temperature
lindane, which may cause neurotoxicity and (B) Infecting organism(s) have been identified by the
hematotoxicity. Accidental ingestion of lindane in microbiology laboratory
children has caused seizures. The answer is D. (C) Results of a Gram stain are available
5. (D) Results of antibacterial susceptibility tests are
Below pH 5.5, methenamine releases formaldehyde, available
which is antibacterial. This pH is achieved in the (E) Specimens have been taken for laboratory tests
urine but nowhere else in the body. Ascorbic acid is and examination
sometimes given with methenamine salts to ensure 2. If amikacin is used in the treatment of this patient,
a low urinary pH. The answer is A. monitoring of serum drug level may be advised
6. because the drug
Resistance emerges very slowly when nitrofurantoin (A) Does not penetrate into cerebrospinal fluid
is used as a urinary antiseptic. There is no cross- (B) Has a narrow therapeutic window
resistance between the drug and other drugs used in (C) Is antagonized by beta-lactam antibiotics
the treatment of bacterial infections of the urinary (D) Is hematotoxic
tract. The answer is D. (E) Is rapidly metabolized by the liver
7. 3. A combination of drugs might be given to this
No antiseptic in current use is able to promote patient to provide coverage against multiple
wound healing, and most agents do the opposite. In organisms or to obtain a synergistic action. Examples
general, cleansing of abrasions and superficial of antimicrobial drug synergism established at the
wounds with soap and water is just as effective as clinical level include the treatment of
and less damaging than the application of topical (A) Cryptococcal meningitis with amphotericin B and
antiseptics. Phenol is only used as a disinfectant of flucytosine
inanimate objects! The answer is E. (B) Coliform infections with sulfamethoxazole and
8. trimethoprim
Urinary tract infections resulting from C trachomatis (C) Enterococcal infections with rifampin and
are likely to respond to all of the drugs listed except vancomycin
nitrofurantoin. However, nitrofurantoin is effective (D) Pseudomonal infections with carbenicillin and
against many bacterial urinary tract pathogens with gentamicin
the exception of Pseudomonas aeruginosa and (E) All of the above
strains of Proteus. The answer is D. Questions 4 and 5. A 27-year-old pregnant patient
9. with a history of pyelonephritis has developed a
Household bleach contains sodium hypochlorite. A severe upper respiratory tract infection that appears
1:10 dilution of bleach is effective for disinfection of to be due to a bacterial pathogen. The woman is
a direct blood spill on a porous surface. In addition hospitalized, and an antibacterial agent is to be
to inactivating HIV, sodium hypochlorite solutions selected for treatment.
have disinfectant activity against other viruses, 4. Assuming that the physician is concerned about
including hepatitis B virus. The answer is A. the effects of renal impairment on drug dosage in
10. this patient, which drug would not require dosage
Acute hemolytic reactions in G6PD deficiency occur modification in renal dysfunction?
with drugs that are oxidizing agents, including (A) Amoxicillin
antimalarials, sulfonamides, and nitrofurans. Severe (B) Cefoperazone
polyneuropathies may occur with nitrofurantoin, (C) Ciprofloxacin
and they are more likely to occur in patients with (D) Trimethoprim-sulfamethoxazole
renal dysfunction. The answer is E. (E) Vancomycin
QUESTIONS 5. Which antibacterial agent appears to be the safest
Questions 1–3. A hospitalized AIDS patient is to use in the pregnant patient?
receiving antiretroviral drugs but no antimicrobial (A) Amikacin
prophylaxis. He develops sepsis with fever, (B) Azithromycin
suspected to be caused by a gram-negative bacillus. (C) Ciprofloxacin
(D) Erythromycin (C) Doxycycline
(E) Tetracycline (D) Levofloxacin
Questions 6 and 7. A 48-year-old patient is (E) Vancomycin
scheduled for a vaginal hysterectomy. An ANSWERS
antimicrobial drug will be used for prophylaxis 1.
against postoperative infection. It is proposed that To delay therapy until laboratory results are
cefazolin, a first-generation cephalosporin, be given available is inappropriate in serious bacterial
intravenously at the normal therapeutic dose infections, but specimens for possible microbial
immediately before surgery and continued until the identification must be obtained before drugs are
patient is released from the hospital. administered. The answer is E.
6. Which statement about the proposed drug 2.
management of this patient is not accurate? Monitoring plasma aminoglycoside levels is
(A) Antibiotic treatment throughout hospitalization important because amikacin and other
will prevent nosocomial infections aminoglycosides have a low therapeutic index;
(B) Likely pathogens include anaerobes, enteric toxicity may occur when plasma levels are only 3–4
gram-negative bacteria, and group B streptococci times higher than minimal inhibitory concentrations.
(C) Prophylaxis with antimicrobial drugs has efficacy Decreases in renal function may elevate the plasma
in this type of surgical procedure levels of aminoglycosides to toxic levels within a few
(D) This drug will not be effective against anaerobes hours. Aminoglycosides undergo renal elimination
(E) Without prophylaxis, the infection rate following and they are not hematotoxic. The answer is B.
this procedure exceeds 5% under optimal conditions 3.
7. If the patient had been scheduled for elective Combinations of antimicrobial drugs are not always
colonic surgery, optimal prophylaxis against infection synergistic and in some cases may even be
would be achieved by mechanical bowel preparation antagonistic. However, all the choices listed are
and the use of established examples of situations in which
(A) Intravenous cefoxitin antimicrobial combinations have greater clinical
(B) Intravenous third-generation cephalosporin efficacy than individual drugs. The answer is E.
(C) Oral amoxicillin 4.
(D) Oral ciprofloxacin Antimicrobial drugs that are eliminated via hepatic
(E) Oral erythromycin and neomycin metabolism or biliary excretion include
8. Which drug increases the hepatic metabolism of erythromycin, cefoperazone, clindamycin,
other drugs? doxycycline, isoniazid, ketoconazole, and nafcillin.
(A) Clarithromycin The answer is B.
(B) Erythromycin 5.
(C) Ketoconazole Several groups of antimicrobial drugs are best
(D) Rifampin avoided in pregnancy if at all possible, including
(E) Ritonavir aminoglycosides, sulfonamides, and tetracyclines.
9. If ampicillin and piperacillin are used in The macrolide azithromycin appears to be safe, but
combination in the treatment of infections resulting studies in animals have shown that clarithromycin is
from Pseudomonas aeruginosa, antagonism may potentially embryotoxic. The answer is B.
occur. The most likely explanation is that 6.
(A) Ampicillin is bacteriostatic With few exceptions, the prophylactic use of
(B) Ampicillin induces beta-lactamase production antibiotics in surgery should not extend beyond the
(C) Autolytic enzymes are inhibited by piperacillin duration of the procedure. After routine surgical
(D) Piperacillin blocks the attachment of ampicillin to procedures, the risk of opportunistic infection (from
penicillin-binding proteins disturbances in microbial flora) increases in a
(E) The 2 drugs form an insoluble complex hospitalized patient if prophylaxis is prolonged;
10. In a patient suffering from pseudomembranous there is also more likelihood of drug toxicity.
colitis due to C difficile with established Cefazolin (or cefoxitin) constitutes the drug(s) of
hypersensitivity to metronidazole the most likely choice for prophylaxis in most hysterectomy
drug to be of clinical value is procedures. The answer is A.
(A) Amoxicillin 7.
(B) Chloramphenicol
Second-generation cephalosporins, including (C) Increased activity of DNA repair mechanisms
cefoxitin and cefotetan, are more active than (D) Increased synthesis of dihydrofolate reductase
cefazolin against bowel anaerobes such as (E) Induction of drug-inactivating enzymes
Bacteroides fragilis and are sometimes used for Questions 3–5. A traveler in a geographical region
prophylaxis in "dirty" surgical procedures. However, where chloroquine-resistant P falciparum is endemic
for elective bowel surgery, most authorities favor used a drug for prophylaxis but nevertheless
the oral use of neomycin together with a poorly developed a severe attack of P vivax malaria.
absorbed formulation of erythromycin, in 3. The drug used for prophylaxis was probably
conjunction with mechanical bowel preparation. In (A) Atovaquone
cases of bowel perforation, the use of a second- or (B) Iodoquinol
third-generation cephalosporin is more appropriate. (C) Mefloquine
The answer is E. (D) Proguanil
8. (E) Tetracycline
Clarithromycin, erythromycin, ketoconazole, and 4. Which drug should be used for oral treatment of
ritonavir inhibit the hepatic metabolism of various the acute attack of P vivax malaria but does not
drugs. Rifampin, an inducer of liver microsomal drug- eradicate exoerythrocytic forms of the parasite?
metabolizing enzymes can increase the metabolism (A) Chloroquine
of other drugs. The answer is D. (B) Mefloquine
9. (C) Primaquine
Gram-negative rods such as Enterobacter and (D) Pyrimethamine-sulfadoxine
Pseudomonas aeruginosa have inducible beta- (E) Quinidine
lactamases. Several beta-lactam antibiotics, 5. Which drug should be given later to eradicate
including ampicillin, cefoxitin, and imipenem, are schizonts and latent hypnozoites in the patient’s
potent inducers of beta-lactamase production. liver?
When such inducers are used with a hydrolyzable (A) Artesunate
penicillin (eg, piperacillin), antagonism may result. (B) Dapsone
The answer is B. (C) Halofantrine
10. (D) Primaquine
Disturbances of gut flora occur commonly during (E) Quinine
treatment with antibiotics, and pseudomembranous Questions 6 and 7. A male patient presents with
colitis has been associated with the use of many lower abdominal discomfort, flatulence, and
agents including ampicillin and clindamycin. occasional diarrhea. A diagnosis of intestinal
Vancomycin can be used in treatment of amebiasis is made, and E histolytica is identified in
pseudomembranous colitis in patients with his diarrheal stools. An oral drug is prescribed, which
established hypersensitivity to metronidazole. The reduces his intestinal symptoms. Later he presents
answer is E. with severe dysentery, right upper quadrant pain,
QUESTIONS weight loss, fever, and an enlarged liver. Amebic
1. Which statement about antiprotozoal drugs is liver abscess is diagnosed, and the patient is
accurate? hospitalized. He has a recent history of drug
(A) Chloroquine is an inhibitor of plasmodial treatment for a cardiac arrhythmia.
dihydrofolate reductase 6. The preferred treatment that he should have
(B) Mefloquine destroys secondary exoerythrocytic received for the initial symptoms (which were
schizonts indicative of mild-to moderate disease) is
(C) Primaquine is a blood schizonticide and does not (A) Diloxanide furoate
affect secondary tissue schizonts (B) Iodoquinol
(D) Proguanil complexes with double-stranded DNA- (C) Metronidazole
blocking replication (D) Metronidazole plus diloxanide furoate
(E) Trimethoprim-sulfamethoxazole is the drug of (E) Paromomycin
choice for Pneumocystis jiroveci pneumonia 7. The drug regimen most likely to be effective in
2. Plasmodial resistance to chloroquine is due to treating severe extraintestinal disease in this patient
(A) Change in receptor structure is
(B) Decreased accumulation of the drug in the food (A) Chloroquine
vacuole (B) Diloxanide furoate plus iodoquinol
(C) Emetine plus diloxanide furoate plus chloroquine Chloroquine is the drug of choice for the oral
(D) Pentamidine followed by mefloquine treatment of an acute attack of malaria caused by P
(E) Tinidazole plus diloxanide furoate vivax but will not eradicate exoerythrocytic forms of
8. After a backpacking trip in the mountains, a 24- the parasite. The answer is A.
year-old man develops diarrhea. He acknowledges 5.
drinking stream water without purification, and you Primaquine is the only antimalarial drug that reliably
suspect he is showing symptoms of giardiasis. acts on tissue schizonts in liver cells. Quinine is a
Because you know that laboratory detection of cysts highly effective blood shizonticide against all 4
or trophozoites in the feces can be difficult, you species of human malaria parasites, but it is not
decide to treat the patient empirically with active against liver stages. Starting about day 4 after
(A) Chloroquine an acute attack, primaquine should be given daily for
(B) Emetine 2 wk. The answer is D.
(C) Pentamidine 6.
(D) Tinidazole Metronidazole plus a luminal amebicide is the
(E) TMP-SMZ treatment of choice in mild to moderate amebic
9. This drug can clear trypanosomes from the blood colitis. Diloxanide furoate (or iodoquinol, or
and lymph nodes and is active in the late CNS stages paramomycin) can be used as the sole agent in
of African sleeping sickness. asymptomatic intestinal infection. The answer is D.
(A) Emetine 7.
(B) Melarsoprol Metronidazole given for 10 d, or tinidazole for 5 d,
(C) Nifurtimox plus a luminal agent is effective in most cases of
(D) Pentamidine hepatic abscess, and these regimens have the dual
(E) Suramin advantage of being both amebicidal and active
10. Metronidazole is not effective in the treatment against anaerobic bacteria. Though active in amebic
of hepatic abscess, treatment with emetine is
(A) Amebiasis contraindicated in patients with a history of cardiac
(B) Infections due to Bacteroides fragilis disease. The answer is E.
(C) Infections due to Pneumocystis jiroveci 8.
(D) Pseudomembranous colitis Giardiasis is a common intestinal protozoan infection
(E) Trichomoniasis caused by Giardia lamblia. A large number of
ANSWERS infections result from fecal contamination of food or
1. water. Metronidazole is frequently used, but
Proguanil (not chloroquine) is an inhibitor of tinidazole is equally effective. The answer is D.
dihydrofolate reductase. Primaquine (not 9.
mefloquine) is the drug that destroys secondary In the advanced stages of African sleeping sickness,
exoerythrocytic schizonts. TMP-SMZ is the drug of melarsoprol is the drug of choice because, unlike
choice for Pneumocystis jiroveci pneumonia. The pentamidine or suramin, it effectively enters the
answer is E. CNS. Nifurtimox is the most commonly used drug for
2. Chagas’ disease. The answer is B.
Resistance to chloroquine in P falciparum can result 10.
from decreased accumulation of the drug in the food Metronidazole is the drug of first choice for all of the
vacuole caused by the activity of a transporter conditions listed except pneumocystosis. The answer
system encoded by the pfcrt gene. The answer is B. is C.
3. QUESTIONS
Mefloquine is a recommended drug for prophylaxis 1. A missionary from the United States is sent to
in regions of the world where chloroquine-resistant work in a geographic region of a Central American
P falciparum is endemic. One dose of mefloquine country where Onchocerca volvulus is endemic.
weekly starting before travel and continuing until 4 Infections resulting from this tissue nematode
wk after leaving the region is the preferred regimen. (onchocerciasis) are a cause of “river blindness,”
Doxycycline (not tetracycline) is an alternative drug because microfilariae migrate through subcutaneous
for this indication, as is atovaquone plus proguanil tissues and concentrate in the eyes. Which drug
(Malarone). The answer is C. should be used prophylactically to prevent
4. onchocerciasis?
(A) Albendazole (A) Albendazole
(B) Diethylcarbamazine (B) Ivermectin
(C) Ivermectin (C) Niclosamide
(D) Oxamniquine (D) Praziquantel
(E) Pyrantel pamoate (E) Pyrantel pamoate
2. A nonindigenous person who develops 6. Which helminthic infection does not respond to
onchocerciasis in an endemic region and receives treatment with praziquantel?
drug treatment is likely to experience a severe (A) Hydatid disease
reaction. Symptoms include headache, weakness, (B) Opisthorchiasis
rash, muscle aches, hypotension, and peripheral (C) Paragonimiasis
edema. Which statement concerning this reaction is (D) Pork tapeworm infection
accurate? (E) Schistosomiasis
(A) Extensive fluid replacement is essential 7. Which drug enhances the actions of GABA in
(B) Symptoms are more intense in indigenous adults nematodes causing muscle paralysis?
than nonindigenous adults (A) Albendazole
(C) The reaction is due to treatment with suramin (B) Diethylcarbamazine
(D) The reaction is due to killing of microfilariae (C) Ivermectin
(E) The symptoms are characteristic of treatment (D) Oxamniquine
with diethylcarbamazine (E) Pyrantel pamoate
3. Which statement about pyrantel pamoate is 8. Which parasite is susceptible to niclosamide?
accurate? (A) Ascaris lumbricoides (roundworm)
(A) Acts as an antagonist at GABA receptors (B) Echinococcus granulosus (hydatid disease)
(B) Equivalent in efficacy to niclosamide in the (C) Fasciola hepatica (sheep liver fluke)
treatment of tapeworm infections (D) Necator americanus (hookworm)
(C) Eradicates adult worms in the colon but not the (E) Taenia solium (pork tapeworm)
eggs 9. Which adverse effect occurs with the use of
(D) Hepatotoxicity is dose-limiting albendazole during intestinal nematode therapy?
(E) Synergistic with praziquantel in cestode (A) Cholestatic jaundice
infections (B) Corneal opacities
4. A student studying medicine at a Caribbean (C) Hirsutism
university develops intestinal bilharziasis and (D) Peripheral neuropathy
oxamniquine is prescribed. Which statement about (E) None of the above
the proposed drug therapy is accurate? 10. A malnourished 12-year-old child who lives in a
(A) Hospitalization is recommended if the patient rural area of the southern United States presents
has a history of seizure disorders with weakness, fever, cough, abdominal pain, and
(B) Oxamniquine is not effective in the late stages of eosinophilia. His mother tells you that she has seen
the disease long, thin worms in the child’s stools, sometimes
(C) Oxamniquine is safe to use in pregnancy with blood. A presumptive diagnosis of ascariasis is
(D) The drug is an antagonist at GABA receptors in confirmed by the presence of the ova of A
trematodes lumbricoides in the stools. However, microscopy also
(E) The drug is very effective in tapeworm infections reveals that the stools contain the eggs of Necator
5. A 22-year-old man from South Korea has recently americanus. The drug most likely to be effective in
moved to Minnesota. He has symptoms of the treatment of this child is
clonorchiasis (anorexia, upper abdominal pain, (A) Diethylcarbamazine
eosinophilia), presumably contracted in his (B) Ivermectin
homeland where the Oriental liver fluke is endemic. (C) Mebendazole
He also has symptoms of diphyllobothriasis (D) Niclosamide
(abdominal discomfort, diarrhea, megaloblastic (E) Praziquantel
anemia), probably caused by consumption of raw ANSWERS
fish from lakes near the Canadian border. Which 1.
drug is most likely to be effective in the treatment of Ivermectin prevents onchocerciasis and is the drug
both clonorchiasis and diphyllobothriasis in this of choice in the individual and mass treatment of the
patient? disease. The only other drug listed with any activity
against Onchocerca volvulus is diethylcarbamazine. receptors. Pyrantel pamoate blocks nicotinic
Suramin (not listed) also has activity against receptors relaxing nematode muscles.
onchocerciasis, but like diethylcarbamazine, it is less Diethylcarbamazine also cause muscle relaxation but
effective and more toxic than ivermectin. The the mechanism is unknown. Dizziness (no driving for
answer is C. 24 h) is a common adverse effect of oxamniquine.
2. The answer is C.
The symptoms described are those of the so-called 8.
Mazzotti reaction. They are due to the killing by Niclosamide is not active against nematodes or
ivermectin of microfilariae, and their intensity flukes with the exception of the large intestinal
correlates with skin microfilaria load and is not a fluke. It is a co-drug of choice (with praziquantel) to
drug toxicity. The reaction occurs more frequently treat common tapeworm infections and is usually
and with greater severity in nonindigenous persons effective in a single dose. The drug is minimally
than in the indigenous inhabitants of endemic areas. absorbed from the gastrointestinal tract and causes
The answer is D. few side effects. The answer is E.
3. 9.
Pyrantel pamoate, an agonist at nicotinic receptors, Doses of albendazole required for intestinal
is equivalent to albendazole and mebendazole in the nematode therapy are almost free of adverse effects
treatment of common nematode infections. It acts even in the malnourished or debilitated patient.
on adult worms in the colon, but not on eggs. The Gastrointestinal distress may occur in children with
drug causes only mild gastrointestinal side effects ascariasis who are heavily parasitized, together with
and is not hepatotoxic. It is not effective in the a slight headache or dizziness. Avoid the drug in
treatment of infections caused by cestodes or flukes. children younger than 2 yr because of rare reports of
The answer is C. seizures. The answer is E.
4. 10.
Oxamniquine may cause seizures, especially in Mebendazole is effective against both nematodes
persons with a history of convulsive disorders. Such causing infection in this child. Albendazole and
persons should be hospitalized or treated with pyrantel pamoate (not listed in this question) are
praziquantel. Oxamniquine is effective in all stages also primary drugs for the treatment of combined
of disease caused by S mansoni, including advanced infections due to hookworm and roundworm. The
hepatosplenomegaly, and it has been used answer is C.
extensively for mass treatment. The drug is not QUESTIONS
effective in other schistosomal diseases, and it is Questions 1–3. A 32-year-old woman underwent
contraindicated in pregnancy. The answer is A. segmental mastectomy for a breast tumor of 3 cm
5. diameter. Lymph node sampling revealed 2 involved
Praziquantel is a primary drug for treatment of nodes. Because chemotherapy is of established
infections caused by the Oriental liver fluke and by value in her situation, she underwent postoperative
the fish tapeworm. Both types of infection are treatment with antineoplastic drugs. The regimen
transmitted mainly via the consumption of raw fish. consisted of doxorubicin followed by
Niclosamide is also a primary drug for fish tapeworm cyclophosphamide/methotrexate/fluorouracil.
infections, but it is not active against Clonorchis Adjunctive drugs included tamoxifen because the
sinensis. Albendazole is not effective in fish tumor cells were hormone receptor-positive.
tapeworm infections, but it is useful in the pork 1. Which of the following best describes the
tapeworm larval stage (cysticercosis). Pyrantel mechanism of anticancer action of cellular
pamoate is not active against cestodes or metabolites of fluorouracil?
trematodes. The answer is D. (A) Cross-linking of double-stranded DNA
6. (B) Inhibition of DNA-dependent RNA synthesis
In hydatid disease, praziquantel has marginal (C) Interference with the activity of topoisomerases I
efficacy because it does not affect the inner germinal (D) Irreversible inhibition of thymidylate synthase
membrane of Echinococcus granulosus present in (E) Selective inhibition of DNA polymerases
hydatid cysts. The answer is A. 2. The chemotherapy undertaken by this patient
7. caused acute hemorrhagic cystitis. Which drug was
Ivermectin and piperazine (not listed) both cause most likely to be responsible for this toxicity?
muscle paralysis in nematodes by actions on GABA (A) Cyclophosphamide
(B) Doxorubicin (A) Changes in the properties of a target enzyme
(C) Fluorouracil (B) Decreased activity of an activating enzyme
(D) Methotrexate (C) Increased expression of a P-glycoprotein
(E) Tamoxifen transporter
3. After several cycles of chemotherapy, the patient (D) Increased production of drug-trapping molecules
was found to have a high resting pulse rate. A (E) Increase in proteins that are involved in DNA
noninvasive radionuclide scan revealed evidence of repair
cardiomyopathy. The drug that is most likely Questions 8 and 9. A 23-year-old man with
responsible for the cardiac toxicity is Hodgkin’s lymphoma was treated unsuccessfully
(A) Cyclophosphamide with the MOPP regimen (mechlorethamine,
(B) Doxorubicin vincristine, prednisone, procarbazine). He
(C) Fluorouracil subsequently underwent a successful course of
(D) Methotrexate therapy with the ABVD regimen (doxorubicin,
(E) Tamoxifen bleomycin, vinblastine, dacarbazine).
4. A patient with multiple myeloma was started on 8. Which of the following classes of anticancer drugs
bortezomib after 2 rounds of other combination used in the treatment of this patient is cell cycle
chemotherapy did not have any effect. What is the specific (CCS) and used in both the MOPP and ABVD
mechanism of action of bortezomib? regimens?
(A) Cross-linking of double-stranded DNA (A) Alkylating agents
(B) Inhibition of DNA-dependent RNA synthesis (B) Antibiotics
(C) Interference with the activity of topoisomerases I (C) Antimetabolites
(D) Inhibition of the 26S proteasome (D) Glucocorticoids
(E) Selective inhibition of DNA polymerases (E) Plant alkaloids
5. An adult patient is being treated for acute 9. During the second course of drug treatment
leukemia with a combination of anticancer drugs (ABVD regimen), this patient developed dyspnea, a
that includes cyclophosphamide, mercaptopurine, nonproductive cough, and intermittent fever. Chest
methotrexate, vincristine, and prednisone. He is also x-ray film revealed pulmonary infiltration. If these
using ondansetron for emesis, a chlorhexidine problems are due to the anticancer drugs to which
mouthwash to reduce mucositis, and laxatives. The he has been exposed, which of the following is the
patient complains of “pins and needle” sensations in most likely causative agent?
the extremities and muscle weakness. He is not able (A) Bleomycin
to execute a deep knee bend or get up out of a chair (B) Dacarbazine
without using his arm muscles. He is also very (C) Doxorubicin
constipated. If these problems are related to the (D) Prednisone
chemotherapy, which of the following is the most (E) Vinblastine
likely causative agent? 10. All the following agents have been used in drug
(A) Cyclophosphamide regimens for the treatment of breast carcinoma.
(B) Mercaptopurine Which one has specific activity in a subset of female
(C) Methotrexate breast cancers?
(D) Prednisone (A) Cyclophosphamide
(E) Vincristine (B) Doxorubicin
6. Which of the following is a drug that is used in (C) Fluoxymesterone
combination therapy for testicular carcinoma and is (D) Methotrexate
also associated with nephrotoxicity? (E) Trastuzumab
(A) Bleomycin Questions 11–13. For each numbered item, select
(B) Cisplatin the ONE lettered option from the following list that
(C) Etoposide is most closely associated with it. Each lettered
(D) Leuprolide option may be selected once, more than once, or
(E) Vinblastine not at all.
7. A cancer cell that is resistant to the effects of both (A) Bleomycin
vincristine and methotrexate probably has (B) Cytarabine
developed the resistance as a result of which of the (C) Dacarbazine
following mechanisms? (D) Doxorubicin
(E) Etoposide Neuropathy is a toxic side effect of vincristine. In its
(F) Flutamide mildest form, paresthesias occur, but it progresses
(G) Fluorouracil to significant muscle weakness, initially in the
(H) Leuprolide quadriceps muscle group. Constipation is the most
(I) Mechlorethamine common symptom of autonomic neuropathy. The
(J) Mercaptopurine answer is E.
(K) Methotrexate 6.
(L) Paclitaxel Nephrotoxicity is a characteristic toxicity of cisplatin.
(M) Procarbazine Renal toxicity can be reduced by slow intravenous
(N) Rituximab infusion, maintenance of good hydration, and
(O) Vincristine administration of mannitol to maximize urine flow.
11. If allopurinol is used adjunctively in cancer For testicular cancer, cisplatin is used in combination
chemotherapy to offset hyperuricemia, the dosage with etoposide and bleomycin. The answer is B.
of this anticancer drug should be reduced to 25% of 7.
normal. The P-glycoprotein family of transporters moves
12. This drug is used in combination therapy for non- foreign molecules out of cells. Cancer cells acquire
Hodgkin’s lymphoma. Due to its selectivity, it is resistance to multiple drugs that act through
expected to be less myelosuppressive compared different mechanisms by increasing the expressions
with the classic agents. of genes encoding these transporters. The answer is
13. This antimetabolite inhibits DNA polymerase and C.
is one of the most active drugs in leukemias. 8.
Although myelosuppression is dose limiting, the drug The cell cycle-specific drugs used in standard
may also cause cerebellar dysfunction, including treatment protocols for Hodgkin’s lymphoma are
ataxia and dysarthria. bleomycin and the vinca alkaloids. Vinblastine is
ANSWERS used in the ABVD regimen, and vincristine (Oncovin)
1. is used in the MOPP regimen. The answer is E.
Fluorouracil (5-FU) undergoes metabolism to form 5- 9.
fluoro-2′-deoxyuridine 5′-phosphate (5-dUMP). This The anticancer drug most commonly associated with
metabolite forms a covalently bound ternary pulmonary toxicity is bleomycin. The answer is A.
complex with thymidylate synthase and its 10.
coenzyme N-methylenetetrahydrofolate. The Each of the drugs listed has been used in drug
synthesis of thymine nucleotides is blocked, DNA regimens for breast cancer, but only trastuzumab
synthesis is inhibited, and a “thymineless death” of has specificity in its actions. The drug is a
cells results. The answer is D. monoclonal antibody to a surface protein in breast
2. cancer cells that overexpress the HER-2 protein.
Acrolein, a toxic metabolite of cyclophosphamide Consequently, trastuzumab has value in a specific
that is concentrated in the urine, is associated with subset of breast cancers. The answer is E.
hemorrhagic cystitis. Mesna, a sulfur-containing 11.
substance that also concentrates in urine, can be Allopurinol, a xanthine oxidase inhibitor, is given to
administered in an attempt to prevent this control the hyperuricemia that occurs as a result of
complication. The answer is A. large cell kills in the successful drug therapy of
3. malignant diseases. The antimetabolite
A high resting pulse rate is one of the first signs of mercaptopurine is metabolized by xanthine oxidase
cardiotoxicity resulting from anthracyclines, which and, in the presence of an inhibitor of this enzyme
can include arrhythmias, cardiomyopathies, and (eg, allopurinol), toxic levels of the drug may be
heart failure. The risk of cardiotoxicity depends on reached rapidly. The answer is J.
cumulative dosage, so doxorubicin should be 12.
discontinued. The answer is B. Rituximab is a monoclonal antibody (“mab”) used in
4. non-Hodgkin’s lymphoma. It induces cell lysis and
Bortezomib is an inhibitor of the proteasome apoptosis. The answer is N.
structure, whose normal function is to break down 13.
ubiquinated proteins. The answer is D. The pyrimidine antimetabolite cytarabine (Ara-C) is
5. commonly used in drug regimens for the acute
leukemias. Cytarabine is dose-limited by Questions 5 and 6. A patient was treated for a
hematotoxicity. Cerebellar dysfunction may also bacterial infection with a penicillin. Within a few
occur with Ara-C, especially if the drug is used at minutes of the antibiotic injection, he developed
high doses. The answer is B. severe bronchoconstriction, laryngeal edema, and
QUESTIONS hypotension. Because of the rapid administration of
1. Cyclosporine is effective in organ transplantation. epinephrine, the patient survived. Unfortunately, a
Which of the following most accurately describes the year later he was treated with an antipsychotic drug
immunosuppressant action of cyclosporine? and developed agranulocytosis.
(A) Activation of NK cells 5. Which type of immunologic process was triggered
(B) Blockade of tissue responses to inflammatory by the penicillin injection?
mediators (A) An autoimmune syndrome
(C) Increased catabolism of IgG antibodies (B) A cell-mediated reaction
(D) Inhibition of the gene transcription of (C) A type II drug allergy
interleukins (D) Mediated by IgE
(E) Interference with MHC II-peptide activation of T (E) Serum sickness
cells 6. Which type of immunologic process was triggered
2. Which of the following is a widely used drug that by the antipsychotic drug?
suppresses cellular immunity, inhibits prostaglandin (A) A type III drug reaction
and leukotriene synthesis, and increases the (B) A type IV drug reaction
catabolism of IgG antibodies? (C) Delayed-type hypersensitivity
(A) Cyclophosphamide (D) Mediated by IgG or IgM antibodies
(B) Cyclosporine (E) Stevens-Johnson syndrome
(C) Infliximab 7. A 24-year-old woman underwent kidney
(D) Mycophenolate mofetil transplantation. A week later, she developed
(E) Prednisone alloantibody-mediated acute rejection (acute
3. A 30-year-old woman has one living child, age 6 humoral rejection [AHR]). She was successfully
years. Her child and her husband are Rh positive and treated with tacrolimus and a second drug that
she is Rho(D) and Du negative. She is now in her targets both B and T lymphocytes. Which of the
ninth month of pregnancy and is in the labor room following is an immunosuppressant that suppresses
having frequent contractions. Her Rh antibody test both B and T lymphocytes via inhibition of de novo
taken earlier in the pregnancy was negative. What synthesis of purines?
immunotherapy is appropriate for this patient? (A) Cyclophosphamide
(A) Cyclosporine (B) Methotrexate
(B) Cyclophosphamide (C) Mycophenolate mofetil
(C) Methotrexate (D) Prednisone
(D) Rho(D) immune globulin (E) Tacrolimus
(E) Tacrolimus 8. Recombinant interleukin-2 has proved useful in
4. A 36-year-old man presents with swollen, painful the treatment of which of the following diseases?
heels, nail changes, and left lower back pain that (A) Graft-versus-host disease in patients with
wakes him from sleep. The back pain gets better hematopoietic stem cell transplantation
with exercise. He reports 1–2 h of morning stiffness. (B) Psoriasis
He has a history of psoriasis and psoriatic arthritis (C) Renal cell carcinoma
since age 12 years. You decide to change his current (D) Rheumatoid arthritis
regimen of indomethacin to a biologic that targets (E) Superficial bladder carcinoma
TNF-α. Which of the following is a chimeric 9. Although sirolimus and cyclosporine have similar
monoclonal antibody that binds to TNF-α and immunosuppressant effects, their toxicity profiles
inhibits its action? differ. Which of the following toxicities is more likely
(A) Etanercept to be associated with sirolimus than with
(B) Infliximab cyclosporine?
(C) Sirolimus (A) An anaphylactic reaction
(D) Trastuzumab (B) Hypertension
(E) Thalidomide (C) Osteoporosis
(D) Renal insufficiency
(E) Thrombocytopenia his psychosis (see clozapine toxicity, Chapter 29).
10. Which of the following is an immune modulator The answer is D.
that increases phagocytosis by macrophages in 7.
patients with chronic granulomatous disease? Mycophenolic acid, formed from mycophenolate
(A) Aldesleukin mofetil, inhibits inosine monophosphate
(B) Interferon-γ dehydrogenase, the rate-limiting enzyme in the de
(C) Lymphocyte immune globulin novo pathway of purine synthesis. This action
(D) Prednisone suppresses both B- and T-lymphocyte activation.
(E) Trastuzumab Mycophenolate mofetil is used in organ
ANSWERS transplantation. The answer is C.
1. 8.
Cyclosporine inhibits calcineurin, a serine Interleukin-2 is a cytokine that stimulates T-cell
phosphatase that is needed for activation of T-cell- proliferation and activates Th1, NK, and LAK cells. It
specific transcription factors such as NF-AT. Gene has shown efficacy in renal cell carcinoma and
transcription of IL-2, IL-3, and interferon-γ is malignant melanoma, 2 cancers that respond poorly
inhibited. The answer is D. to conventional cytotoxic anticancer drugs. The
2. answer is C.
The corticosteroid prednisone is used extensively as 9.
an immunosuppressant in autoimmune diseases and Cyclosporine and tacrolimus both are associated
organ transplantation. Glucocorticoids have multiple with renal toxicity and hypertension. In contrast,
actions, including those described. The answer is E. sirolimus appears to spare the kidney and instead is
3. more likely to cause gastrointestinal disturbance,
Rho(D) immune globulin contains antibodies against hypertriglyceridemia, and myelosuppression,
Rho(D) antigens. If an injection of Rho(D) antibody is especially in the form of thrombocytopenia. The
administered to the Rh-negative mother within 24– answer is E.
72 h after the birth of an Rh-positive infant, the 10.
mother’s own antibody response to the foreign Interferon-γ is approved for use in chronic
Rho(D)-positive cells is suppressed because the granulomatous disease, a condition that results from
infant’s red cells are cleared from circulation before phagocyte deficiency. The agent markedly reduces
the mother can generate a B-cell response against the frequency of recurrent infections. The answer is
Rho(D). Therefore, she has no memory B cells that B.
can activate upon subsequent pregnancies with an QUESTIONS
Rho(D)-positive fetus. The answer is D. 1. The light brownish color of smog often apparent
4. in a major metropolitan area on a hot summer day is
Infliximab is a chimeric monoclonal antibody that mainly due to
binds to TNF-α. Etanercept also binds to TNF-α, but (A) Carbon monoxide
it is a chimeric protein containing a portion of the (B) Hydrocarbons
human TNF-α receptor linked to the Fc region of a (C) Ozone
human IgG. Thalidomide is a small molecule that (D) Nitrogen dioxide
appears to inhibit production of TNF-α. Trastuzumab (E) Sulfur dioxide
is a humanized monoclonal antibody against HER- 2. You are stuck in traffic in New York City in summer
2/neu. The answer is B. for 3 or 4 h and you begin to get a headache, a
5. feeling of tightness in the temporal region, and an
The patient experienced an anaphylactic response to increased pulse rate. What is the antidote based on
the penicillin. This is a type I (immediate) drug the most likely cause of these effects?
reaction, mediated by IgE antibodies. The answer is (A) Activated charcoal
D. (B) Atropine
6. (C) Fomepizole
Agranulocytosis (and systemic lupus erythematosus) (D) Oxygen
are autoimmune syndromes that can be drug- (E) Pralidoxime
induced. They are type II reactions involving IgM and 3. A farm worker was accidentally in the field during
IgG antibodies that bind to circulating blood cells. the aerial spraying with parathion. He was brought
The patient was probably treated with clozapine for to the emergency department. Which of the
following will be used in the treatment of this (C) Nitrogen mustards
patient? (D) Organophosphates
(A) Antiseizure drugs (E) Rotenones
(B) Atropine and pralidoxime Questions 8–10. The matching questions in this
(C) Hemodialysis section consist of a list of lettered options followed
(D) Hyperbaric oxygen by several numbered items. For each numbered
(E) Measures to reduce pulmonary edema item, select the ONE lettered option that is most
4. A compound that is toxic to bone marrow cells in closely associated with it. Each lettered option may
the early stages of development and that may also be selected once, more than once, or not at all.
be leukemogenic is (A) Aldicarb
(A) Benzene (B) Benzene
(B) Carbon monoxide (C) Carbon monoxide
(C) Glyphosate (D) Carbon dioxide
(D) DDT (E) DDT
(E) Pyrethrum (F) Dioxin
5. A compound or group of compounds that (G) Malathion
damages the skin and whose use in manufacturing (H) Nitrogen dioxide
has largely been eliminated because of extensive (I) Paraquat
persistence in the environment and bioaccumulation (J) Pyrethrum
is (K) Rotenone
(A) Aromatic hydrocarbons such as benzene (L) Sulfur dioxide
(B) Dichlorvos (M) Tetrachloroethylene
(C) Phenoxyacetic acids such as 2,4- (N) Toluene
dichlorophenoxyacetic acid 8. Asthma is often exacerbated in patients exposed
(D) Polychlorinated biphenyls (PCBs) to this reducing agent when concentrations in the air
(E) 2,3,7,8-tetrachlorodibenzo-p-dioxin (TCDD) are as low as 1–2 ppm. It is formed mainly from
6. An employee of a company engaged in clearing combustion of fossil fuels.
vegetation from county roadsides accidentally 9. Acute exposure to this aliphatic hydrocarbon
ingested a small quantity of an herbicidal solution solvent causes CNS depression; chronic exposure has
that contained paraquat. Within led to impairment of memory and peripheral
2 h, he was admitted to the emergency department neuropathy.
of a nearby hospital. Which of the following best 10. This compound is a potential environmental
describes his probable signs and symptoms in the hazard that is formed as a contaminating by-product
emergency department? in the manufacture of herbicides. It causes
(A) Diarrhea, vomiting, sweating, and profound acneiform lesions and may be carcinogenic.
skeletal muscle weakness ANSWERS
(B) Dizziness, nausea, agitation, and hyperreflexia 1.
(C) Dyspnea, pulmonary dysfunction, and elevated Smog color is derived in part from suspended
body temperature particulate matter. When smog is light brown, the
(D) Gastrointestinal irritation with hematemesis and color derives from nitrogen oxides. All of the other
bloody stools air pollutants listed are colorless. The answer is D.
(E) Hypotension, tachycardia, and respiratory 2.
impairment The symptoms described are those of carbon
7. Chemical warfare agents that had been monoxide inhalation. Oxygen is the antidote. The
manufactured in the 1950s were being stored at a answer is D. Note atropine and pralidoxime are used
military installation. Several civilian workers at the in insecticide poisoning with acetylcholinesterase
facility began to feel unwell, with symptoms that inhibitors, and fomepizole is used in methanol and
included dyspnea, abdominal cramps, and diarrhea. ethylene glycol poisoning.
They also had copious nasal and tracheobronchial 3.
secretions. Which type of toxic compound is most Organophosphate poisoning is treated with the
likely to be the cause of these effects? muscarinic receptor antagonist atropine and
(A) Aliphatic hydrocarbons pralidoxime, which regenerates cholinesterase. The
(B) Botulinum toxins answer is B.
4. (A) Acetylcysteine
The aromatic hydrocarbon benzene is used as a (B) Deferoxamine
solvent in industry. Long-term exposure is associated (C) EDTA
with increased risk of leukemia. The answer is A. (D) Penicillamine
5. (E) Succimer
The polychlorinated biphenyls (PCBs) are 2. A young woman employed as a dental laboratory
dermatotoxic drugs that persist in the environment technician complains of conjunctivitis, skin irritation,
and accumulate in living organisms. PCBs have been and hair loss. On examination, she has perforation of
banned from manufacture in the United States since the nasal septum and a “milk and roses” complexion.
1979. However, many electrical transformers still These signs and symptoms are most likely due to
retain traces of them. The answer is D. (A) Acute mercury poisoning
6. (B) Chronic inorganic arsenic poisoning
Paraquat is highly corrosive to the gastrointestinal (C) Chronic mercury poisoning
tract. Oral ingestion of the herbicide leads to marked (D) Excessive use of supplementary iron tablets
gastrointestinal irritation, hematemesis, and usually (E) Lead poisoning
blood in the stools. Signs of pulmonary impairment
do not appear for several days and are usually 4. A young engineer involved in the smelting process
progressive, resulting in severe pulmonary fibrosis of cobalt and gold presented with severe GI
and, often, death. The answer is D. discomfort, rice water stools, and a sweet garlicky
7. breath. Acute inorganic arsenic poisoning was
Highly potent organophosphate inhibitors of diagnosed. Which of the following drugs should be
acetylcholinesterase (eg, sarin, tabun) have been included in the management of this patient?
developed for chemical warfare purposes. Their (A) Deferoxamine
storage represents a potential toxicologic hazard. It (B) Dimercaprol
is important to recognize the signs and symptoms of (C) EDTA
excess acetylcholine (DUMBBELSS; see Chapter 7), (D) Penicillamine
which include those described. The answer is D. (E) Succimer
8.
Sulfur dioxide is a reducing agent that forms 5. A 24-year-old man was employed in the supply
sulfurous acid on contact with moist surfaces. This is department of a company that manufactures
responsible for irritant effects on mucous semiconductors. After an accident at the plant, he
membranes of the eye, the oropharyngeal cavity, presented with nausea and vomiting, headache,
and the respiratory tract. Nitrogen dioxide causes hypotension, and shivering. Laboratory analyses
similar problems, but it is an oxidizing agent formed showed hemoglobinuria and a plasma free
from fires and in silage on farms. The answer is L. hemoglobin level greater than 1.4 g/dL. This young
9. man was probably exposed to
Three hydrocarbon solvents are listed: benzene, (A) Arsine
tetrachloroethylene, and toluene. Each can cause (B) Inorganic arsenic
CNS effects such as headache, fatigue, and loss of (C) Mercury vapor
appetite. However, benzene and toluene are (D) Methylmercury
aromatic hydrocarbons. The answer is M. (E) Tetraethyl lead
10. 6. A 2-year-old child was brought to the emergency
Dioxin is a contaminant formed in the manufacture department 1 h after ingestion of tablets he had
of chlorophenoxy acid herbicides, including 2,4- managed to obtain from a bottle on top of the
dichlorophenoxyacetic acid and 2,4,5- refrigerator. His symptoms included marked
trichlorophenoxyacetic acid. The answer is F. gastrointestinal distress, vomiting (with
QUESTIONS hematemesis), and epigastric pain. Metabolic
1. A small child is brought to a hospital emergency acidosis and leukocytosis were also present. This
department suffering from severe gastrointestinal patient is most likely to have ingested tablets
distress and abdominal colic. If this patient has containing
severe acute lead poisoning with signs and (A) Acetaminophen
symptoms of encephalopathy, treatment should be (B) Aspirin
instituted immediately with (C) Diphenhydramine
(D) Iron The treatment of choice in acute arsenic poisoning is
(E) Vitamin C intramuscular dimercaprol. Although succimer is less
Questions 7–10. The matching questions in this toxic, it is only available in an oral formulation, and
section consist of a list of lettered options followed its absorption may be impaired by the severe
by several numbered items. For each numbered gastroenteritis that occurs in acute arsenic
item, select the ONE lettered option that is most poisoning. The answer is B.
closely associated with it. Each lettered option may 5.
be selected once, more than once, or not at all. From the signs and symptoms alone, a diagnosis of
(A) Arsine arsine gas poisoning cannot be made. However,
(B) Deferoxamine clues to the cause of poisoning are often provided by
(C) Dimercaprol a patient’s occupation. The laboratory reports
(D) Edetate calcium disodium suggest marked hemolysis. Arsine gas binds to
(E) Inorganic mercury hemoglobin and decreases erythrocyte glutathione
(F) Iron levels, causing membrane fragility and resulting
(G) Methylmercury hemolysis. The answer is A.
(H) Mercury vapor 6.
(I) Penicillamine This question emphasizes that the ingestion of iron
(J) Succimer tablets is a relatively common cause of accidental
(K) Tetraethyl lead poisoning in young children. The signs and
(L) Trivalent arsenic symptoms described usually occur in the first 6 h
7. This toxic compound can be produced in seawater after ingestion. In a child whose body weight is 22 lb,
by the action of bacteria and algae. It is also the ingestion of 600 mg can cause severe, perhaps
synthesized chemically for commercial use as a lethal, toxicity. The answer is D.
fungicide. 7.
8. This agent is used in the treatment of Wilson’s Methylmercury is used as a fungicide to prevent
disease and has been reported to cause lupus mold growth in seed grain. The answer is G.
erythematosus and hemolytic anemia. 8.
9. High doses of this agent can cause histamine Autoimmune diseases such as lupus erythematosus
release and extreme vasodilation. and hemolytic anemia have occurred during the
10. Gingivitis, discolored gums, and loose teeth are treatment of Wilson’s disease with penicillamine.
common symptoms of chronic exposure to this The answer is I.
agent. 9.
ANSWERS Deferoxamine can cause shock if given by rapid
1. intravenous infusion. The answer is B.
Encephalopathy in severe lead poisoning is a medical 10.
emergency. Of the drugs listed, intravenous EDTA is Oral and gastrointestinal complaints are common in
the most effective chelating agent. Oral succimer is chronic mercury poisoning, and tremor involving the
used in children with mild to moderate lead fingers and arms is often present. The answer is E.
poisoning and may be initiated 4–5 d after the
parenteral use of EDTA or dimercaprol in severe QUESTIONS
poisoning. The answer is C. Questions 1–3. A 2-year-old girl presented with
lethargy, increased respiratory rate, and an elevated
2. temperature that appeared to result from a drug
The “milk and roses” complexion, which results from poisoning. Laboratory testing revealed the following
vasodilation and anemia, is a characteristic of serum concentrations: glucose, 36 mg/dL; Na+, 148
chronic inorganic arsenic poisoning, whereas mEq/L; K+, 5 mEq/L; Cl−, 111 mEq/L; HCO3−, 12
patients with lead poisoning often have a gray pallor. mEq/L; BUN, 21 mg/dL; osmolality, 300 mOsm/L.
Other signs and symptoms of arsenic poisoning 1. The anion gap in this patient is
include gastrointestinal distress, hyperpigmentation, (A) −60 mEq/L
and white lines on the nails. We hope you were not (B) −20 mEq/L
led astray by her employment. The answer is B. (C) +5 mEq/L
4. (D) +30 mEq/L
(E) +304 mEq/L
2. The osmolar gap in this patient is 8 times the threshold for toxicity. Pharmacokinetic
(A) −40 mOsm/L parameters for digoxin in this patient include a
(B) −5 mOsm/L clearance of 7 L/h and an elimination half-life of 56
(C) +15 mOsm/L h. If no procedures are instituted to decontaminate
(D) +60 mOsm/L this patient, the time taken to reach a safe level of
(E) +305 mOsm/L digoxin will be approximately
3. The patient’s signs, symptoms, and laboratory (A) 3.5 d
values are most consistent with an overdose of (B) 7 d
(A) Acetaminophen (C) 14 d
(B) Aspirin (D) 28 d
(C) Ethylene glycol (E) 56 d
(D) Lead Questions 8 and 9. A patient is brought to the
(E) Phencyclidine emergency department having taken an overdose
4. An 18-month-old boy presented in a (unknown quantity) of a sustained-release
semiconscious state with profound hypotension and preparation of theophylline by oral administration 2
bradycardia after ingesting a number of his h previously. He has marked gastrointestinal distress
grandmother’s metoprolol tablets. In this case, the with vomiting, is agitated, and exhibits hyperreflexia
most appropriate antidote is and hypotension.
(A) Atropine 8. The plasma level of theophylline measured
(B) Esmolol immediately upon hospitalization was 80 mg/L. If the
(C) Glucagon oral bioavailability of theophylline is 98%, the
(D) Naloxone clearance is 50 mL/min, volume of distribution is 35
(E) Neostigmine L, and the elimination half-life is 7.5 h, the amount
5. An 81-year-old woman with type 2 diabetes ingested must have been at least
presents to the emergency department in a coma (A) 0.3 g
and with tachypnea, tachycardia, hypotension, and (B) 0.6 g
severe lactic acidosis approximately 9 h after (C) 1.6 g
ingesting a number of her metformin tablets. Her (D) 2.8 g
serum glucose concentration is 148 mg/dL. (E) 8.0 g
Metformin is a base with a pKa of 12.4. The 9. A short-acting antidote that can reduce this
procedure that is most likely to improve her patient’s tachycardia is
condition is (A) Acetylcysteine
(A) Administration of activated charcoal (B) Deferoxamine
(B) Administration of glucagon (C) Esmolol
(C) Administration of syrup of ipecac (D) Fomepizole
(D) Gastric lavage (E) Pralidoxime
(E) Hemodialysis 10. A contraindication to the use of gastric lavage for
6. A 24-year-old female was rushed to the the removal of drugs from the stomach of victim of
emergency department after she was found in her poisoning is
room hypotensive, with seizures. In the emergency (A) An overdose of iron pills
department, the electrocardiogram confirmed (B) An unconscious patient
ventricular arrhythmias. An overdose of which of the (C) Ingestion of a corrosive
following drugs is the most likely cause of her (D) Overdose with a sustained-release formulation
symptoms? ANSWERS
(A) Acetaminophen 1.
(B) Amitriptyline Anion gap is calculated by subtracting measured
(C) Diazepam serum anions (bicarbonate plus chloride) from
(D) Ethylene glycol cations (potassium plus sodium). Increases in anion
(E) Morphine gap above normal are due to the presence of
unmeasured anions that accompany acidosis. The
7. A patient with heart failure has accidentally taken gap in this case is 30 mEq/L, a value that is well in
an overdose of digoxin. The blood concentration of excess of the normal gap (12–16 mEq/L). The answer
the drug is is D.
2. digoxin in this patient, the time taken to reach a safe
The osmolar gap is the difference between the plasma level of the drug (12.5% of the measured
measured serum osmolality and the osmolarity level) is 3 half-lives, or approximately 7 d. The
calculated from the serum sodium, glucose, and BUN answer is B.
concentrations according to the equation above for 8.
calculating the osmolar gap. In this case, the Estimations of the quantity of a drug or toxin
measured osmolality is 300 mOsm/L, whereas the ingested may be of value in the management of the
calculated osmolality is 305 mOsm/L; the difference poisoned patient. Applying toxicokinetic principles, a
is −5 mOsm/L. The answer is B. rough estimate of ingested dose of theophylline
3. could be made by multiplying the peak plasma level
Of the drugs listed, the 2 that are likely to cause an of the drug (80 mg/L) by its volume of distribution
anion gap are aspirin and ethylene glycol. However, (35 L) to give a value of 2800 mg, or 2.8 g. Because
if the child had ingested ethylene glycol, she would only about one-fourth of a half-life has passed since
be expected to exhibit a significant osmolar gap. The ingestion, the amount eliminated since that time will
anion gap, lethargy, tachypnea, and hyperthermia all be rather small. The answer is D.
are consistent with aspirin poisoning. The answer is 9.
B. The short-acting β blocker esmolol helps reverse the
4. tachycardia and possibly the vasodilation associated
Glucagon (Chapter 41) stimulates heart rate and with an overdose of theophylline. The answer is C.
contractility through cardiac glucagon receptors that 10.
are coupled to adenylyl cyclase and the cAMP Neither gastric lavage nor syrup of ipecac should be
signaling pathway. This ability to increase cardiac used in patients who have ingested a corrosive
cAMP without requiring access to β receptors makes because of the risk of esophageal damage. Gastric
it valuable for β-blocker overdose. The answer is C. lavage can be used in a comatose patient if the
5. airway has been protected with a cuffed
In this woman with severe signs of poisoning due to endotracheal tube. The answer is C.
the ingestion of metformin, hemodialysis can be
used to accelerate the elimination of both QUESTIONS
metformin and lactic acid. Since most of the 1. A 55-year-old woman with type 1 diabetes of 40
metformin has been absorbed by the time she years’ duration complains of severe bloating and
presented (9 h after drug ingestion), efforts to abdominal distress, especially after meals.
decontaminate her gastrointestinal tract with Evaluation is consistent with diabetic gastroparesis.
activated charcoal, gastric lavage, or syrup of ipecac Which of the following is a prokinetic drug that could
are unlikely to be beneficial. Furthermore, syrup of be used in this situation?
ipecac has fallen out of favor and should not be used (A) Alosetron
in unconscious patients. Unlike other drugs used to (B) Cimetidine
treat type 2 diabetes, metformin in overdose is (C) Loperamide
unlikely to cause hypoglycemia (see Chapter 41), and (D) Metoclopramide
this patient’s serum glucose is in the normal range so (E) Sucralfate
that glucagon administration is not required. The 2. A patient who is taking verapamil for hypertension
answer is E. and angina has become constipated. Which of the
6. following drugs is an osmotic laxative that could be
Tricyclic antidepressants such as amitriptyline are used to treat the patient’s constipation?
extremely toxic in overdose because of their effects (A) Aluminum hydroxide
in the CNS and cardiovascular system. In addition to (B) Diphenoxylate
hypotension, seizures, and cardiac arrhythmias, the (C) Magnesium hydroxide
tricyclics have strong antimuscarinic effects. The (D) Metoclopramide
answer is B. (E) Ranitidine
7. 3. A 40-year-old male CEO came to the emergency
Estimations of the time period required for drug or department with severe burning chest pain radiating
toxin elimination may be of value in the into his neck. His electrocardiogram was normal and
management of the poisoned patient. If no test for troponin was negative. A diagnosis of GERD
procedures were used to hasten the elimination of was made and he was sent home with a prescription
for a drug that inhibits stomach acid. Which of the (B) Methotrexate
following is a drug that irreversibly inhibits the (C) Misoprostol
H+/K+ ATPase in the parietal cells? (D) Ondansetron
(A) Cimetidine (E) Sucralfate
(B) Diphenoxylate Questions 9 and 10. The following matching
(C) Esomeprazole questions consist of a list of lettered options
(D) Metoclopramide followed by several numbered items. For each
(E) Sulfasalazine numbered item, select the ONE option that is most
4. Which drug is most likely to be useful in the closely associated with it.
treatment of inflammatory bowel disease? (A) Aluminum hydroxide
(A) Diphenhydramine (B) Balsalazide
(B) Diphenoxylate (C) Castor oil
(C) Mesalamine (D) Cimetidine
(D) Ondansetron (E) Dexamethasone
(E) Ursodiol (F) Methotrexate
5. A 34-year-old woman has irritable bowel (G) Metoclopramide
syndrome with diarrhea that is not responsive to (H) Mineral oil
conventional therapies. Despite the small risk of (I) Omeprazole
severe constipation and ischemic colitis, the patient (J) Linaclotide
decides to begin therapy with alosetron. Alosetron (K) Pancrelipase
has which of the following receptor actions? (L) Sucralfate
(A) 5-HT3 receptor antagonist
(B) 5-HT4 receptor agonist 9. Which drug stimulates chloride secretion into the
(C) D2 receptor antagonist gut lumen and is used for irritable bowel syndrome?
(D) NK1 receptor antagonist 10. This is a small molecule that polymerizes in
(E) Muscarinic receptor antagonist stomach acid and coats the ulcer bed, resulting in
6. On your way to an examination, you experience accelerated healing and reduction of symptoms.
the vulnerable feeling that an attack of diarrhea is ANSWERS
imminent. If you stopped at a drugstore, which one 1.
of the following antidiarrheal drugs could you buy Of the drugs listed, only metoclopramide is
without a prescription even though it is related considered a prokinetic agent (ie, one that increases
chemically to the strong opioid analgesic propulsive motility in the gut). The answer is D.
meperidine? 2.
(A) Aluminum hydroxide A laxative that mildly stimulates the gut would be
(B) Diphenoxylate most suitable in a patient taking a smooth muscle
(C) Loperamide relaxant drug such as verapamil. By holding water in
(D) Magnesium hydroxide the intestine, magnesium hydroxide provides
(E) Metoclopramide additional bulk and stimulates increased
7. A 45-year-old man with a duodenal ulcer was contractions. A helpful mnemonic is magnesium
treated with a combination of drugs intended to heal “magnifies” stool, aluminum hALts the stool. The
the mucosal damage and to eradicate Helicobacter answer is C.
pylori. Which of the following antibacterial drugs is 3.
used commonly to eradicate intestinal H pylori? Esomeprazole, the (S) isomer of omeprazole, is a
(A) Cefazolin prodrug converting spontaneously in the parietal cell
(B) Ciprofloxacin canaliculus to a sulfonamide that irreversibly
(C) Clarithromycin inactivates the proton pump. The answer is C.
(D) Clindamycin 4.
(E) Vancomycin Mesalamine is a form of 5-aminosalicylic acid that is
8. A patient is receiving highly emetogenic active in the large intestine and thereby provides a
chemotherapy for metastatic carcinoma. To prevent local anti-inflammatory effect that is useful in
chemotherapy-induced nausea and vomiting, she is inflammatory bowel disease. The answer is C.
likely to be treated with which of the following? 5.
(A) Levodopa
Serotonin plays a major regulatory role in the enteric can take anything to prevent that. You tell him there
nervous system, and the potent 5-HT3 receptor is an endogenous antioxidant that may help. Some
antagonist alosetron has shown efficacy in treating clinical trials suggest that patients with statin-
women with IBS that is accompanied by diarrhea. associated myopathy had reduced muscle pain after
The answer is A. receiving this supplement. The supplement is which
6. of the following?
Aluminum hydroxide is constipating but is not (A) Coenzyme Q10
related chemically to meperidine; magnesium (B) Glucosamine
hydroxide is a strong laxative. The 2 antidiarrheal (C) Melatonin
drugs that are structurally related to opioids are (D) Tyrosine
diphenoxylate and loperamide. Loperamide is (E) Vitamin E
available over-the-counter; diphenoxylate is mixed 3. Which drug has a biochemical effect that most
with atropine alkaloids, and the product (Lomotil, closely resembles the proposed mechanism of action
others) requires a prescription. The answer is C. of the psychoactive constituent(s) of St. John’s wort?
7. (A) Alprazolam
The macrolide antibiotic clarithromycin is commonly (B) Fluoxetine
used in antibiotic regimens designed to treat (C) Levodopa
duodenal ulcers caused by H pylori. The other (D) Methylphenidate
antibiotics that are used include amoxicillin, (E) Morphine
tetracycline, and metronidazole. Bismuth also has an 4. A 67-year-old male reports difficulty starting a
antibacterial action. The answer is C. urine stream and feeling an urge to urinate again
8. soon after urinating. Examination confirms benign
The 5-HT3 receptor antagonists are highly effective prostatic hyperplasia (BPH). He prefers not to take
at preventing chemotherapy-induced nausea and prescription medications. Which of the following is
vomiting, which can be a dose-limiting toxicity of an alternative medicine that is commonly used to
anticancer drugs. The answer is D. treat the urinary symptoms associated with BPH?
9. (A) Echinacea
Linaclotide is approved for the treatment of chronic (B) Ephedra
constipation and IBS with predominant constipation. (C) Ginseng
Linaclotide activates guanylyl cyclase-C on the (D) Milk thistle
luminal intestinal epithelial surface, which leads to (E) Saw palmetto
activation of the cystic fibrosis transmembrane 5. Which of the following is a derivative of serotonin
conductance regulator (CFTR) leading to increased that may have value in managing symptoms of jet
chloride-rich secretion and acceleration of intestinal lag?
transit. The answer is J. (A) Ephedra
10. (B) Garlic
Sucralfate is a small molecule that polymerizes in (C) Ginseng
stomach acid and forms a protective coat over the (D) Glucosamine
ulcer bed. The answer is L. (E) Melatonin
6. Which compound enhances immune function in
QUESTIONS vitro and is used to decrease the symptoms of the
1. A patient has accidentally ingested mushrooms common cold?
identified as Amanita phalloides. Which herbal (A) Echinacea
substance is claimed to protect against hepatic (B) Ginkgo
dysfunction? (C) Garlic
(A) Echinacea (D) Melatonin
(B) Ginkgo (E) Milk thistle
(C) Melatonin 7. Rejection of heart transplants has occurred in
(D) Milk thistle patients being treated with standard doses of
(E) Saw palmetto cyclosporine when they also used which of the
2. Your patient, a 45-year-old male, requires following dietary supplements?
atorvastatin. He enjoys lifting weights and is worried (A) Echinacea
about statin-induced muscle pain. He asks you if he (B) Ginkgo
(C) Milk thistle tricyclic antidepressants and SSRIs such as
(D) St. John’s wort fluoxetine. The answer is B.
(E) Saw palmetto
8. In 2003, a study published in the Annals of 4.
Internal Medicine found that this botanical Saw palmetto, a complex extract from the berries of
substance accounted for more than 60% of adverse Serenoa repens or Sabal serrulata, is widely
events associated with dietary supplements used in purported to improve the symptoms of BPH. The
the United States. The “herbal” in question, which is answer is E.
used to aid weight loss and promote sports 5.
performance, is which of the following? Garlic might get you a row of seats to yourself, but
(A) Echinacea the compound that will help in jet lag is melatonin.
(B) Ephedra The answer is E.
(C) Ginkgo 6.
(D) Ginseng The freshly pressed juice of the aerial parts of
(E) Saw palmetto Echinacea purpurea is purported to reduce the
9. Which of the following is a popular supplement symptoms of the common cold and the time of
whose purported efficacy in osteoarthritis is recovery if ingested within 24 h of onset. The answer
believed to be due to its role as a precursor to the is A.
glycosaminoglycans that form joint cartilage? 7.
(A) Coenzyme Q10 St. John’s wort induces the formation of hepatic
(B) Dehydroepiandrosterone (DHEA) enzymes that metabolize cyclosporine, and its use
(C) Glucosamine can decrease the effectiveness of the
(D) Nicotinic acid immunosuppressant drug in organ and tissue
(E) Melatonin transplantation. The answer is D.
10. Couples who are attempting to conceive a child 8.
should avoid chronic use of which of the following? Concern about the risks of using products containing
(A) Echinacea ephedra during heavy workouts or in diet programs
(B) Ephedra that stress the cardiovascular system has led to a
(C) Ginkgo ban on such nutritional supplements in the United
(D) Ginseng States. The answer is B.
(E) Melatonin 9.
ANSWERS The amino sugar glucosamine, a building block for
1. glycosaminoglycans, has become popular among
Milk thistle contains compounds that may have people with osteoarthritis of the knee. The answer is
cytoprotective actions against liver toxins, including C.
those present in Amanita mushrooms. The answer is 10.
D. Chronic use of melatonin appears to suppress LH
2. secretion in women and to decrease sperm quality in
HMG-CoA reductase, the enzyme required for men. The answer is E.
coenzyme Q10 synthesis, is inhibited by statins (see
Chapter 35). This inhibition may contribute to statin- QUESTIONS
associated myopathy. A small double-blind clinical 1. A 55-year-old patient currently receiving a drug
trial found a significant reduction in muscle pain in for a psychiatric condition is to be started on diuretic
patients with statin-associated myopathy who were therapy for mild heart failure. Consideration should
treated with coenzyme Q10. The answer is A. be given to the fact that thiazides are known to
3. reduce the excretion of which of the following?
Extracts of the flowers of St. John’s wort contain (A) Diazepam
chemicals with possible antidepressant activity. In (B) Fluoxetine
vitro studies have shown that these chemicals (C) Imipramine
interfere with the neuronal reuptake of amine (D) Lithium
neurotransmitters in a fashion similar to the (E) Trifluoperazine
proposed mechanism of antidepressant actions of 2. A hypertensive patient has been using nifedipine
for some time without untoward effects. If he
experiences a rapidly developing enhancement of
the antihypertensive effect of the drug, it is most 7. Which drug has resulted in severe hematotoxicity
likely due to which of the following? when administered to a patient being treated with
(A) Concomitant use of antacids azathioprine?
(B) Foods containing tyramine (A) Allopurinol
(C) Furanocoumarins in grapefruit juice (B) Cholestyramine
(D) Induction of drug metabolism (C) Digoxin
(E) Over-the-counter decongestants (D) Lithium
3. A patient suffering from a depressive disorder is (E) Theophylline
being treated with imipramine. If he uses Questions 8–10. The following section consists of a
diphenhydramine for allergic rhinitis, a drug list of lettered options followed by several
interaction is likely to occur because numbered items. For each numbered item, select
(A) Both drugs block muscarinic receptors the ONE option that is most closely associated with
(B) Both drugs block reuptake of norepinephrine it.
released from sympathetic nerve endings (A) Allopurinol
(C) Diphenhydramine inhibits imipramine (B) Carbamazepine
metabolism (C) Cholestyramine
(D) Imipramine inhibits the metabolism of (D) Cimetidine
diphenhydramine (E) Clarithromycin
(E) The drugs compete with each other for renal (F) Cyclosporine
elimination (G) Digoxin
4. If phenelzine is administered to a patient taking (H) Erythromycin
fluoxetine, which of the following is most likely to (I) Fluoxetine
occur? (J) Ibuprofen
(A) A decrease in the plasma levels of fluoxetine (K) Lovastatin
(B) Antagonism of the antidepressant action of (L) Phenelzine
fluoxetine (M) Rifampin
(C) Agitation, muscle rigidity, hyperthermia, seizures (N) Ritonavir
(D) Decreased metabolism of fluoxetine (O) Theophylline
(E) Priapism 8. In patients with HIV infection, the inhibitory action
5. AJ is a 45-year-old homeless man participating in a of this agent on drug metabolism has clinical value.
drug rehabilitation program that supplies daily 9. This drug enhances the toxicity of methotrexate
methadone. AJ reports that he needs more by decreasing its renal clearance.
methadone to avoid withdrawal, since he started 10. Concomitant use of St. John’s wort is reported to
treatment for his tuberculosis. Which of the increase the effectiveness of this drug.
following drugs might have caused this scenario? ANSWERS
(A) Ciprofloxacin 1.
(B) Cyclosporine Thiazides reduce the clearance of lithium by about
(C) Erythromycin 25%. They do not alter the clearance of the other
(D) Rifampin agents listed. The answer is D.
(E) Tetracycline 2.
6. A 43-year-old woman with type 2 diabetes and Compounds in grapefruit juice can increase the rate
hypertension visits your clinic. Her current blood and extent of bioavailability of several
pressure (measured 3 times) is not at target: 155/98 dihydropyridine calcium channel blockers, including
mm Hg, despite the fact that she is taking felodipine and nifedipine. This interaction may be
hydrochlorothiazide and captopril. The due to inhibition of the metabolism of the
antihypertensive effects of captopril can be dihydropyridines by intestinal wall CYP3A4 or
antagonized (reduced) by which of the following? inhibition of the P-glycoprotein transporter in the
(A) Angiotensin II receptor blockers same location. The answer is C.
(B) Loop diuretics 3.
(C) NSAIDs This is a good example of an additive drug
(D) Sulfonylurea hypoglycemics interaction resulting from 2 drugs acting on the
(E) Pioglitazone same type of receptor. Most tricyclic
antidepressants, phenothiazines, and older In contrast, the herb decreases the effectiveness of
antihistaminic drugs (those available without other drugs (including cyclosporine, estrogens, and
prescription) are blockers of muscarinic receptors. protease inhibitors) via its induction of drug-
Used concomitantly, any pair of these agents will metabolizing enzymes. The answer is I.
demonstrate a predictable increase in atropine-like
adverse effects. The answer is A.
4.
The drug interaction between the inhibitors of
monoamine oxidase used for depression and the
drugs that selectively block serotonin reuptake
(SSRIs) is called the serotonin syndrome. In the case
of phenelzine and fluoxetine, the interaction has
resulted in a fatal outcome. Key interventions
include control of hyperthermia and seizures. The
answer is C.
5.
Rifampin is an effective inducer of hepatic P450
isozymes. Cyclosporine and tetracycline have no
significant effects on drug metabolism. Ciprofloxacin
and erythromycin are inhibitors of drug metabolism.
The answer is D.
6.
NSAIDs interfere with the antihypertensive action of
angiotensin-converting enzyme inhibitors; the other
drugs listed enhance the blood pressure-lowering
effects of captopril and other members of the “pril”
drug family. Pioglitazone is a hypoglycemic drug
used in patients with type 2 diabetes mellitus and
has no significant effect on blood pressure. The
answer is C.
7.
Azathioprine is converted to mercaptopurine, which
is responsible for both its immunosuppressant action
and its hematotoxicity. Allopurinol inhibits xanthine
oxidase, the enzyme that metabolizes
mercaptopurine. The answer is A.
8.
Ritonavir inhibits the metabolism of other HIV
protease inhibitors and is used in low-dose
combinations with indinavir or lopinavir. The answer
is N.
9.
Several NSAIDs, including aspirin, ibuprofen, and
piroxicam, increase serum levels of methotrexate by
interfering with its renal clearance. The adverse
effects of methotrexate, including its hematotoxicity,
are predictably increased. The answer is J.

10.
Concomitant use of St. John’s wort enhances the
effects of selective serotonin reuptake inhibitors.
Note that enhancement is not always positive and
too much serotonin is harmful (serotonin syndrome).

Potrebbero piacerti anche